0% found this document useful (0 votes)
842 views372 pages

FR Part 1 (Chapter 1-16) (Jun 2023)

Uploaded by

najihah Anual
Copyright
© © All Rights Reserved
We take content rights seriously. If you suspect this is your content, claim it here.
Available Formats
Download as PDF, TXT or read online on Scribd
0% found this document useful (0 votes)
842 views372 pages

FR Part 1 (Chapter 1-16) (Jun 2023)

Uploaded by

najihah Anual
Copyright
© © All Rights Reserved
We take content rights seriously. If you suspect this is your content, claim it here.
Available Formats
Download as PDF, TXT or read online on Scribd

ACCA Financial Reporting (FR)

ACCA-FR
FINANCIAL REPORTING
Sunway TES

STUDY GUIDE
(Student Copy)
PART 1

June 2023
ACCA-FR: FINANCIAL REPORTING
TABLE OF CONTENTS

Table of Contents

CHAPTER 1: CONCEPTUAL FRAMEWORK......................................................................................................... 7


1. LEARNING OUTCOME.............................................................................................. 7
1.1 Overview ........................................................................................................................ 8
1.2 Scope of Conceptual Framework...................................................................................... 9
1.3 The Qualitative Characteristic of Useful Financial Information ......................................... 12
1.4 Understanding of Financial Statement............................................................................ 15
1.5 Measurement ............................................................................................................... 20
1.6 Concepts of Capital and Capital Maintenance ................................................................. 24
1.7 Check Understanding .................................................................................................... 26
1.8 Check Understanding: Answer ....................................................................................... 29
1.9 Chapter 1 Summary....................................................................................................... 31
CHAPTER 2: REGULATORY FRAMEWORK ....................................................................................................... 32
2. LEARNING OUTCOME ........................................................................................... 32
2.1 The Needs for Regulatory Framework ............................................................................ 33
2.2 Functions of Various Accounting Bodies ......................................................................... 36
2.3 Understanding of National Standard Setting ................................................................... 39
Check Understanding ................................................................................................................ 41
Check Understanding - Answer .............................................................................................. 43
2.4 Chapter 2 Summary....................................................................................................... 44
CHAPTER 3: IAS 1 PRESENTATION OF FINANCIAL STATEMENTS .................................................................. 45
3. LEARNING OUTCOME ........................................................................................... 45
3.1 Overall Consideration .................................................................................................... 46
3.2 Components of Financial Statements ............................................................................. 48
3.3 Identification of Financial Statements ............................................................................ 49
3.4 Statement of Financial Position (SOFP) ........................................................................... 50
3.5 Statement of Profit or Loss & Other Comprehensive Income (SOPL & OCI) ....................... 53
3.6 Statement of Changes in Equity (SOCE) .......................................................................... 55
3.7 Statement of Cash Flow (SOCF) ...................................................................................... 56
3.8 Notes to the Financial Statements.................................................................................. 56
3.9 Check Understanding .................................................................................................... 57
3.10 Chapter 3 Summary....................................................................................................... 61
CHAPTER 4: IAS 16 PROPERTY, PLANT AND EQUIPMENT ............................................................................. 62
4. LEARNING OUTCOME ........................................................................................... 62
Overview.................................................................................................................................. 63
4.1 Introduction ................................................................................................................. 63
4.2 Recognition Criteria....................................................................................................... 65
4.3 Measurement Criteria ................................................................................................... 67
4.4 Depreciation ................................................................................................................. 74
4.5 Depreciation Methods................................................................................................... 76

2
ACCA-FR: FINANCIAL REPORTING
TABLE OF CONTENTS

4.6 Subsequent measurement (at year end) ......................................................................... 78


4.7 De-recognition of PPE.................................................................................................... 90
4.8 Disclosure..................................................................................................................... 90
4.9 Check Understanding .................................................................................................... 93
4.10 Chapter 4 Summary....................................................................................................... 98
CHAPTER 5: IAS 38 INTANGIBLE ASSETS.......................................................................................................... 99
5. LEARNING OUTCOME ........................................................................................... 99
5.1 Overview .....................................................................................................................100
5.2 Scope of IAS 38 ............................................................................................................102
5.3 Recognition .................................................................................................................102
5.4 Measurement ..............................................................................................................106
5.5 Different ways of obtaining intangible assets .................................................................111
5.6 Amortisation................................................................................................................118
5.7 De-recognition .............................................................................................................121
5.8 Disclosure....................................................................................................................121
5.9 Check Understanding ...................................................................................................122
5.10 Chapter 5 Summary......................................................................................................126
CHAPTER 6: IAS 36 IMPAIRMENT OF ASSETS ............................................................................................... 127
6. LEARNING OUTCOME ......................................................................................... 127
6.1 Overview .....................................................................................................................128
6.2 Identifying an asset that may be impaired .....................................................................130
6.3 Measuring recoverable amount ....................................................................................131
6.4 Measuring an impairment loss ......................................................................................134
6.5 Recognising an impairment loss ....................................................................................135
6.6 Cash-generating units...................................................................................................136
6.7 Reversal of Impairment Loss .........................................................................................142
6.8 Check Understanding ...................................................................................................143
6.9 Chapter 6 Summary......................................................................................................150
CHAPTER 7: IAS 20 GOVERNMENT GRANT & DISCLOSURE OF GOVERNMENT ASSISTANCE .................. 151
7. LEARNING OUTCOME ......................................................................................... 151
7.1 Overview .....................................................................................................................152
7.2 Purpose of government assistance ................................................................................154
7.3 Accounting treatment of Government Grants ................................................................154
7.4 Repayment of Government Grants ................................................................................159
7.5 Disclosure....................................................................................................................161
7.6 Check Understanding ...................................................................................................161
7.7 Chapter 7 Summary......................................................................................................163
CHAPTER 8: IAS 23 BORROWING COST ......................................................................................................... 164
8. LEARNING OUTCOME ......................................................................................... 164
8.1 Overview .....................................................................................................................165
8.2 Recognition .................................................................................................................167

3
ACCA-FR: FINANCIAL REPORTING
TABLE OF CONTENTS

8.3 Measurement ..............................................................................................................168


8.4 Period of Capitalisation ................................................................................................170
8.5 Disclosure....................................................................................................................172
8.6 Check Understanding ...................................................................................................173
8.7 Chapter 8 Summary......................................................................................................174
CHAPTER 9: IAS 40 INVESTMENT PROPERTY ................................................................................................ 175
9. LEARNING OUTCOME ......................................................................................... 175
Overview.................................................................................................................................176
9.1 Relational Mapping ......................................................................................................176
9.2 Definition ....................................................................................................................177
9.3 Classification of Property ..............................................................................................178
9.4 Recognition Criteria......................................................................................................179
9.5 Conceptual Framework ................................................................................................179
9.6 Measurement Criteria ..................................................................................................180
9.7 Transfer.......................................................................................................................183
9.8 De-recognition .............................................................................................................184
9.9 Disclosures ..................................................................................................................184
9.10 Check Understanding ...................................................................................................185
9.11 Chapter 9 Summary......................................................................................................191
CHAPTER 10: IAS 37 PROVISIONS, CONTINGENT LIABILITIES & CONTINGENT ASSETS ........................... 192
10. LEARNING OUTCOME ......................................................................................... 192
10.1 Overview .....................................................................................................................193
10.2 Definition ....................................................................................................................194
10.3 Recognition .................................................................................................................195
10.4 Contingent Liabilities ....................................................................................................198
10.5 Contingent Assets ........................................................................................................200
10.6 Measurement ..............................................................................................................201
10.7 Reimbursement ...........................................................................................................207
10.8 Changes in provisions and judgement required ..............................................................208
10.9 Specific applications .....................................................................................................208
10.10 Disclosures ..................................................................................................................214
10.11 Check Understanding ...................................................................................................221
10.12 Chapter 10 Summary....................................................................................................224
CHAPTER 11: IAS 10 EVENTS AFTER THE REPORTING PERIOD .................................................................... 225
11. LEARNING OUTCOME ......................................................................................... 225
11.1 Overview .....................................................................................................................226
11.2 Events after the reporting period ..................................................................................226
11.3 Recognition and measurement .....................................................................................227
11.4 Adjusting events after the reporting period ...................................................................227
11.5 Non-adjusting events after the reporting period ............................................................230
11.6 Dividends ....................................................................................................................230

4
ACCA-FR: FINANCIAL REPORTING
TABLE OF CONTENTS

11.7 Going Concern .............................................................................................................231


11.8 Disclosure....................................................................................................................232
11.9 Check Understanding ...................................................................................................233
11.10 Chapter 11 Summary....................................................................................................237
CHAPTER 12: IAS 8 ACCOUNTING POLICIES, CHANGES IN ACCOUNTING ESTIMATES & ERRORS .......... 238
12. LEARNING OUTCOME ......................................................................................... 238
12.1 Overview .....................................................................................................................239
12.2 Accounting Policies ......................................................................................................240
12.3 Consistency of Accounting Policies ................................................................................241
12.4 Changes in Accounting Estimates ..................................................................................244
12.5 Prior Period Errors........................................................................................................246
12.6 Check Understanding ...................................................................................................248
12.7 Chapter 12 Summary....................................................................................................254
CHAPTER 13: IAS 33 EARNINGS PER SHARE .................................................................................................. 255
13. LEARNING OUTCOME ......................................................................................... 255
13.1 Overview .....................................................................................................................256
13.2 Needs for IAS 33 ..........................................................................................................257
13.3 Basic EPS .....................................................................................................................257
13.4 Situation involving a Change in the number of Ordinary Shares outstanding during the year
259
13.5 Diluted EPS.......................................................................................................................264
13.5 Check Understanding ...................................................................................................269
13.6 Chapter 13 Summary....................................................................................................283
CHAPTER 14: IFRS 3 BUSINESS COMBINATION ............................................................................................ 284
14. LEARNING OUTCOME ......................................................................................... 284
14.1 Concept of a group.......................................................................................................285
14.2 Accounting principles underlying group accounts ...........................................................285
14.3 Control and power .......................................................................................................286
14.4 Types of group structures .............................................................................................288
14.5 Composition of group accounts.....................................................................................288
14.6 Non-controlling interest ...............................................................................................292
14.7 Issue of related parties .................................................................................................292
14.8 Goodwill ......................................................................................................................293
14.9 Cost of Investment .......................................................................................................295
14.10 Calculation of Fair Value of Net Asset Acquired ..............................................................299
14.11 Check Understanding ...................................................................................................300
14.12 Chapter 14 Summary....................................................................................................305
CHAPTER 15: CONSOLIDATED FINANCIAL STATEMENTS ............................................................................ 306
15. LEARNING OUTCOME ......................................................................................... 306
15.1 Consolidated Statement of Financial Position ................................................................307
15.2 Consolidated Statement of Profit and Loss ....................................................................330

5
ACCA-FR: FINANCIAL REPORTING
TABLE OF CONTENTS

15.3 Disposal of Subsidiary...................................................................................................336


15.4 Check Understanding ...................................................................................................337
15.5 Chapter 15 Summary....................................................................................................356
CHAPTER 16: IAS 28 INVESTMENT IN ASSOCIATES ...................................................................................... 357
16. LEARNING OUTCOME ......................................................................................... 357
16.1 Overview .....................................................................................................................358
16.2 Formation of associate companies ................................................................................358
16.3 Equity method .............................................................................................................359
16.4 Adjustment..................................................................................................................360
16.5 Steps in preparing Consolidated Statement of Financial Position.....................................362
16.6 Check Understanding ...................................................................................................364
16.7 Chapter 16 Summary....................................................................................................371

6
ACCA-FR: FINANCIAL REPORTING
CHAPTER 1: CONCEPTUAL FRAMEWORK

CHAPTER 1: CONCEPTUAL
FRAMEWORK

LEARNING OUTCOME
At the end of the chapter, you should be able to:

TLO A1a. Describe what is meant by a conceptual framework for financial reporting

TLO A1b. Discuss whether a conceptual framework is necessary and what an alternative system
might be

TLO A1c. Discuss what is meant by relevance and faithful representation and describe the
qualities that enhance these characteristics

TLO A1d. Discuss whether faithful representation constitutes more than compliance with
accounting standards

TLO A1e. Discuss what is meant by understandability and verifiability in relation to the provision
of financial information

TLO A1f. Discuss the importance of comparability and timeliness to users of financial
statements

TLO A1g. Discuss the principle of comparability in accounting for changes in accounting policies

TLO A2a. Define what is meant by ‘recognition’ in financial statements and discuss the
recognition criteria
TLO A2b. Apply the recognition criteria to:
I) assets and liabilities.
II) income and expenses.

TLO A2c. Explain and compute amounts using the following measures:
I) historical cost
II) current cost
III) value in use
IV) fair value
TLO A2d. Discuss the advantages and disadvantages of historical cost accounting

TLO A2e. Discuss whether the use of current value accounting overcomes the problems of
historical cost accounting

TLO A2f. Describe the concept of financial and physical capital maintenance and how this
affects the determination of profits

7
ACCA-FR: FINANCIAL REPORTING
CHAPTER 1: CONCEPTUAL FRAMEWORK

1.1 Overview

Learning Outcome (ACCA Study Guide Area A)


A1a: Describe what is meant by a conceptual framework for financial reporting
A1b: Discuss whether a conceptual framework is necessary and what an alternative system might
be

Regulation in financial reporting practice is important to ensure an entity adopts accounting treatment
that is consistent in preparing and presenting financial statements, which provide information that is
useful to a wide range of users, to assists their decisions making.

Regulatory bodies set accounting standards to ensure that entities adopt similar accounting
treatments for similar items and account for similar transactions in the same way over time. This makes
it possible to compare the financial statements of different entities and to compare an entity’s
performance for the current year with its performance in previous years.

Without regulation, management would adopt whichever accounting treatment presented its results
and position in the best possible light. Sometimes management might deliberately mislead users of
the financial statements.

Diagram 1.1: Overview of conceptual framework

8
ACCA-FR: FINANCIAL REPORTING
CHAPTER 1: CONCEPTUAL FRAMEWORK

1.1.1 The Conceptual Framework


15.1.1
In March 2018, the International Accounting Standards Board (IASB) finished its revision of The
Framework for Financial Reporting (the Framework).

The primary purpose of financial information is to be useful to existing and potential investors,
lenders and other creditors (users) when making decisions about the financing of the entity and
exercising rights to vote on, or otherwise influence, management’s actions that affect the use of the
entity’s economic resources.

The purpose of the Framework is to:


• assist the IASB to develop and revise its standards
• assist entities to develop consistent accounting policies when no standard applies to a particular
transaction or other event, or when a standard allows a choice of accounting policy, and
• assist all stakeholders to understand and interpret the standards
IFRSs take precedence over the Framework. However, should new IFRSs depart from the Framework,
the IASB will explain the reasons in the Basis for Conclusions on that standard.

1.2 Scope of Conceptual Framework

1.2.1 Scope of IASB’s Conceptual Framework

The revised 2018 conceptual framework covers 8 area:


Diagram 1.2.1: Conceptual framework for financial reporting

9
ACCA-FR: FINANCIAL REPORTING
CHAPTER 1: CONCEPTUAL FRAMEWORK

1.2.2 The Objective of Financial Reporting

Diagram 1.2.2: Objective of financial reporting

10
ACCA-FR: FINANCIAL REPORTING
CHAPTER 1: CONCEPTUAL FRAMEWORK

1.2.3. The needs for adopting IFRS

• Widely accepted as a set of high-quality and transparent global standards to achieve consistency
and comparability across the world

• Produced with co-operation of other internationally renowned standard setters, with the
aspiration of achieving consensus and global convergence

• Companies that uses IFRS and have their FS audited in accordance to IFRS will have an enhanced
status and reputation

• The International Organisation of Securities Commission (IOSCO) recognize IFRS for listing
purposes – thus companies that adopt IFRS need to produce only one set of FS for any securities
listing for countries that are members of IOSCO – hence easier and cheaper to raise finance in
international markets

• Companies with foreign subsidiaries will find process of consolidation simplified if all subsidiaries
adopt IFRS. It enables accounting staff to be transferred between group of companies in different
countries.

• Companies that adopts IFRS find their FS easily comparable with other companies who also
adopts IFRS. This could obviate the need for any reconstruction from local GAAP to IFRS when
analyst assess performance.

1.2.4 IFRS vs US GAAP

Besides IFRS, General Accepted Accounting Principles (GAAP) also become one of reference for
company reporting preparation. GAAP refer to a common set of accepted accounting principles,
standards, and procedures that companies and their accountants must follow when they compile their
financial statements. The comparison between IFRS and GAAP are as follow:

Table 1.2.4: Comparison between IFRS and US GAAP

IFRS US GAAP
Based on description with series of detailed
Formulated accordance to principles sets in CF
rules
Principle based system Rule based system
Relies on generally accepted accounting
principles and exercises judgment in dealing Less flexible (rules and requirements lengthy
with transactions and in applying the principles and complex) but often more comparable
to different industries
Disadvantage: Accountants and auditors expect to find
• Inconsistencies between reporting entities specific rules to cover every situation in every
• Manipulation of FS industry that they are involved in. Hence, less
• Burden on accountants and auditors burden.

11
ACCA-FR: FINANCIAL REPORTING
CHAPTER 1: CONCEPTUAL FRAMEWORK

1.3 The Qualitative Characteristic of Useful Financial Information

Learning Outcome (ACCA Study Guide Area A)


A1c: Discuss what is meant by relevance and faithful representation and describe the qualities
that enhance these characteristics
A1d: Discuss whether faithful representation constitutes more than compliance with accounting
standards
A1e: Discuss what is meant by understandability and verifiability in relation to the provision of
financial information
A1f: Discuss the importance of comparability and timeliness to users of financial statements

Financial information that is useful should be relevant and faithfully represent what it purports to
represent.

The usefulness of financial information is further enhanced if it is comparable, verifiable, timely and
understandable.

Diagram 1.3: Qualitative Characteristics

12
ACCA-FR: FINANCIAL REPORTING
CHAPTER 1: CONCEPTUAL FRAMEWORK

1.3.1 Fundamental qualitative characteristics


15.1.2

Qualitative characteristics are the attributes that make information provided in financial statements
useful to others.

Relevance

Has the ability to influence the economic decision of users, where information that is relevant has
both predictive and confirmatory values:

Table [Link]: Relevance value

Predictive value Confirmatory value


• Helps users to predict future outcomes • Helps users to confirm or correct
previous evaluations and assessments

Materiality has a direct impact on the relevance of information.

Information is material if omitting, misstating or obscuring it could reasonably be expected to


influence decisions that the primary users of general purpose financial statements make on the basis
of those financial statements, which provide financial information about a specific reporting entity.

Materiality is an entity-specific aspect of relevance, based on the nature or magnitude (or both) of
the items to which the information relates in the context of an individual entity’s financial report.

Faithful Representation (formerly known as reliability)

To be useful, financial information must not only be relevant, it must also represent faithfully the
phenomena it purports to represent. Faithful representation means representation of the substance
of an economic phenomenon instead of representation of its legal form only (Substance over form).

A faithful representation is, to the maximum extent possible, complete, neutral and free from error.

a. Complete
To ensure all impacts of transactions must be recorded and not omitted with necessary
narrations

b. Neutral
Free from bias – FS will not be considered as neutral if specific information was deliberately
selected and presented in order to achieve a predetermined result or outcome

Neutrality is supported by the exercise of prudence. Prudence is the exercise of caution when
making judgements under conditions of uncertainty. Prudence does not allow for
overstatement or understatement of assets, liabilities, income or expenses.

c. Free from error (Accuracy)


Free from material misstatement or omission as material error, or either an omission or
misstatement can cause the financial statements to be false or misleading.

13
ACCA-FR: FINANCIAL REPORTING
CHAPTER 1: CONCEPTUAL FRAMEWORK

Faithful representation is affected by measurement uncertainty. Measurement uncertainty does not


prevent information from being useful. For example, in some cases, relevant information may have a
high level of measurement uncertainty, which may reduce its usefulness. Slightly less relevant
information with a lower measurement uncertainty may be preferable in such cases.

1.3.2 Enhancing qualitative characteristics

To improve the usefulness of information that is relevant and faithfully represented

Comparability

Enables users to identify and understand similarities in, and differences among, items. To ensure
information is comparable, there must be:

• consistent application of accounting policies from one period to the next;


• disclosure of accounting policies being applied so that users able to distinguish between different
accounting policies in order to be able to make a valid comparison of similar items in the accounts
of different entities.

Timeliness

Having information available for decision makers in time (timely information), capable of influencing
their economic decisions. As a general rule, older information is less useful than recent information.

Verifiability

Information is verifiable (capable of being verify) in the sense that it should ensure credibility and
objectivity. It requires that independent observers reach the same or similar conclusions that:

• not biased or contains material errors and


• recognition of the chosen method of assessment is applied free from material error and
subjectivity

Understandability

Classifying, characterising and presenting information clearly and concisely makes it understandable.
While some phenomena are inherently complex and cannot be made easy to understand, to exclude
such information would make financial reports incomplete and potentially misleading.

Financial reports are prepared for users who have a reasonable knowledge of business and economic
activities and who review and analyse the information with diligence.

Enhancing qualitative characteristics should be maximised to the extent necessary. However,


enhancing qualitative characteristics (either individually or collectively) cannot render information
useful if that information is irrelevant or not represented faithfully.

14
ACCA-FR: FINANCIAL REPORTING
CHAPTER 1: CONCEPTUAL FRAMEWORK

Cost Constraint on Financial Reporting

Cost is a pervasive constraint on the information that can be provided by general purpose financial
reporting and it is therefore important to determine whether the benefits to users of the information
justify the cost incurred by the entity providing it.

1.4 Understanding of Financial Statement

Learning Outcome (ACCA Study Guide Area A)


A2a: Define what is meant by ‘recognition’ in financial statements and discuss the recognition
criteria
A2b: Apply the recognition criteria to:
I) assets and liabilities.
II) income and expenses.

1.4.1 Objective and scope of financial statements


15.1.3

The objective of financial statements is to provide information about an entity's assets, liabilities,
equity, income and expenses that is useful to financial statements users in assessing the prospects
for future net cash inflows to the entity and in assessing management's stewardship of the entity's
economic resources.

When considering the objective of general-purpose financial reporting, the Board reintroduced the
concept of ‘stewardship’. The importance of stewardship by management is inherent within the
existing Framework and within financial reporting, so this statement largely reinforces what already
exists.

Users base their expectations of returns on their assessment of:


▪ the amount, timing and uncertainty of future net cash inflows to the entity, and
▪ management’s stewardship of the entity’s resources.

This information is provided in the: -

1. statement of financial position;


2. the statement(s) of financial performance; and
3. other statements and notes.

15
ACCA-FR: FINANCIAL REPORTING
CHAPTER 1: CONCEPTUAL FRAMEWORK

Diagram 1.4.1: Scope of financial statement

1.4.2 Underlying Assumption


15.1.4
The Conceptual Framework of Accounting mentions the underlying assumption of going concern. In
addition, the concepts of accrual, accounting entity, monetary unit, and time period are also
important in preparing and interpreting financial statements.

Going concern basis

Going concern is referred to by the IASB’s Conceptual Framework as the 'underlying assumption'

The accrual basis of accounting assumes that an entity is a going concern. Under this basis, financial
statements are prepared on the assumption that the entity will continue in operation for the
foreseeable future (that is 12 months after the reporting date), in that management has neither the
intention nor the need to liquidate the entity by selling all its assets, paying off all its liabilities and
distributing any surplus to the owners.

Examples of the importance of the Going Concern basis are as follows:


a. The measurement of receivables from trade customers is made on the basis that there is no time
limit over which management will chase slow payers. If the entity were to cease operation in, say,
three months, a number of balances might have to be regarded as bad (irrecoverable) debts.

b. The measurement of non-current assets is made on the basis that they can be utilised throughout
their planned life. Otherwise, they would have to be valued at what they could immediately be
sold for, which might not be very much, in the case of assets used in markets where there is excess
capacity.

16
ACCA-FR: FINANCIAL REPORTING
CHAPTER 1: CONCEPTUAL FRAMEWORK

Break-up basis

One of the key assumptions made under the accruals basis is that the business will continue as a going
concern. However, this will not necessarily always be the case. There may be an intention or need to
sell off the assets of the business. Such a sale typically arises where the business is in financial
difficulties and needs the cash to pay its creditors.

Where this is the case an alternative method of accounting must be used (in accordance with IAS 1
Presentation of Financial Statements). In these circumstances the financial statements will be
prepared on a basis other than going concern, which is commonly referred to as the ‘break-up’ basis.
The break-up basis values assets and liabilities today as if the entity was about to cease trading and
had to dispose of all its assets and liabilities.

The effect of this is seen primarily in the statement of financial position as follows:
a. Classification of assets
All assets and liabilities would be classified as current rather than non-current.

b. Valuation of assets
Assets would be valued on the basis of the recoverable amount on sale. This is likely to be
substantially lower than the carrying amount of assets held under historical cost accounting.

1.4.3 Reporting entity


15.1.5
A reporting entity is an entity that is required, or chooses, to prepare financial statements. A reporting
entity is not necessarily a legal entity. It can comprise:-

• A single entity – for example, one company;


• A portion of an entity – for example, a division of one company;
• More than one entities – for example, a parent and its subsidiaries reporting as a group.

As a result, we have a few types of financial statements:

• Consolidated: a parent and subsidiaries report as a single reporting entity;


• Unconsolidated: e.g. a parent alone provides reports, or
• Combined: e.g. reporting entity comprises two or more entities that are not all linked by
parent-subsidiary relationship.

Generally, consolidated financial statements are more likely to provide useful information to users of
financial statements than unconsolidated financial statements.

Therefore, it is important to determine the boundaries of a reporting entity. Determining the


appropriate boundary of a reporting entity is driven by the information needs of the primary users of
the reporting entity’s financial statements.

17
ACCA-FR: FINANCIAL REPORTING
CHAPTER 1: CONCEPTUAL FRAMEWORK

1.4.4 Elements of Financial Statements


15.1.6
i. Asset and Liability (Statement of Financial Position)

Diagram 1.4.3(a): Elements of financial statement

*No practical ability to avoid


The ‘no practical ability to avoid’ criterion is applied in the following circumstances:
(a) if a duty or responsibility arises from the entity’s customary practices, published policies or
specific statements—the entity has an obligation if it has no practical ability to act in a manner
inconsistent with those practices, policies or statements.

(b) if a duty or responsibility is conditional on a particular future action that the entity itself may
take—the entity has an obligation if it has no practical ability to avoid taking that action

ii. Income and Expenses (Statement of Financial Performance)

Diagram 1.4.3(b): Elements of financial statement

18
ACCA-FR: FINANCIAL REPORTING
CHAPTER 1: CONCEPTUAL FRAMEWORK

1.4.5 Recognition
15.1.7

Only items that meet the definition of an asset, a liability or equity are recognised in the statement of
financial position and only items that meet the definition of income or expenses are to be recognised
in the statement of financial performance.

Not all items that meet the definition of one of the elements listed above are recognized in the
financial statements. The Framework requires recognising the elements only when the recognition
provides useful information – relevant with faithful representation, because the aim is to provide
information that is useful to investors, lenders and other creditors.

A key change to this is the removal of a ‘probability criterion’. This has been removed as different
financial reporting standards apply different criterion; for example, some apply probable, some
virtually certain and some reasonably possible. This also means that it will not specifically prohibit the
recognition of assets or liabilities with a low probability of an inflow or outflow of economic resources.

We can summarise the recognition criteria for assets, liabilities, income and expenses, based on the
definition of recognition given above:

Diagram 1.4.4: Recognition criteria

19
ACCA-FR: FINANCIAL REPORTING
CHAPTER 1: CONCEPTUAL FRAMEWORK

1.4.6 Derecognition
15.1.8
Derecognition means removal of all or part of a recognised asset or liability from an entity’s statement
of financial position. Derecognition normally occurs when: -

▪ For an asset, when the entity loses control of all or part of the recognised asset
▪ For a liability, when the entity no longer has a present obligation for all or part of the
recognised liability

Derecognition should aim to faithfully represent both:


▪ assets and liabilities retained after the transaction that led to the derecognition
▪ the change in assets and liabilities as a result of the transaction that led to the derecognition

1.5 Measurement

Learning Outcome (ACCA Study Guide Area A)


A2c: Explain and compute amounts using the following measures:
I) historical cost
II) current cost
III) value in use
IV) fair value
A2d: Discuss the advantages and disadvantages of historical cost accounting
A2e: Discuss whether the use of current value accounting overcomes the problems of historical
cost accounting
A2f: Describe the concept of financial and physical capital maintenance and how this affects the
determination of profits

The Framework discusses two basic measurement basis:

1.5.1 Historical cost


15.1.9
This measurement is based on the transaction price at the time of recognition of the element. For an
asset, this would be the cost incurred in acquiring/creating the asset. For a liability, this would be the
value of the consideration received to incur/take on the liability.

The historical cost of both an asset and a liability should be adjusted over time to reflect the usage (in
the form of depreciation or amortisation). For example, historical cost of assets is reduced if they
become impaired and historical cost of liabilities is increased if they become onerous.

20
ACCA-FR: FINANCIAL REPORTING
CHAPTER 1: CONCEPTUAL FRAMEWORK

1.5.2 Current value

It measures the element updated to reflect the conditions at the measurement date. Here, several
methods are included:

• Fair value;
• Value in use;
• Current cost.

Fair value

Fair value is the price that would be received to sell an asset, or paid to transfer a liability, in an orderly
transaction between market participants at the measurement date (in line with IFRS 13 Fair Value
Measurement).

It reflects market participants’ current expectations about the amount, timing and uncertainty of
future cash flows.

Value in use (for assets) or Fulfilment value (for liabilities)

Value in use is the present value of future cash flows that an entity expects to derive from the use of
an asset and from its ultimate disposal.

It reflects entity-specific current expectations about the amount, timing and uncertainty of future
cash flow.

Fulfilment value is the present value of future cash that an entity expects to be obliged to transfer as
it fulfils a liability.

Current cost

Current cost reflects the current amount that would be:

• paid to acquire an equivalent asset; or


• received to take on an equivalent liability

Current cost is different from fair value and value in use, as current cost is an entry value. This looks
at the value in which the entity would acquire the asset (or incur the liability) at current market prices,
whereas fair value and value in use are exit values, focusing on the values which will be gained from
the item.

21
ACCA-FR: FINANCIAL REPORTING
CHAPTER 1: CONCEPTUAL FRAMEWORK

Example 1.5 (Current Value)


ACE Co bought a factory building for $200,000 on 1 Jan 20X5 and it is now 31 Dec 20X8. A third party
approached ACE Co. offering to buy the factory building at $250,000. Currently, a similar factory
building in that area is available at $280,000. The management accountant has analysed the income
generating ability of the factory building over an estimated remaining useful life of 10 years as follows:

Year Discounted future cash flow ($)


1 38,000
2 35,000
3 32,000
4 25,000
5-10 20,000
150,000

What should be the amount recorded in SOFP as at 31 Dec 20X8 under each of the valuation bases?

i. Historical costs : $200,00


ii. Fair value : $250,000
iii. Current cost : $280,000
iv. Value in use : $150,000

1.5.3 Advantages and Disadvantages of Historical Cost Accounting


15.1.10
Diagram 1.5.3: Advantages and disadvantages of historical cost accounting

22
ACCA-FR: FINANCIAL REPORTING
CHAPTER 1: CONCEPTUAL FRAMEWORK

From these disadvantages, various issues arise:

Undervalued of assets will depress a company's share price and make it vulnerable to takeover. In
practice, listed companies avoid this by revaluing land and buildings in line with market values.

Understated depreciation and understated cost of sales lead to overstatement of profits compounded
by price inflation.

Overstated profits can lead to:


• too much dividends being distributed to shareholders, leaving insufficient amounts for
investment
• shareholders to expect higher dividends and employees to demand higher wages
• overstated tax bills

1.5.4 Presentation and Disclosure


15.1.11

According to IAS 1 (presentation of financial statement), an entity should meet the fair presentation
requirement which require it to be comply with IFRS standard. Financial statements should meet fair
presentation of the financial position, financial performance and cash flows of an entity.

Fair presentation requires the faithful representation of the effects of transactions, other events and
conditions in accordamce with the definations and recognition criteria for assets, liabilities, income
and expenses set out in the Conceptual Framework.

IAS 1 highligted following principles for presentation of financial statement:

• Compliance with IFRS should be disclosed


• All relevant IFRS must be followed if compliance with IFRS is disclosed
• Use of an inappropriate accounting treatment cannot be rectified either by disclosure of
accounting policies or notes/explanatory material

IAS 1 states what is required for a fair presentation:

• Selection and application of accounting policies


• Presentation of information in manner which provides relevant, realible, comparable and
understandable information
• Additional disclosure where required

23
ACCA-FR: FINANCIAL REPORTING
CHAPTER 1: CONCEPTUAL FRAMEWORK

1.6 Concepts of Capital and Capital Maintenance

The concept of capital maintenance is concerned with how an entity defines the capital that it seeks
to maintain. It provides the linkage between the concepts of capital and the concepts of profit because
it provides the point of reference by which profit is measured.

Diagram 1.6: Concepts of capital

The selection of the appropriate concept of capital by an entity should be based on the needs of the
users of its FS.

Thus, a financial concept of capital should be adopted if the users of FS are primarily concerned with
the maintenance of nominal invested capital. If, however, the main concern of users is with the
operating capability of the entity, a physical concept of capital should be used.

Example 1.6 (Capital Maintenance)


Consider the following opening and closing SOFP of a company:

Opening Closing
$ $
Inventory (100 items at cost) 500 600
Other net assets 1000 1000
Capital 1500 1600

Assuming that no new capital has been introduced during the year, and no capital has been distributed
as dividends, for financial capital maintenance concept, the profit would be $100, being the excess
of closing capital over opening capital. And yet in physical terms the company is no better off: it still
has 100 units of inventory (which cost $5 each at the beginning of the period, but $6 each at the end)
and its other net assets are identical.

24
ACCA-FR: FINANCIAL REPORTING
CHAPTER 1: CONCEPTUAL FRAMEWORK

For physical capital maintenance concept, no profit would be recognised because the physical
substance of the company is unchanged over the accounting period. Capital is maintained if at the end
of the period the company is in a position to achieve the same physical output as it was at the
beginning of the period.

1.6.1 Current Cost Accounting (CCA)


15.1.12
CCA is based on physical concept of capital maintenance. Profit is recognised after the operating
capability of the business has been maintained.

Based on the same scenario above, assuming that no new capital has been introduced during the year,
and no capital has been distributed as dividends, the profit shown in historical cost accounts would
be $100, being the excess of closing capital over opening capital.

And yet in physical terms the company is no better off: it still has 100 units of inventory (which cost
$5 each at the beginning of the period, but $6 each at the end) and its other net assets are identical.
The 'profit' earned has merely enabled the company to keep pace with inflation.

1.6.2 Advantages and Disadvantages of CCA


15.1.13

Advantages of CCA Disadvantages of CCA


a) By excluding holding gains from profit, CCA can a) It is impossible to make valuations of Economic
be used to indicate whether the dividends paid Value or NRV without subjective judgements.
The measurements used are therefore not
to shareholders will reduce the operating
objective.
capability of the business. b) There are several problems to be overcome in
b) Assets are valued after management has deciding how to provide an estimate of
considered the opportunity cost of holding replacement costs for non-current assets.
them, and the expected benefits from their c) The mixed value approach to valuation means
future use. CCA is therefore a useful guide for that some assets will be valued at replacement
management in deciding whether to hold or cost, but others will be valued at net realisable
sell assets. value or economic value. It is arguable that the
c) It is relevant to the needs of information users total assets will, therefore, have an aggregate
in: value which is not particularly meaningful
- Assessing the stability of the business entity because of this mixture of different concepts.
- Assessing the vulnerability of the business (eg
to a takeover), or the liquidity of the business
- Evaluating the performance of management in
maintaining and increasing the business
substance
- Judging future prospects

25
ACCA-FR: FINANCIAL REPORTING
CHAPTER 1: CONCEPTUAL FRAMEWORK

1.7 Check Understanding

Learning Outcome (ACCA Study Guide Area A)


A1a: Describe what is meant by a conceptual framework for financial reporting
A1b: Discuss whether a conceptual framework is necessary and what an alternative system might
be
A1c: Discuss what is meant by relevance and faithful representation and describe the qualities that
enhance these characteristics
A1d: Discuss whether faithful representation constitutes more than compliance with accounting
standards
A1e: Describe what is meant by a conceptual framework for financial reporting
A1f: Discuss the importance of comparability and timeliness to users of financial statements
A1g: Discuss the principle of comparability in accounting for changes in accounting policies
A2a: Define what is meant by ‘recognition’ in financial statements and discuss the recognition
criteria
A2b: Apply the recognition criteria to:
I) assets and liabilities
II) income and expenses
A2c: Explain and compute amounts using the following measures:
I) historical cost
II) current cost
III) value in use
IV) fair value
A2d: Discuss the advantages and disadvantages of historical cost accounting
A2e: Discuss whether the use of current value accounting overcomes the problems of historical
cost accounting
A2e: Describe the concept of financial and physical capital maintenance and how this affects the
determination of profits

Question 1
Describe what is meant by a conceptual framework

Question 2
Explain the main reasons for having a conceptual framework

Question 3
Explain the purpose of the Conceptual Framework

Question 4
Discuss the extent to which IFRS are relevant to not-for-profit entities

26
ACCA-FR: FINANCIAL REPORTING
CHAPTER 1: CONCEPTUAL FRAMEWORK

Question 5
According to the IASB's Conceptual Framework for Financial Reporting, which TWO of the following
are part of faithful representation?

I. It is neutral
II. It is relevant
III. It is presented fairly
IV. It is free from material error

A. I and II
B. II and III
C. I and IV
D. III and IV

Question 6
Listed below are some characteristics of financial information.
I. Relevance
II. Comparability
III. Faithful representation
IV. Timeliness

Which of these are fundamental characteristics, according to the IASB's Conceptual Framework for
Financial Reporting?

A. I and II only
B. II and IV only
C. III and IV only
D. I and III only

Question 7
Capital maintenance means that:
A. The non-current assets of the business are maintained in an excellent state of repair
B. All assets of the business are maintained in an excellent state of repair
C. The owner has not taken home any of the business assets
D. The money capital at the end of the accounting period is the same as at the beginning

Question 8
The IASB’s Framework recognize a liability as:
A. Obligations to pay which may arise depending on some future events
B. Amounts a business may have to pay after the balance sheet date
C. Present obligations to transfer an economic resource as a result of past events
D. Unpaid portion of expenses incurred in the current accounting period

27
ACCA-FR: FINANCIAL REPORTING
CHAPTER 1: CONCEPTUAL FRAMEWORK

Question 9
Which of the following is NOT advantage of historical cost accounting?
A. Amount recorded are reliable since it based on original invoices
B. Lead to understatement of assets
C. Easily understood
D. Less opportunity for creative accounting

Question 10
Schrute owns a herd of cattle, which produce milk. Schrute then turns this into cheese. On 1 April
20X5, Shrute purchased a flock of sheep for $100,000, which included transaction costs of $5,000. At
31 March 20X6, the flock was valued at $120,000. Every time animals are sold there is a 5%
commission fee payable to the national farming agency.

Shrute uses the historical cost model and charges all depreciation as an operating expense. In addition
to this, Schrute uses a number of items of specialised farm machinery. This mach inery cost Schrute
$200,000 on 1 April 20X2 and has a 10-year useful life. At 31 March 20X6, there is only one supplier
who still sells this machinery and the current price of new machinery is $300,000.

Using current cost accounting, what is the value of the machinery at 31 March 20X6?
A. $120,000
B. $180,000
C. $200,000
D. $300,000

28
ACCA-FR: FINANCIAL REPORTING
CHAPTER 1: CONCEPTUAL FRAMEWORK

1.8 Check Understanding: Answer

Question 1
A conceptual framework could be defined as a coherent system of interrelated objectives and
fundamental principles. It is a statement of generally accepted theoretical principles which form the
frame of reference for Financial Reporting. As the purpose of financial reporting is to provide useful
information as a basis for economic decision making, a conceptual framework will form a theoretical
basis for determining how transactions should be measured (historical value or current value) and
reported (ie: how they are presented or communicated to users).

Question 2
"It enables accounting standards and GAAP to be developed in accordance with agreed prin ciples
and underlying assumptions and concepts. Therefore, avoids 'firefighting', whereby accounting
standards are developed in a piecemeal way in response to specific problems or abuses. Such an
approach can lead to inconsistencies between different accounting standards and also between
accounting standards and relevant local legislation."

The lack of a conceptual framework may mean that certain critical issues are not addressed. For
example, until the Framework for preparation and presentation of financial statements was published
there was no definition of basic terms such as 'asset' or 'liability' in any accounting standard which is
obviously fundamental to a consistent treatment of accounting transactions and events.

In a world where transactions have become more complex and businesses more sophisticated an
overall, conceptual framework can help preparers of financial statements and their auditors deal with
complex transactions and particularly those which are not the subject of an accounting standard .

Question 3
The stated purpose of the Framework is to:
▪ assist the IASB to develop IFRS Standards based on consistent concepts, resulting in financial
information that is useful to investors, lenders and other creditors
▪ assist preparers of financial reports to develop consistent accounting policies for transactions
or other events when no Standard applies or a Standard allows a choice of accounting policies
▪ assist all parties to understand and interpret

Question 4
The main aim of not-for-profit entities is to provide value for money rather than making a profit. Value
for money is achieved by a combination of effectiveness, efficiency and economy.

Effectiveness means achieving the objectives (usually non-monetary) of the organization. The
objectives of not-for-profit and public sector entities will differ depending upon the type of entity. For
example, a school may have the objectives of teaching a certain number of children and achieving
certain academic standards. A hospital may have the objectives of treating out-patients within a
particular time scale or minimising the number of empty beds. Effectiveness is therefore measured by
identifiable measures of achievement in reaching those goals or objectives.

29
ACCA-FR: FINANCIAL REPORTING
CHAPTER 1: CONCEPTUAL FRAMEWORK

Efficiency means using the resources available well. It is effectively the quantity of output obtained
for a given measure of input. Efficiency means getting more out of fewer inputs and thereby reducing
the cost of output. In a school it might be measured by the pupil to teacher ratio and in a hospital by
the number of patients seen by a consultant during a surgery.

Economy means keeping the cost of input resources as low as possible. This is achieved by paying
less for the inputs that are required to meet the objectives or provide the service. In a school giving
more teaching time to classroom assistants rather than higher paid teachers would be a form of
economy or in a hospital scheduling duties to a nurse rather than a doctor.

Not-for-profit and public sector organizations do not aim to achieve a profit but will have to account
for their income and costs. Such entities will have to account for their effectiveness, economy and
efficiency even if they do not have to produce financial statements for the public (although in many
cases may do so). Therefore, some measurement accounting standards will be relevant such as those
relating to inventory, non-current assets, leasing etc. However, others relating purely to reporting
such as earnings per share will not be so relevant.

Question 5
C

Question 6
D

Question 7
D

Question 8
C

Question 9
B

Question 10
B

30
ACCA Financial Reporting (FR)
CHAPTER 1: CONCEPTUAL FRAMEWORK

1.9 Chapter 1 Summary


Diagram 1.9: Summary of Conceptual Framework

31
ACCA Financial Reporting (FR)
CHAPTER 2: REGULATORY FRAMEWORK

CHAPTER 2: REGULATORY FRAMEWORK


LEARNING OUTCOME
At the end of the chapter, you should be able to:
TLO A3a. Explain why a regulatory framework is needed including the advantages and disadvantages
of IFRS over a national regulatory framework

TLO A3b. Explain why accounting standards on their own are not a complete regulatory framework

TLO A3c. Distinguish between a principle-based and a rules-based framework and discuss whether
they can be complementary

TLO A3d. Describe the IASB’s Standard setting process including revisions to and interpretations of
Standards

TLO A3e. Explain the relationship of national standard setters to the IASB in respect of the standard
setting process

TLO A3f. Explain the purpose and role of the International Sustainability Standards Board (ISSB)

32
ACCA Financial Reporting (FR)
CHAPTER 2: REGULATORY FRAMEWORK

2.1 The Needs for Regulatory Framework

Learning Outcome (ACCA Study Guide Area A)


A3a: Explain why a regulatory framework is needed including the advantages and disadvantages of
IFRS over a national regulatory framework
A3b: Explain why accounting standards on their own are not a complete regulatory framework
A3c: Distinguish between a principles based and a rules based framework and discuss whether they
can be complementary
A3d: Describe the standard setting process of the International Accounting Standards Board (IASB)
including revisions to and interpretations of Standards
A3f: Explain the purpose and role of the International Sustainability Standards Board (ISSB™)

Regulation in financial reporting practice is important to ensure an entity adopts accounting treatment
that is consistent in preparing and presenting financial statements that provide information that is useful
to a wide range of users, to assists their decisions making.

Regulatory bodies set accounting standards to ensure that entities adopt similar accounting treatments
for similar items and account for similar transactions in the same way over time. This makes it possible to
compare the financial statements of different entities and to compare an entity’s performance for the
current year with its performance in previous years.

2.1.1 Purpose of a Regulatory Framework

Diagram 2.1.1: Regulatory Framework Overview

A regulatory framework is required to ensure that relevant and reliable financial reporting is achieved to
meet the needs of shareholders and other users.

33
ACCA Financial Reporting (FR)
CHAPTER 2: REGULATORY FRAMEWORK

Accounting standards alone would not be a complete regulatory framework. In order to fully regulate the
preparation of financial statements and the obligations of companies and directors, legal and market
regulations are also required.

Without regulation, management would adopt whichever accounting treatment presented its results and
position in the best possible light. Sometimes management might deliberately mislead users of the
financial statements.

2.1.2 Advantages and disadvantages of IFRS


15.1.14
Diagram 2.1.2: Advantages and disadvantages of IFRS

34
ACCA Financial Reporting (FR)
CHAPTER 2: REGULATORY FRAMEWORK

2.1.3 Sources of Regulation


15.1.15
The main sources of regulation are:

• Accounting standards
• Company law
• For listed companies, the listing rules of the relevant Stock Exchange

Accounting standard are authoritative statements of how particular types of transactions and events are
reflected in the financial statements. In some countries they have legal authority, but in most countries
they do not have the force of law.

Company law varies from country to country, but typically it sets out rules for determining profits available
for distribution, issuing and redeeming share capital, the reserves that a company must have and the uses
to which they can be put. In some countries example, in the European Union, company law prescrib es the
format of the main financial statement. These matters are not covered in accounting standards.

Listing rules set out the information which entities must supply when they apply for a listing. They also set
out the information, including financial reports, which entities must prepare and provide to the market
while they are listed.

2.1.4 Principles-based and rules-based framework


15.1.16
Diagram 2.1.4: Principles-based and rules-based framework

35
ACCA Financial Reporting (FR)
CHAPTER 2: REGULATORY FRAMEWORK

Conceptual Framework lies under principle-based system. It provides basic principles within the standards
can developed. The main intention is to avoid any conflict in the standard set with each other. In general
principles-based system works within a set of principles set.

However, for Rules-based it regulates for issues as they arise. In absence of reporting framework, more
rules-based approach has to be adopted. This leads to a large mass of regulation designed to cover every
eventuality, as in the US. As we have seen over the past few years, a large volume of regulatory measures
does not always detect or prevent financial irregularity. One presumed advantage of rules-based systems
is that the exercise of judgement is minimised. Auditors who fear litigation tend to prefer rules -based
systems. It could be that a rules-based approach is appropriate for controversial areas in accounting.

2.2 Functions of Various Accounting Bodies

2.2.1International Financial Reporting Standard Foundation (IFRSF)


Foundation
The
15.1.17
International Financial Reporting Standard (IFRS) Foundation formerly known as the International
Accounting Standard Committee Foundation was formed in March 2001 as a not-for-profit corporation
and was the parent entity of the IASB.

In 2010 it was renamed as The IFRS Foundation is an independent organization and its trustees exercise
oversight and raise necessary funding for the IASB to carry out its role as standard-setter. It also oversees
the work of the IFRS Interpretations Committee (formerly called the International Financial Reporting
Interpretations Committee (IFRIC)) and the IFRS Advisory Council (formerly called the Standards Advisory
Council (SAC)).

The IFRS Foundation and its independent standard-setting bodies, the International Accounting Standards
Board (IASB) and the newly created International Sustainability Standards Board (ISSB) , provide public
accountability through the transparency of their work, the consultation with the full range of interested
parties in the standard setting process, and their formal accountability links to the public.

Main objectives of the IFRSF are:

• Develop, in the public interest, a single set of high quality global accounting standards
• Promote the use and rigorous application of those standards
• To take account of the special needs of small & medium sized entities and emerging economies
• To bring about the convergence of national accounting standards and the IASs

2.2.2 International Accounting Standard Board (IASB)


15.1.18
The IASB is responsible for developing international accounting standards. Members of IASB have a variety
of backgrounds and include auditors, preparers of financial statements, users of financial statements and
academics.

36
ACCA Financial Reporting (FR)
CHAPTER 2: REGULATORY FRAMEWORK

Main objectives of the IASB are:

• To develop, in the public interest, a single set of high-quality, understandable, enforceable and globally
accepted financial reporting standards based on clearly articulated principles. These standards should
require high quality, transparent and comparable information in financial statements and other
financial reporting to help investors, other participants in the various capital markets of the world and
other users of the information to make economic decisions.
• To promote the use and rigorous application of those standards
• In fulfilling the above objectives to take account of, as appropriate, the needs of a range of sizes and
types of entities in diverse economic settings
• To promote and facilitate the adoption of IFRS through the convergence of national accounting
standards and IFRSs.

2.2.3 IFRS Interpretation Committee (IFRSIC)


15.1.19
The role of IFRS Interpretations Committee is to issue rapid guidance where there are differing possible
interpretations of an IAS. Its role is therefore to:

• Interpret IAS & IFRS


• Issue timely guidance on issues not covered by an IAS or IFRS, within the context of the IASB Framework
• Publish draft Interpretations for public comment. After studying responses to the draft Interpretation,
it will obtain IASB approval for a final (published Interpretation.)

2.2.4 IFRS Advisory Council (IFRSAC)


15.1.20
The Advisory Council (AC) provides a forum through which the IASB and ISSB are able to gather opinions
and advice from different countries and industries. The AC consists of experts from different countries
and different business sectors, who offer advice to the IASB and ISSB.

2.2.5 International Sustainability Standards Board (ISSB)

In response to overwhelming demand for global sustainability standards, on 3 November 2021, the IFRS
Foundation established the ISSB which aims to develop a comprehensive global baseline of sustainability
disclosures for the financial markets, the IFRS Sustainability Disclosure Standards. IFRS Sustainability Disclosure
Standards set out how a company discloses information about sustainability-related factors that may help or
hinder a company in creating value.

The ISSB has set out four key objectives:


1. to develop standards for a global baseline of sustainability disclosures
2. to meet the information needs of investors
3. to enable companies to provide comprehensive sustainability information to global capital markets; and

37
ACCA Financial Reporting (FR)
CHAPTER 2: REGULATORY FRAMEWORK

4. to facilitate interoperability with disclosures that are jurisdiction-specific and/or aimed at broader
stakeholder groups

IASB and ISSB work closely to ensure their both the IAS or IFRS and IFRS Sustanainability Disclosure Standards
complement each other to provide investors with transparent and reliable information about a company’s
financial position and performance, as well as information about sustainability factors that could create or
erode its enteprise value in the short, medium and long term.

2.2.5 International Organisation of Securities Commissions (IOSCO)


15.1.21

The prominence of the IASB has been enhanced by its relationship with the International Organization of
Securities Commissions (IOSCO). IOSCO is an influential organization of the world's security commissions
(stock exchanges). In 1995 the IFRSF agreed to develop a core set of standards which, when endorsed by
IOSCO, would be used as an acceptable basis for cross-border listings. In May 2005, this was achieved. As
part of its harmonization process, the European Union requires listed companies in all member states to
prepare consolidated FS using IFRS from 2005.
Main objectives of the IOSCO are:

The main objective of IOSCO is to develop international investment. IOSCO views that international
investment will be encouraged if all major companies use the same accounting standards for reporting
their financial position and performance.

38
ACCA Financial Reporting (FR)
CHAPTER 2: REGULATORY FRAMEWORK

2.2.6 International Accounting Standards Setting Process


15.1.22
Diagram 2.2.6: International Accounting Standards Setting Process

The enforcement of IAS is in the hands of the regulatory systems of the individual adopting countries as
IASB has no power to enforce international accounting standards within those countries/ companies that
choose to adopt them.

2.3 Understanding of National Standard Setting

Learning Outcome (ACCA Study Guide Area A)


A3e: Explain the relationship of national standard setters to the IASB in respect of the standard
setting process

When the new Constitution of the IASC Foundation was set up in 2001, the IASB became th e body
responsible for:

• Developing and publishing accounting standards as IFRS and


• Approving and publishing Interpretations of IFRS.

Before the new Constitution was established, standards have been published as IAS and Interpretations
were published as Standard Interpretative Committee (SIC) Interpretations.

The IASB decided that all IAS and SIC that had been issued previously would continue to be applicable,
unless they are subsequently amended or withdrawn. This means that IAS still in issue have the sam e
status as IFRS.

39
ACCA Financial Reporting (FR)
CHAPTER 2: REGULATORY FRAMEWORK

2.3.4 National Standard Setters and IASB


15.1.23
The IASB has no power to enforce its standards. Without the support of the major national standard
setters, IFRS are unlikely to be adopted.

There are increasing calls for international convergence. International convergence means that the
accounting standards of different countries are increasingly similar. As most major national standard
setting bodies are represented on the IASB, they can contribute significantly in influencing the
development of new standards in their own countries. Most national standard setters are committed to
the principle of international convergence.

In addition, the IASB has been working with national standard setters on specific projects. For example:

• The IASB and the UK ASB worked together to develop IAS 36 Impairment of Assets and IAS 37
Provision, contingent liabilities and contingent assets
• Some other recent standards have been developed by a group of national standard setters and
the IASB working together.
• The IASB Framework was originally based on work carried out by the US FASB; the Framework has
in turn influenced the UK ASB in developing its own conceptual framework
• National standard setters sometimes carry out the research for IASB projects. For eg, the Canadian
ASB has carried out work on measurement objectives and have issued a discussion paper on this
topic.

2.3.5 US Financial Accounting Standard Board


15.1.24
The US Financial Accounting Standard Boards (FASB has a special role in developing new international
standards. The IASB and the US FASB are currently working together in the following ways:

• A short term convergence project aims to reduce differences between certain IAS & IFRS and
certain US standards. The IASB has issued IFRS 5 Non-Current Assets held for sale & Discontinued
Operations as a result of this project.
• Several joint projects are in progress to develop new standards on business combinations,
revenue recognition and performance reporting.
• The 2 standard setters are jointly developing a new conceptual framework. This will eventually
replace the current IASB Framework.

40
ACCA Financial Reporting (FR)
CHAPTER 2: REGULATORY FRAMEWORK

Check Understanding

Learning Outcome (ACCA Study Guide Area A)


A3a: Explain why a regulatory framework is needed including the advantages and disadvantages of
IFRS over a national regulatory framework
A3b: Explain why accounting standards on their own are not a complete regulatory framework
A3c: Distinguish between a principles based and a rules based framework and discuss whether they
can be complementary
A3d: Describe the IASB’s Standard setting process including revisions to and interpretations of
Standards
A3e: Explain the relationship of national standard setters to the IASB in respect of the standard
setting process
A3f: Explain the purpose and role of the International Sustainability Standards Board (ISSB™)

Question 1
Identify three (3) advantages of IFRS implementation.

Question 2
Principles-based framework is set based on ___________________ while rules-based framework is
based on _________________.

Question 3
The process for developing an International Financial Reporting Standard involves a number of stages.

Following receipt and review of comments on a Discussion Paper, what will be the next step undertaken
by the IASB?

A. Publication of an Exposure Draft


B. Establishment of an Advisory Committee
C. Consultation with the Advisory Committee
D. Issue of a final IFRS

Question 4
“Accounting standards alone would not be a complete regulatory framework. In order to fully regulate
the preparation of financial statements and the obligations of companies and directors, legal and market
regulations are also required.”

Identify statement above whether it is true or false.

True

False

41
ACCA Financial Reporting (FR)
CHAPTER 2: REGULATORY FRAMEWORK

Question 5
The _____________ of accounting standards refers to the goal of establishing a single set of accounting
standards that will be used internationally.

A. Divergence
B. Merging
C. Convergence
D. Coherence

42
ACCA Financial Reporting (FR)
CHAPTER 2: REGULATORY FRAMEWORK

Check Understanding - Answer

Question 1
1. Entities applying IFRS are better prepared to access the capital markets
2. Reduce administrative burdens for entities that form part of an international group
3. Increase transparency

Question 2
Principles-based framework is set based on principles guideline while rules-based framework is based
on issues arise.

Question 3
A

Question
True

Question 5
C

43
ACCA Financial Reporting (FR)
CHAPTER 2: REGULATORY FRAMEWORK

2.4 Chapter 2 Summary


Diagram 2.4: Summary of Regulatory Framework

44
ACCA Financial Reporting (FR)
CHAPTER 3: IAS 1 PRESENTATION OF FINANCIAL STATEMENTS

CHAPTER 3: IAS 1 PRESENTATION OF


FINANCIAL STATEMENTS
LEARNING OUTCOME
At the end of the chapter, you should be able to:

TLO D1a. Prepare an entity’s statement of financial position and statement of profit or loss and
other comprehensive income in accordance with the structure and content prescribed
within IFRS and with accounting treatments as identified within syllabus areas A, B
and C

TLO D1b. Prepare and explain the contents and purpose of the statement of changes in equity

45
ACCA Financial Reporting (FR)
CHAPTER 3: IAS 1 PRESENTATION OF FINANCIAL STATEMENTS

3.1 Overall Consideration

IAS 1 applies to all general purpose financial statements prepared in accordance with IFRS, that is, those
intended to meet the needs of users who are not in a position to demand reports tailored to their specific
needs. It prescribes the basis for the presentation of financial statements, so as to ensure comparability
with:

• The entity’s own financial statements of previous periods, and


• The financial statements of other entities

IAS 1 is concerned with the overall considerations about the minimum content of a set of financial
statements; detailed rules about recognition, measurement.

Diagram 3.1 Overview IAS 1

3.1.1 Fair Presentation

Financial statements shall present fairly the financial position, financial performance and cash flows of an
entity. Fair presentation requires the faithful representation of the effects of transactions, other events
and conditions in accordance with the definitions and recognition criteria for assets, liabilities, income
and expenses set out in the Framework.

The application of IFRS, with additional disclosure when necessary, is presumed to result in financial
statements that achieve a fair presentation.

46
ACCA Financial Reporting (FR)
CHAPTER 3: IAS 1 PRESENTATION OF FINANCIAL STATEMENTS

3.1.2 Offsetting
15.1.25
Assets and liabilities, and income and expenses, may not be offset unless required or permitted by an
IFRS. Income and expenses can be offset only when:

• An IFRS requires/permits it, or


• Gains, losses and related expenses arising from the same/similar transactions are not material (in
aggregate).

3.1.3 Going Concern


15.1.26
When preparing financial statements, management shall make an assessment of an entity's ability to
continue as a going concern. Financial statements shall be prepared on a going concern basis unless
management either intends to liquidate the entity or to cease trading, or has no realistic alternative but
to do so.

When management is aware, in making its assessment, of material uncertainties related to events or
conditions that may cast significant doubt upon the entity's ability to continue as a going concern, those
uncertainties shall be disclosed. When financial statements are not prepared on a going concern basis,
that fact shall be disclosed, together with the basis on which the financial statements are prepared and
the reason why the entity is not regarded as a going concern.

3.1.4 Comparative Information


15.1.27
An entity shall disclose comparative information in respect of the previous period for all amounts reported
in the Financial Statement, both on the face of the Financial Statement and in the notes, unless another
Standard requires otherwise.

* A third Statement of Financial Position is required to be presented if the entity retrospectively applies
an accounting policy, restates items, or reclassifies items, and those adjustments had a material effect on
the information in the Statement of Financial Position at the beginning of the comparative period.

3.1.5 Consistency of Preparation


15.1.28
The presentation and classification of items in the financial statements shall be retained from one
period to the next unless:

• Resulted in more appropriate application of accounting policies (more reliable & relevance
information
• Required by new IFRS

3.1.6 Frequency of Reporting


15.1.29
An entity shall present a complete set of financial statements (including comparative information) at
least annually.

47
ACCA Financial Reporting (FR)
CHAPTER 3: IAS 1 PRESENTATION OF FINANCIAL STATEMENTS

3.1.7 Accrual Basis of Accounting


15.1.30
An entity shall prepare its financial statements, except for cash flow information, using the accrual basis
of accounting. (ie. Expenses and revenue are recorded when incurred and earned not when the money is
paid or received)

3.1.8 Materiality and Aggregation


15.1.31
Each material class of similar items shall be presented separately in the financial statements.
Dissimilar items may be aggregated only if there are individually immaterial.
Example: Materiality

If a statement of financial position shows non-current assets of $2 million and inventories of $30,000 an
error of $20,000 in the depreciation calculations might not be regarded as material, whereas an error of
$20,000 in the inventory valuation probably would be. In other words, the total of which erroneous item
forms part must be considered.

If a business has a bank loan of $50,000 and a $55,000 balance on bank deposit account, it might well be
regarded as a material misstatement if these two amounts were displayed in the statement of financial
position as 'cash at bank $5,000'. In other words, incorrect presentation may amount to material
misstatement even if there is no monetary error.

3.2 Components of Financial Statements

Diagram 3.2 Components of Financial Statements

48
ACCA Financial Reporting (FR)
CHAPTER 3: IAS 1 PRESENTATION OF FINANCIAL STATEMENTS

3.3 Identification of Financial Statements

As a result of the above point, it is most important that entities distinguish the Financial Statements
clearly from any other information published with them. This is because all IAS / IFRS apply only to the
Financial Statement (i.e. the main statements and related notes), so readers of the annual report must be
able to differentiate between the parts of the report which are prepared under IFRS, and other parts
which are not.

IAS 1 also requires disclosure of the following information in a prominent position:

(i) Name of the reporting entity (or other means of identification).


(ii) Whether the accounts cover the single entity only or a group of entities.
(iii) The date of the end of the reporting period or the period covered by the Financial Statement
(iv) The presentation currency.
(v) The level of rounding used in presenting amounts in the financial statements.

Judgement must be used to determine the best method of presenting this information. In particular, the
standard suggests that the approach to this will be very different when the Financial Statement are
communicated electronically.

The level of rounding is important, as presenting figures in thousands or millions of units make the figures
more understandable. The level of rounding must be disclosed, however, and it should not obscure
necessary details or make the information less relevant.

49
ACCA Financial Reporting (FR)
CHAPTER 3: IAS 1 PRESENTATION OF FINANCIAL STATEMENTS

3.4 Statement of Financial Position (SOFP)


Learning Outcome (ACCA Study Guide Area D)
D1a : Prepare an entity’s statement of financial position and statement of profit or loss and other
comprehensive income in accordance with the structure and content prescribed within IFRS and
with accounting treatments as identified within syllabus areas A, B and C.
Statement of Financial Position consist of Assets (Non-current and current), equity and Liabilities (Non-
current and current).

Refer table 3.4 for sample of complete statement of Financial Position

XYZ - STATEMENT OF FINANCIAL POSITION AT 31 DECEMBER 20X7 20X6


Assets $'000 $'000
Non-current assets (N1)
Property, plant and equipment xx xx
Investment property xx xx
Other intangible assets xx xx
Investments in subsidiary/ associate xx xx
Financial assets xx xx
Biological assets xx xx
xx xx
Current assets (N1)
Inventories xx xx
Trade and other receivables xx xx
Contract asset xx xx
Assets held for sale xx xx
Cash and cash equivalents xx xx
xx xx
Total assets XXX XXX

Equity and liabilities


Equity
Share capital xx xx
Other component of equity ( i.e Share premium) xx xx
Retained earnings xx xx
Revaluation surplus xx xx
Other reserves xx xx
Total equity xx xx

50
ACCA Financial Reporting (FR)
CHAPTER 3: IAS 1 PRESENTATION OF FINANCIAL STATEMENTS

Non-current liabilities (N2)


% Loan notes xx xx
% Redeemable preference shares xx xx
Deferred tax xx xx
Deferred income - government grants xx xx
Lease liabilities xx xx
Total non-current liabilities xx xx

Current liabilities (N2)


Trade and other payables xx xx
Lease liabilities xx xx
Accrued expenses xx xx
Current tax payable xx xx
Short-term provisions xx xx
Bank overdraft xx xx
Total current liabilities xx xx
Total liabilities xx xx
Total equity and liabilities XXX XXX

N1: Current asset

An asset should be classified as a current asset when it:


• is expected to be realised in, or is held for consumption in, the normal course of the entity’s operating
cycle; or
• is held primarily for trading purposes or for the short-term and expected to be realised within 12
months of the end of reporting period; or
• is cash or a cash equivalent unless restricted from being exchanged or used to settle a liability due after
12 months.

All other assets should be classified as non-current assets.

N2: Current liability

A liability should be classified as a current liability when it:


• is expected to be settled in the normal course of the entity’s operating cycle; or
• is held primarily for the purpose of trading; or
• is due to be settled within 12 months after the end of the reporting period; or when
• the entity does not have unconditional right to defer settlement of the liability for at least 12 months
after the end of reporting period

All other liabilities should be classified as non-current liabilities.

When long-term debt is expected to be refinanced under an existing loan facility, and the entity has the
discretion, the debt is classified as non-current even if due within 12 months.

If a liability has become payable on demand on or before the Statement of Financial Position date (following
breach of covenants) the liability is current even if the lender has agreed, after the reporting period date and
before authorisation of the accounts not to demand payment as a consequence of the breach.

51
ACCA Financial Reporting (FR)
CHAPTER 3: IAS 1 PRESENTATION OF FINANCIAL STATEMENTS

3.4.1 Information which must be presented in the statement of financial position

IAS 1 specifies various items which must be presented in the statement of financial position as a
minimum disclosure.

• Property, plant and equipment


• Investment property
• Intangible assets
• Financial assets
• Investments accounted for using the equity method
• Assets classified as held for sale
• Inventories
• Trade and other receivables
• Cash and cash equivalents
• Trade and other payables
• Provisions
• Financial liabilities
• Current and deferred tax assets and liabilities
• Non-controlling interest
• Issued capital and reserves attributable to owners of the parent

Any other line items, headings or sub-totals should be shown in the statement of financial position when
it is necessary for an understanding of the entity's financial position.

This decision depends on judgements based on the assessment of the following factors:

• Nature and liquidity of assets and their materiality. Thus goodwill and assets arising from
development expenditure will be presented separately, as will monetary/non-monetary assets
and current/non-current assets.
• Function within the entity. Operating and financial assets, inventories, receivables and cash and
cash equivalents are therefore shown separately.
• Amounts, nature and timing of liabilities. Interest-bearing and non-interest-bearing liabilities and
provisions will be shown separately, classified as current or non-current as appropriate.

The standard also requires separate presentation where different measurement bases are used for assets
and liabilities which differ in nature or function. According to IAS 16 Property, Plant and Equipment, for
example, it is permitted to carry certain items of property, plant and equipment at cost or at a revalued
amount. Property, plant and equipment may therefore be split to show classes held at historical cost
separately from those that have been revalued.

52
ACCA Financial Reporting (FR)
CHAPTER 3: IAS 1 PRESENTATION OF FINANCIAL STATEMENTS

3.4.2 Information presented either in the SOFP or in the notes


15.1.32

Certain pieces of information may be presented either in the Statement of Financial Position (SOFP) or in
the notes to the financial statements.

These comprise:

• Further sub-classification of line items from the statement of financial position. Disclosures will
vary from item to item, which will in part depend on the requirements of IFRS. For example,
tangible assets are classified by class of asset (eg land and buildings, plant and equipment) as
required by IAS 16 Property, Plant and Equipment
• Details about each class of share capital
• Details about each reserve within equity

3.5 Statement of Profit or Loss & Other Comprehensive Income (SOPL & OCI)

Learning Outcome (ACCA Study Guide Area D)


D1a : Prepare an entity’s statement of financial position and statement of profit or loss and other
comprehensive income in accordance with the structure and content prescribed within IFRS and
with accounting treatments as identified within syllabus areas A, B and C.
IAS 1 allowed income and expense items to be presented either:

a) in a single SOPL & OCI, or


b) in two statements:
• a separate statement of profit or loss; and
• a second statement beginning with profit or loss and displaying components of other
comprehensive income (statement of comprehensive income)

As a minimum, the SOPLOCI shall include line items that present the following amounts for the period:

a) revenue;
b) finance costs;
c) share of the profit or loss of associates and joint ventures accounted for using the equity method;
d) tax expense;
e) a single amount comprising the total of:
i. the post-tax profit or loss of discontinued operations and
ii. the post-tax gain or loss recognised on the measurement to fair value less costs to sell or
on the disposal of assets or disposal group(s) constituting the discontinued operation;
f) profit or loss;
g) each component of other comprehensive income classified by nature (excluding amounts in (h));
h) share of the other comprehensive income of associates accounted for using the equity method;
and total comprehensive income.

53
ACCA Financial Reporting (FR)
CHAPTER 3: IAS 1 PRESENTATION OF FINANCIAL STATEMENTS

Table 3.5: Classification of expenses by function

XYZ - STATEMENT OF PROFIT OR LOSS AND OTHER COMPREHENSIVE INCOME FOR THE YEAR ENDED
31 DECEMBER 20X7
20X7 20X6
$'000 $'000
Revenue xx xx
Cost of sales (xx) (xx)
Gross profit xx xx
Other income xx xx
Distribution costs (xx) (xx)
Administrative expenses (xx) (xx)
Other expenses (xx) (xx)
Finance costs (xx) (xx)
Profit before tax xx xx
Income tax expense (xx) (xx)
Profit for the year from continuing operations xx xx
Loss for the year from discontinued operations (xx) (xx)
Profit/ (Loss) for the year xx xx

Other comprehensive income:


Gain / (loss) on property revaluation xx xx
Gain / (loss) on valuation of investment at FVTOCI xx (xx)
Deferred tax on revaluation (xx) (xx)
Other comprehensive income for the year, net of tax xx xx
Total comprehensive income for the year xx xx

54
ACCA Financial Reporting (FR)
CHAPTER 3: IAS 1 PRESENTATION OF FINANCIAL STATEMENTS

3.6 Statement of Changes in Equity (SOCE)

Learning Outcome (ACCA Study Guide Area D)


D1b: Prepare and explain the contents and purpose of the statement of changes in equity.

SOCE shows the movement in equity section of the SOFP

Table 3.6: Sample of Statement of charges in Equity (SOCE)

XYZ - STATEMENT OF CHANGES IN EQUITY FOR THE YEAR ENDED 31 DECEMBER 20X7

Share Share Equity FVTOCI Revaluation Retained


capital Premium option reserves surplus earnings Total
$'000 $'000 $'000 $'000 $'000 $'000 $'000
Balance at 1 Jan 20X6 xx xx xx xx xx xx xx
Changes in Accounting
Policies/ - - - - - xx xx
Correction of error
Restated balance xx xx xx xx xx xx xx
Changes in equity for
20X6
Dividends - - - - - (xx) (xx)
Total comprehensive
income for the year - - - xx xx xx xx
Balance at 31 Dec
20X6 xx xx xx xx xx xx xx
Changes in equity for 20X7
Issue of share capital xx xx - - - - xx
Dividends - N1 - - - - - (xx) (xx)
Total comprehensive
income for the year - - - xx xx xx xx
Transfer to retained - - - - (xx) xx -
earnings - N2
- - xx - - - xx
Convertible loan notes
Balance at 31 Dec
20X7 xx xx xx xx xx xx xx

N1: Dividends
- comprises of interim dividend paid & dividend declared before reporting period
- dividend declared after reporting period – only disclosed in the notes (IAS 10)

N2: Transfer to RE

- due to excess depreciation adjustment or realisation of revaluation surplus for non -current
asset disposed off during the year (IAS 16)

55
ACCA Financial Reporting (FR)
CHAPTER 3: IAS 1 PRESENTATION OF FINANCIAL STATEMENTS

3.7 Statement of Cash Flow (SOCF)

Provides users of Financial Statement with a basis to assess the ability of the entity to generate cash
& cash equivalents and the needs of the entity to utilise those cash flows.

IAS 7 sets out the requirement for the presentation and related disclosures.

This topic will be discussed in detail in Chapter 24 (IAS 7 Cash Flow statements)

3.8 Notes to the Financial Statements

56
ACCA Financial Reporting (FR)
CHAPTER 3: IAS 1 PRESENTATION OF FINANCIAL STATEMENTS

3.9 Check Understanding

Question 1

The accountant of Wislon Co has prepared the following list of account balances as at
31 December 20X7.
$’000
50c ordinary shares (fully paid) 450
10% Loan notes (secured) 200
Retained earnings 1.1.X7 242
General reserve 1.1.X7 171
Land and buildings 1.1.X7 (cost) 430
Plant and machinery 1.1.X7 (cost) 830
Accumulated depreciation:
Buildings 1.1.X7 20
Plant and machinery 1.1.X7 222
Inventory 1.1.X7 190
Sales 2,695
Purchases 2,152
Ordinary dividend paid 15
Loan interest paid 10
Wages and salaries 254
Light and heat 31
Sundry expenses 113
Suspense account (credit balance) 135
Trade accounts receivable 179
Trade accounts payable 195
Cash 126

Notes:
(a) Sundry expenses include $9,000 paid in respect of insurance for the year ending 1 September
20X8. Light and heat does not include an invoice of $3,000 for electricity for the three months
ending 2 January 20X8, which was paid in February 20X8. Light and heat also includes $20,000
relating to salesmen’s commission.

(b) The suspense account is in respect of the following items.


$’000
Proceeds from the issue of 100,000 ordinary shares 120
Proceeds from the sale of plant 300
420
Less consideration for the acquisition of Mary & Co 285
135

(c) The property was acquired some years ago. The buildings element of the cost was
estimated at $100,000 and the estimated useful life of the assets was fifty years at the time
of purchase. As at 31 December 20X7 the property is to be revalued at $800,000.

(d) The plant which was sold had cost $350,000 and had a carrying amount of $274,000 as on
1.1.X7. $36,000 depreciation is to be charged on plant and machinery for 20X7.

57
ACCA Financial Reporting (FR)
CHAPTER 3: IAS 1 PRESENTATION OF FINANCIAL STATEMENTS

(e) The management wish to provide for:


(i) Loan interest due
(ii) A transfer to general reserve of $16,000
(iii) Audit fees of $4,000

(g) Inventory as at 31 December 20X7 was valued at $220,000 (cost).

(h) Taxation is to be ignored.

Required
Prepare the financial statements of Wilson Co as at 31 December 20X7 in accordance with IAS1.
You do not need to produce notes to the financial statements.

58
ACCA Financial Reporting (FR)
CHAPTER 3: IAS 1 PRESENTATION OF FINANCIAL STATEMENTS

Question 2

The following trial balance has been extracted from the books of Arran as at 31 March 20X7:

$’000 $’000
Administration expenses 250
Distribution costs 295
Share capital (all ordinary shares of $1 each) 270
Share premium 80
Revaluation reserve 20
Dividend paid 27
Cash at bank and in hand 3
Receivables 233
Interest paid 25
Dividends received 15
Interest received 1
Land and buildings at cost (land 380, buildings 100) 480
Land and buildings: accumulated depreciation at 1/4/X6 30
Plant and machinery at cost 400
Plant and machinery: accumulated depreciation at 1/4/X6 170
Retained earnings (at 1/4/X6) 235
Purchases 1260
Sales 2165
Inventory at 1/4/X6 140
Trade payables 27
Bank loan 100
3113 3113

Notes :

(1) Inventory at 31 March 20X7 was valued at a cost of $95,000. Included in this balance were
goods that had cost $15,000. These goods had become damaged during the year and it is considered
that following remedial work the goods could be sold for $5,000.

(2) Depreciation for the year to 31 March 20X7 is to be charged against cost of sales as follows:

Buildings 5% on cost (straight line)


Plant and machinery 30% on carrying value (CV) (reducing balance)
(3) Income tax of $165,000 is to be provided for the year to 31 March 20X7.

(4) Land is to be revalued upwards by $100,000.

Required:

Prepare the financial statements for the year ended 31 March 20X7. You do not need to produce
notes to the statements.

59
ACCA Financial Reporting (FR)
CHAPTER 3: IAS 1 PRESENTATION OF FINANCIAL STATEMENTS

Question 3

The trial balance of Penguin, a company as at 31 December 2010 was as follows:

Dr Cr
$ $
Sales and purchases 20,000 50,000
Opening Inventory 8,000
Distribution costs 10,400
Administration expenses 15,550
Receivables and payables 10,000 20,000
Cash at bank 8,100
Ordinary shares 50c 8,000
10% irredeemable preference shares $1 9,000
10% loan notes 8,000
Non-current assets at net book value 35,000
Share premium 3,000
Retained earnings at 1 January 2010 3,000
Loan note Interest paid 400
Preference dividend paid 450
Interim ordinary dividend paid 1,600
Income tax 500
Suspense 8,000
109,500 109,500

The following is to be taken into account:

Prepare the following statements for the year ended 31 December 2010:

(1) A building whose net book value is currently $5,000 is to be revalued to $11,000.

(2) A final ordinary dividend of 10c per share is to be proposed.

(3) The balance on the income tax account represents an overprovision of tax for the previous year.
Income tax for the current year is estimated at $3,000.

(4) Closing inventory is $12,000.

(5) The balance on the suspense account represents the proceeds from the issue of 4,000 ordinary
shares.

Prepare the following statements for the year ended 31 December 2010:

(1) statement of profit or loss and other comprehensive income

(2) statement of financial position

(3) Statement of changes in equity

60
ACCA Financial Reporting (FR)
CHAPTER 3: IAS 1 PRESENTATION OF FINANCIAL STATEMENTS

3.10 Chapter 3 Summary

Diagram 3.10.: Summary of IAS 1 Presentation of Financial Statements

61
ACCA Financial Reporting (FR)
CHAPTER 4: IAS 16 PROPERTY, PLANT AND EQUIPMENT

CHAPTER 4: IAS 16 PROPERTY, PLANT


AND EQUIPMENT
LEARNING OUTCOME
At the end of the chapter, you should be able to:

TLO B1a. Define and compute the initial measurement of Non-current asset/Property, Plant &
Equipment (PPE) including borrowing costs and an asset that has been self-constructed.

TLO B1b. Identify subsequent expenditure that may be capitalised, distinguishing between capital
and revenue expenditure.

TLO B1c. Discuss the requirements of relevant accounting standards in relation to the revaluation
of Non-Current Asset/Property, Plant and Equipment (PPE).

TLO B1d. Account for revaluation and disposal gains and losses for Non-current assets/Property,
Plant and Equipment (PPE).

TLO B1e. Compute depreciation based on the cost and revaluation models and on assets that have
two or more significant parts (complex assets).

TLO B1f. Discuss why the treatment of investment properties should be differ from other
properties.

TLO B1g. Apply the requirements of relevant accounting standards to an investment property.

62
ACCA Financial Reporting (FR)
CHAPTER 4: IAS 16 PROPERTY, PLANT AND EQUIPMENT

Overview

IAS 16 in relation to other standards in context of Non-Current Asset

Diagram 4.1 Relational Mapping

4.1 Introduction

4.1.1 Objective of IAS 16

Learning Outcome (ACCA Study Guide Area B)


B1c : Discuss the requirements of relevant accounting standards in relation to the revaluation of
Non-Current Asset/Property, Plant and Equipment (PPE).

The International Accounting Standards Board (IASB) developed this revised IAS 16 as part of its
project on improvements to international accounting standards.

IAS 16 PPE provides guidance on the accounting treatment for tangible non-current assets for:
• The recognition of assets
• The determination of their carrying amounts
• The depreciation charges & impairment losses relating to them.

This provides the users of financial statements with information about an entity’s investment in its
PPE.

63
ACCA Financial Reporting (FR)
CHAPTER 4: IAS 16 PROPERTY, PLANT AND EQUIPMENT

4.1.2 Scope of IAS 16

Property plant and equipment (PPE) is defined as tangible assets that an entity holds for its own use
in the production or supply of goods or services, or for rental to others, or for administrative purposes
and that the entity expects to use during more than one period.

Diagram 4.1.2 Scope of IAS 16

IAS 16 should be followed when


accounting for Property, Plant
and Equipment (PPE) unless
another IAS or IFRS requires a
different treatment such as IAS
40 Investment Properties or
IFRS 5 Non-current assets held
for sale and discontinued
operations.

One should determine whether the item is an asset of the reporting entity (Conceptual
Framework) and then determine whether that asset is an item of PPE (IAS 16).

Identifying PPE (IAS 16)

Property, plant and equipment are tangible items that are held for use for the following purposes
and expected to be used during more than one period:

(a) the production


(for example, machinery used in a production line to manufacture cars) or supply of goods
(for example, a retailer’s point-of-sale equipment) or services (an architect’s tools),
(b) for rental to others (for example, a car hire’s rental fleet);
(c) for administrative purposes (for example, computer equipment used by an entity’s
administration staff)

Example 1:

Cars held for resale by a motor dealer are inventories (a current asset) whereas cars
held for use by employees on company business are PPE.

Other Examples:

PPE includes freehold and leasehold land and buildings (IFRS 16) and plant and
machinery, and forms the major part of assets of certain types of business, such as
manufacturing and transport businesses.

64
ACCA Financial Reporting (FR)
CHAPTER 4: IAS 16 PROPERTY, PLANT AND EQUIPMENT

This Standard does not apply to:

(a) property, plant and equipment classified as held for sale in accordance with IFRS 5 Non-current
Assets Held for Sale and Discontinued Operations;
(b) biological assets related to agricultural activity (discussed in Chapter 27 IAS 41 Agriculture);
(c) the recognition and measurement of exploration and evaluation assets (IFRS 6 Exploration for
and Evaluation of Mineral Resources);
(d) mineral rights and mineral reserves such as oil, natural gas and similar non-regenerative
resources’.

Note:

Productive assets held by entities in the extractive industries are subject to the same recognition

4.2 Recognition Criteria

4.2.1 Definition and Recognition Criteria

Learning Outcome (ACCA Study Guide Area B)


A2b: Apply the recognition criteria to:
i) assets and liabilities.

The key elements in financial statements, identified in the IASB Conceptual Framework, which are
relevant to PPE are:

1. Definition and Recognition of Asset:

Definition of Asset

IASB Conceptual Framework 2010 New Conceptual Framework 2018

Resources controlled by the entity as a result of A present economic resource controlled by the
past events and from which future economic entity as a result of past events.
benefits are expected to flow into the entity An economic resource is a right that has the
potential to produce economic benefits

It is important to note that the definition of asset in IAS 16 was NOT revised following the revision
of the definition of an asset in the Conceptual Framework for Financial Reporting issued in 2018,
hence the definition and recognition criteria for PPE under IAS 16 shall prevail.

65
ACCA Financial Reporting (FR)
CHAPTER 4: IAS 16 PROPERTY, PLANT AND EQUIPMENT

2. Recognition criteria set out in IAS 16:

The cost of an item of PPE shall be recognised as an asset if, and only if:
a) Probable inflow of future economic benefits associated with the item will flow to the
entity; AND
b) The cost of the item can be measured reliably

The recognition criteria must be applied to cost incurred initially to acquire or construct a PPE &
cost incurred subsequently to add to, replace part of or service the PPE.

This means the definition and recognition criteria must be applied at any time over the life of
the item of PPE when expenditure on it is incurred; it is not only applied on the initial acquisition
or construction of the item.

4.2.2 Underlying Principles of Recognition Criteria

The definition and recognition criteria on income and expenses set out in IAS 16 is consistent with
the Existing Conceptual Framework including:
(i) Gains, which are a part of income
Gains increases in economic benefits through increases in assets, or decreases in liabilities,
that result in increases in equity, other than those relating to contributions from holders of
equity claims.
(ii) Losses, which are included in expenses
Losses decreases in economic benefits through decreases in assets, or increases in liabilities,
that result in decreases in equity, other than those relating to distributions to holders of equity
claims.

Gains and losses relate to the subsequent depreciation, revaluation, impairment and disposal of
PPE. The depreciation charge of PPE reflecting the consumption pattern of PPE is consistent with the
definition of expenses in the Conceptual Framework.

(iii) Presentation

However, the Existing Conceptual Framework does not include any principle to determine which
items of income or expense should be presented in SOPL and which in OCI. The proposals in the ED
in CF includes new principles for splitting information about financial performance into SOPL and
other comprehensive income (OCI): the rebuttable presumption is that all items of income and
expenses are reported in SOPL. Income and expenses are included in OCI only if that enhances the
relevance of profit or loss and if they relate to assets or liabilities re-measured to current values.

66
ACCA Financial Reporting (FR)
CHAPTER 4: IAS 16 PROPERTY, PLANT AND EQUIPMENT

4.2.3 Cessation of Recognition


Recognition of costs in the carrying amount of an item of property, plant and equipment ceases
when the item is in the location and condition necessary for it to be capable of operating in the
manner intended by management.

The following costs are not included in the carrying amount of an item of PPE:

(a) Cost incurred while an item capable of operating in the manner intended by management has yet
to be brought into use or is operated at less than full capacity;
(b) Initial operating losses, such as those incurred while demand for the item’s output builds up; and
(c) Cost of relocating or reorganizing part or all of an entity’s operations.

4.3 Measurement Criteria

4.3.1 Initial Measurement

Learning Outcome (ACCA Study Guide Area B)


B1a : Define and compute the initial measurement of Non-current asset/Property, Plant &
Equipment (PPE) including borrowing costs and an asset that has been self-constructed.
B1b : Identify subsequent expenditure that may be capitalised, distinguishing between capital and
revenue expenditure.
An item of property, plant and equipment that qualifies for recognition as an asset shall be measured
initially (measurement at the date of purchase) at its cost.
Cost is the amount of cash or cash equivalents paid or the fair value of the other consideration given
to acquire an asset at the time of its acquisition or construction.

Diagram 4.3.1 Initial Measurement

67
ACCA Financial Reporting (FR)
CHAPTER 4: IAS 16 PROPERTY, PLANT AND EQUIPMENT

Initial Expenditure

Initial Expenditure involves:

1) Purchase price that includes import duties and non-refundable purchase taxes, after deducting
trade discounts and rebates

2) Any cost directly attributable to bringing the asset to the location and condition necessary for it
to be capable of operating in the manner intended by management

3) Dismantling costs and Restoration of site costs

(i) Directly attributable costs given in IAS 16 (to be capitalised)

▪ Costs of employee benefits (ie. Salaries & IAS 16 allows capitalization of costs to take place
wages) that arise directly from the only in respect of the period in which the activities
construction or acquisition of PPE necessary to bring the asset to the location and
▪ Cost of site preparation condition necessary for it to be capable of
▪ Initial delivery & handling cost operating in the manner intended by management
▪ Installation and assembly cost are being undertaken.
▪ Professional fees (ie. Consultancy fees, legal
fees, stamp duty, architect fees) Thus, capitalization should cease when
▪ Testing costs (commissioning costs) substantially all the activities necessary to get the
▪ Borrowing costs (IAS 23) Example BC directly asset ready for use are completed, even if the
attributable to construction of PPE assets has not yet been brought into use.

68
ACCA Financial Reporting (FR)
CHAPTER 4: IAS 16 PROPERTY, PLANT AND EQUIPMENT

(ii) NOT Directly attributable costs given in IAS 16 (costs NOT to be capitalised)

▪ Costs of opening new facility


▪ Costs of introducing new product or service (i.e. advertising and promotional activities)
▪ Costs of conduction biz in a new location or with new customers (including staff training)
▪ Administration and other general overhead cost
▪ Initial operating losses
▪ Costs of relocating or reorganizing part or all of entity’s operations
▪ Abnormal costs (ie. Errors, idle time, wastage during plant construction)
▪ Start up and pre-production costs
▪ Costs incurred after asset is physically ready for use
▪ Incidental Operations
- May occur before or during the construction or development activities.
- Not necessary to bring an item to the location and condition necessary for it to be
capable of operating in the manner intended by management
- The inc and related exp recog in P&L - included in their respective classifications of
inc & exp

Exercise 4.3.1(a)
The components of the cost of a major item of equipment are given below:
$
Purchase price 780,000
Import duties 117,000
VAT (refundable) 78,000
Site preparation 30,000
Installation 28,000
Testing 10,000
Initial losses before asset reaches planned performance 50,000
Discounted cost of dismantling and removal at end of useful life 40,000
1,133,000

What amount should be recognised as the cost of the asset in accordance to IAS 16 PPE?
A. $896,000
B. $1,045,000
C. $1,005,000
D. $1,133,000

69
ACCA Financial Reporting (FR)
CHAPTER 4: IAS 16 PROPERTY, PLANT AND EQUIPMENT

Exercise 4.3.2

Broadoak has recently purchased an item of plant from PlantCo, the details of this are:

$ $
Basic list price of plant 240000
Trade discount applicable to Broadoak 12.5% on list price

Ancillary costs:
Shipping and handling costs 2750
Estimated pre-production testing 12500
Maintenance contract for three years 24000

Site preparation costs:


Electrical cable installation 14000
Concrete reinforcement 4500
Own labour costs 7500
26000

Broadoak paid for the plant (excluding the ancillary costs) within four weeks of order, thereby
obtaining an early settlement discount of 3%.

Broadoak had incorrectly specified the power loading of the original electrical cable to be
installed by the contractor. The cost of correcting this error of $6,000 is included in the above
figure of $14,000.

The plant is expected to last for 10 years. At the end of this period there will be compulsory costs
of $15,000 to dismantle the plant and $3,000 to restore the site to its original use condition.

Calculate the amount at which the plant will be measured at recognition. (Ignore discounting to
present value)

70
ACCA Financial Reporting (FR)
CHAPTER 4: IAS 16 PROPERTY, PLANT AND EQUIPMENT

(iii) Other initial expenditure

Definition: Items acquired for safety or environmental reasons

The acquisition of such PPE although not directly increasing the for economic benefits of any
particular existing item of PPE, may be necessary for an entity to obtain the for economic benefits
from its other assets.

Such items of PPE qualify for recognition as assets because they enable an entity to derive for
economic benefits from related assets in excess of what could be derived had those items not been
acquired.

Example 4.3.1 (a) Other initial expenditure in compliance with environmental requirements:

A chemical manufacturer may install new chemical handling processes to comply with
environmental requirements for the production and storage of dangerous chemicals; related
plant enhancements are recognised as an asset because without them the entity is unable to
manufacture and sell chemicals.

The resulting carrying amount of such an asset and related assets is reviewed for impairment in
accordance with IAS 36 Impairment of Assets.

(iv) Incidental Operations

Incidental operations may occur before or during the construction or development activities.

Example 4.3.1 (b) Incidental Operations:


Income may be earned through using a building site as a car park until construction starts.

Incidental operations are not necessary to bring an item to the location and condition necessary
for it to be capable of operating in the manner intended by management, the income and related
expenses of incidental operations are recognised in profit or loss and included in their respective
classifications of income and expenses.

(v) Self-Constructed Asset

The cost of a self-constructed asset is determined using the same principles as for an acquired
asset including the following criteria:

(a) If an entity makes similar assets for sale in the normal course of business, the cost is usually
the same as the cost of constructing an asset for sale (IAS 2).
(b) Any internal profits are eliminated in arriving at such costs.
(c) The cost of abnormal amounts of wasted material, labour, or other resources incurred in self-
construction an asset is not included in the cost of the asset.
(d) Borrowing cost on self-constructed asset if the asset satisfied the criteria as a qualifying
asset.

71
ACCA Financial Reporting (FR)
CHAPTER 4: IAS 16 PROPERTY, PLANT AND EQUIPMENT

Subsequent Expenditure (after date of purchase)

Definition: Subsequent costs incurred are recognized as:


(a) Capital expenditure (included as part of the cost of PPE) if:

Probable future economic benefit in excess of the amount originally assessed

▪ modifications that enhances economic benefits of an asset


▪ Increases its life/production capacity
▪ Upgrades an asset with the effect of improving its quality

(b) Revenue expenditure (expenses) if:

Cost of day to day servicing costs/repairs, maintenance, routine type of expenditure

▪ maintenance
▪ repair, normal service
▪ overheads to be expensed in SOPL

Example 4.3.1(c)

A furnace may require relining after a specified number of hours of use, or aircrafts interiors
such as seats and galleys may require replacement several times during the life of the
airframe. Items of property, plant and equipment may also be acquired to make a less
frequently recurring replacement, such as replacing the interior walls of a building, or to make
a non-recurring replacement.

An entity recognizes in the carrying amount of an item of property, plant and equipment the
cost of replacing part of such an item when the cost is incurred if the recognition criteria are
met. The carrying amount of those parts that are replaced is derecognised.

Example 4.3.1(d) – Subsequent expenditure

A piece of machinery has an annual (recurring type) service costing $10,000 (rev exp P/L).
During the most recent service, it was decided to replace (replacement restores its previous
life) part of the engineering meaning that it will work faster and produce more units of product
per hour. The cost of replacement part is $20,000. (Enhances future economic benefit: Capita l
expenditure + Non Carrying Amount (NCA).

72
ACCA Financial Reporting (FR)
CHAPTER 4: IAS 16 PROPERTY, PLANT AND EQUIPMENT

(i) Replacing parts:

Example 4.3.1 (e)

The head office (H/O) building acquired on 1 January 20x1 at a cost of $1 million which
included the cost of interior design of the wall amounting to $0.1 million. The useful economic
life of the building is 20 years. The interior design of the wall needs to be replaced every 5
years. On 1 January 20x5 the interior walls design is replaced at a cost of $0.2 million

Required:
Determine the carrying amount of the head office building at 1 January 20x5 after the
replacement of the interior wall and the depreciation charge for the year ended 31 Dec
20x5.

Solution to cost recognition:


At 1 Jan 20x5 CA of H/O building before replacement
900,000 – (900,000/20 x 4 years) + 100,000 – (100,000/5 x 4 years) = 740,000
New CA of replaced cost 200,000
Old CA being replaced 100,000 – 100,000/5 x 4 years (20,000)
New CA of H/O building after replacement 920,000

Depreciation charge for the year ended 31 Dec 20x5


720,000/16 + 200,000/5 = 85,000

(ii) Major inspections:


A condition of continuing to operate an item of property, plant and equipment (for example, an
aircraft) may be performing regular major inspections for faults regardless of whether parts of the
item are replaced. When each major inspection is performed, its cost is recognised in the carrying
amount of the item of property, plant and equipment as a replacement if the recognition criteria are
satisfied. Any remaining carrying amount of the previous inspects (as distinct from physical parts) is
derecognised.

Example 4.3.1(f)

An aircraft has a carrying amount of $10 million. The latest inspection currently costs $0.2 million.

The previous inspection has a carrying amount of $0.1 million included in the carrying amount
of $10 million.

Solution to cost recognitions:


Add the cost of the new inspection, and remove the carrying amount of the old inspection

73
ACCA Financial Reporting (FR)
CHAPTER 4: IAS 16 PROPERTY, PLANT AND EQUIPMENT

4.4 Depreciation

4.4.1 Depreciation

Learning Outcome (ACCA Study Guide Area B)


B1e : Compute depreciation based on the cost and revaluation models and on assets that have two
or more significant parts (complex assets).

Depreciation is defined as the systematic allocation of the depreciable amount of an asset over its
useful life.

Each part of an item of PPE with a cost that is significant in relation to the total cost of the item shall
be depreciated separately (depreciation by component). This is because different parts will have
different useful lives & therefore different depreciation rates. For example, it may be appropriate to
depreciate separately the airframe and engines of an aircraft.

I: Replaced every 5 years $200

II: Ship; HC $500, EUL 50 years

III: Replaced every 10 years, HC $300


Past practice (WRONG!)
1. Depreciation $1000 ÷ 50 years = $20 p.a.
2. Create a provision for replacement parts
Provision 1: $200 set up over 5 years
Provision 2: $300 set up over 10 years
Criticism
1. Double charge
2. Provision may never be used (ship could be sold in the meantime)

Currently: Correct!
NCA must be split into smaller assets and depreciate over its respective life

IAS 37
IAS 16 No provision must be created for
Depn p.a. replacement parts
Ship: Cost ÷ EUL = 40
Asset I 200 5 future events
10 no present obligations
Asset II 500 50 30
Asset III 300 10
1000 80 SOPL

74
ACCA Financial Reporting (FR)
CHAPTER 4: IAS 16 PROPERTY, PLANT AND EQUIPMENT

4.4.2 Dealing with depreciation

When dealing with depreciation, 3 basic things must be noted:

(i) Depreciable amount


Depreciable amount is simply HOW MUCH you are going to depreciate. It is the cost of an asset,
or other amount substituted for cost, less its residual value

(ii) Depreciation period


Depreciation period is simply HOW LONG you are going to depreciate and it is basically asset’s
useful life.

(iii) Depreciation method (discussed in Notes 4.5)

4.4.3 Depreciation Period

Depreciation of an asset commence when it is available for use, irrespective of whether it is actually
used (i.e when it is in the location and condition necessary for it to be capable of operating in the
manner intended by management).

(i) Cessation of depreciation


Depreciation of an asset ceases at the earlier of the date that the asset is classified as held for sale
(or included in a disposal group that is classified as held for sale) in accordance with IFRS 5 and
the date that the asset is derecognized. Therefore, depreciation does not cease when the asset
becomes idle or is retired from active use unless the asset is fully depreciated.

(ii) Factors determining the useful life

• Expected usage of the asset. Usage is assessed by reference to the asset’s expected capacity or
physical output.
• Expected physical wear and tear, which depends on operational factors such as the number of
shifts for which the asset is to be used and the repair and maintenance programme, and the
care and maintenance of the asset while idle.
• Technical or commercial obsolescence arising from changes or improvements in production, or
from a change in the market demand for the product or service output of the asset.
• Legal or similar limits on the use of the asset, such as the expiry dates of related leases

75
ACCA Financial Reporting (FR)
CHAPTER 4: IAS 16 PROPERTY, PLANT AND EQUIPMENT

(iii) Land and Buildings

Land and buildings are separable assets are accounted for separately, even when they are
acquired together.

• Land has an unlimited useful life and therefore is not depreciated


• Buildings have a limited useful life and therefore are depreciable assets

4.5 Depreciation Methods

Depreciation method is simply HOW; IN WHAT MANNER you are going to depreciate.

The depreciation method used shall reflect the pattern in which the asset’s future economic benefits
are expected to be consumed by the entity.

An entity may select from variety of depreciation methods, such as straight-line method, diminishing
balance method and the units of production methods.

Selected method shall be reviewed at least at the end of each financial year. If there is a change in
the expected pattern of asset’s usage, then the depreciation method shall be changed and be
accounted for as a change in an accounting estimate in line with IAS8 (no restatement of
previous periods).

Depreciation shall be recognized in profit or loss.

Formula
𝐻𝐶−𝐸𝑅𝑉
• SLM = 𝐸𝑈𝐿
• RBM = NBV x Depreciation %
Output p.a.
• Units of production method = Total output
x [HC − ERV]

76
ACCA Financial Reporting (FR)
CHAPTER 4: IAS 16 PROPERTY, PLANT AND EQUIPMENT

Depreciable amount and depreciation period Depreciation method

• Allocation: The depreciable amount • Selection: method that reflects the expected
allocated on a systematic basis over its pattern of consumption of the future
useful life. economic benefits embodied in the asset.

• Review of RV: at least at each financial year- • Application: applied consistently


end, if change: change in an accounting
estimate IAS 8.
• Review: reviewed at least at each financial
• Selection of UL: asset’s expected utility to year-end
the entity.
• Change: change in accounting estimate in
• The useful life of an asset may be shorter accordance with IAS 8.
than its economic life.
• Types: Straight-line depreciation results in a
A matter of judgement based on the experience constant charge.
of the entity with similar assets.
• Factors affecting UL: all the following • The reducing balance method results in a
factors are considered in determining the decreasing charge over the useful life.
useful life of an asset:
(a) expected usage of the asset, physical
wear and tear
(b) technical or commercial obsolescence
(c) legal limits such as the expiry dates of
related leases.

77
ACCA Financial Reporting (FR)
CHAPTER 4: IAS 16 PROPERTY, PLANT AND EQUIPMENT

4.6 Subsequent measurement (at year end)

IAS 16 permits

Cost Model Revaluation Model


SOFP = HC – AD – acc imp loss Revl amount – subsq depn – subs imp loss
(NBV)

I/S Depreciation p.a.  How to determine the revaluation amount?


Either: ⚫ SLM depreciation p.a. based on revl amt ÷ RUL
⚫ RBM

 Procedure for revaluation

**Note: All PPE with a finite UL must be depreciated with the exception of a freehold land.

Diagram 4.6 Subsequent Measurement

78
ACCA Financial Reporting (FR)
CHAPTER 4: IAS 16 PROPERTY, PLANT AND EQUIPMENT

4.6.1 Cost Model

Learning Outcome (ACCA Study Guide Area B)


B1e : Compute depreciation based on the cost and revaluation models and on assets that have two
or more significant parts (complex assets).

After recognition as an asset, an item of property, plant and equipment shall be carried at its cost less
accumulated depreciation and any accumulated impairment losses.

4.6.2 Revaluation Model

Learning Outcome (ACCA Study Guide Area B)


B1d : Account for revaluation and disposal gains and losses for Non-current assets/Property, Plant
and Equipment (PPE).
B1e : Compute depreciation based on the cost and revaluation models and on assets that have
two or more significant parts (complex assets).

Rules:

1. Only permitted if fair value (FV) can be measured reliably


Price that would be received to sell an asset or paid to transfer a liability in an orderly transaction
between market participants at the measurement date [exit price] [IFRS 13 Fair Value
measurement]

2. Same class of assets should be revalued at the same time [i.e. L&B, MV, P&M]
To prevent selective revaluation of certain assets and to avoid disclosing a mixture of costs and
Fair Value from different dates in the FS

3. Regular revaluation to be carried out to ensure that the assets’ carrying value is not materially
different from its market value

Fair value is the price that would be received to sell an asset or paid to transfer a liability in an
orderly transaction between market participants at the measurement date.

79
ACCA Financial Reporting (FR)
CHAPTER 4: IAS 16 PROPERTY, PLANT AND EQUIPMENT

(i) Class of Property, Plant and Equipment


o Land
o Land and buildings;
o Machinery;
o Ships;
o Aircrafts;
o Motor vehicles;
o Furniture and fixtures; and
o Office equipment.

(ii) Accounting Treatment for Revaluations

For a surplus [gain on revaluation], the increase is to be recognized in OCI and accumulated
in equity [revl surplus]. The exception is when the increase is reversing a previous decrease
which was recognized as an expense. Then it is treated as income.

For a decrease in value in revaluation, any decrease should be recognized as an expense,


except when it offsets a previous increase taken as a revaluation surplus. In this case, the
decrease will be recognized in OCI, reducing the balance on the revaluation surplus. Any
decrease greater than the previous upwards increase in value must be taken as an expense in
SOPL.

Example 4.6.2 (a) [Revaluation surplus]


Snoopy Co has a land with a carrying value of $15,000 but two years later, a slump in land values
led the co to reduce the carrying value to $13,000. This was taken as an expense in SOPL. Currently,
there has been a surge in land prices and the land is now worth $20,000.

The double entry is as follows:


Dr Land [SOFP]
Cr SOPL
Cr Revl Surplus [OCI]

Example 4.6.2(b) [Revaluation deficit]


Let’s swap the example above. The original cost was $15,000, revalued upwards to $20,000 two
years ago and now decreased to $13,000.

The double entry is as follows:


Dr Revaluation surplus [OCI]
Dr SOPL
Cr Land [SOFP]

80
ACCA Financial Reporting (FR)
CHAPTER 4: IAS 16 PROPERTY, PLANT AND EQUIPMENT

(iii) Usage of PPE after Revaluation Gain

Some of the revaluation surplus may be transferred to retained earnings as the asset is used
by an entity. In such case, the amount of the surplus transferred would be:

EDA = (Depreciation on revalued amount – Depreciation before revaluation)

Or

EDA = Surplus/Remaining useful life

Transfers from revaluation surplus to retained earnings are NOT made through profit or loss,
instead it is through SOCE.

81
ACCA Financial Reporting (FR)
CHAPTER 4: IAS 16 PROPERTY, PLANT AND EQUIPMENT

4.6.3 Snapshot from Revaluation Model

82
ACCA Financial Reporting (FR)
CHAPTER 4: IAS 16 PROPERTY, PLANT AND EQUIPMENT

Exercise 4.6.3 (a) [Gain/ Surplus on Revaluation]


A company bought a NCA costing $100 on 01.01.X1, which has an Estimated Useful Life (EUL) of 10
years. The asset is to be depreciated on Straight Line Method (SLM) over its EUL. At the beginning of
20X2, the company decided to revalue the asset to $120 before it was disposed-off for $110 at the
beginning of 20X3.

Required: Show the extract of SOFP and SOPL & OCI for the relevant years.

Workings: Revaluation at beginning of the year

Year 1 HC model
01.01.20X1 Historical Cost (HC) $100
EUL 10 years
HC depreciation per annum $10 [SOPL]
31.12.20X1 Net Book Value (NBV) $90 [SOFP]

Year 2 Revaluation model


The company decided to revalue NCA at beginning of the year, revalued amount at $120.
S1: Cal Gain/ [Loss] on Revl = Revl amt – NBV at revl date = $30 (Gain)

S2: Charge depn by year end. Cal Revl depn = Revl amt/ RUL = $13

S3: Cal excess depn = RS/ RUL = $3

[transfer from RS (-) to RE (+) so it’s available for distribution]


NBV of NCA in SOFP = Bal in RS =

Year 3 Year of disposal.


S4: Cal Gain/ [Loss] on disposal = SP – NBV at disposal date [must de-recognition in SOFP]

Bal in RS pertaining to NCA disposed-off – transfer RS to RE =

Extract of SOPL & OCI fye 31.12 X1 X2 X3


Other income – Gain on disposal
Other expenses – Depreciation p.a.

OCI: Gain on Revaluation

Extract of SOFP as at 31.12 X1 X2 X3


NCA: HC / Revaluation Amount
AD/ Subsequent AD
NBV

Equity : RS

83
ACCA Financial Reporting (FR)
CHAPTER 4: IAS 16 PROPERTY, PLANT AND EQUIPMENT

Exercise 4.6.3 (b) [Loss/ Deficit on Revaluation]

A company bought a NCA costing $10,000 on 01.01.X1, which has an EUL of 5 yrs. At the beginning of
X3, the company decided to revalue the asset to $5,000.

Required: Show the extract of SOFP and SOPL & OCI for the relevant years.

Workings:

Year 1 HC Year 2 HC model


model
01.01 HC $10,000 01.01 NBV
EUL 5 years Depn pa
HC depreciation per $2,000 [SOPL] 31.12 NBV
annum
31.12 NBV $8,000 [SOFP]

Year 3 Revaluation model [Revaluation at beginning of the year]

S1: Cal Gain/ [Loss] on Revaluation = Revaluation amount – NBV at revaluation date
=

S2: Charge depreciation by year end. Cal Revl depn = Revl amt/ RUL =

S3: No EDA!

NBV of NCA in SOFP = Bal in RS =

Extract of SOPL & OCI fye 31.12 X3


Other expenses – Depn p.a.
Other expenses – Loss on Revl

Extract of SOFP as at 31.12 X3


NCA: HC / Revl Amount
AD/ Subsq AD
NBV

Equity : RS

84
ACCA Financial Reporting (FR)
CHAPTER 4: IAS 16 PROPERTY, PLANT AND EQUIPMENT

Exercise 4.6.3 (c) [Gain/ Surplus on Revaluation]

A company bought a NCA costing $10,000 on 01.01.X1, which has an EUL of 5 yrs. At the beginning of
X3, the company decided to revalue the asset to $9,000, and at the beginning of X4 to $5,500

Required: Show the extract of SOFP and SOPL & OCI for the relevant years.

Workings:
Year 1 HC model Year 2 HC model
01.01 HC $10,000 01.01 NBV
EUL 5 years Depn pa
HC depn pa $2,000 31.12 NBV
[SOPL]
31.12 NBV $8,000
[SOFP]
Year 3 Revaluation model [Revaluation at beginning of the year]
S1: Cal Gain/ [Loss] on Revl = Revl amt – NBV at revl date

S2: Charge depn by year end. Cal Revl depn = Revl amt/ RUL

S3: Cal excess depn = RS / RUL

NBV of NCA in SOFP = Bal in RS =

Year 4 Revaluation to $5,500 at the beginning of X4


S1: Cal Gain/ [Loss] on Revl = Revl amt – NBV at revl date

S2: Charge depn by year end. Cal Revl depn = Revl amt/ RUL

S3: Cal excess depn = RS / RUL

NBV of NCA in SOFP = Bal in RS =

Extract of SOPL & OCI fye 31.12 X1 X2 X3 X4


Other income – Gain on disposal
Other expenses – Depn p.a.

OCI – Gain/ (Loss) on Revl

Extract of SOFP as at 31.12 X1 X2 X3 X4


NCA: HC / Revl Amount
AD/ Subsq AD
NBV

Equity : RS

85
ACCA Financial Reporting (FR)
CHAPTER 4: IAS 16 PROPERTY, PLANT AND EQUIPMENT

Exercise 4.6.3 (d) [Revaluation at the END of the year]

A company bought a NCA costing $8,000 on 01.01.X1, along with direct acquisition costs of $2,000
which has an EUL of 5 years, with nil scrap value. On 31.12.X2, the company decided to revalue the
asset to $9,000, and on 01.01.X5, the company sold the NCA for $5,000.

Required: Show the extract of SOFP and SOPL & OCI for the relevant years.
Workings:
Year 1 HC model
01.01 HC
EUL 5 years
HC depn pa
31.12 NBV

Year 2 Revaluation model [Revaluation at END of the year]


S1: Charge depn from 01.01.X2 to 31.12.X2 based on HC

NBV at 31.12 =

S2: Cal Gain/ [Loss] on Revl = Revl amt – NBV at revl date

[No revl depn & EDA]


NBV of NCA in SOFP = Bal in RS =

Year 3 Revaluation model


S1: Cal Revl depn = Revl amt/ RUL

S2: Cal excess depn = RS / RUL

NBV of NCA in SOFP = Bal in RS =

Year 4 Revaluation model


S1: Charge depn by year end

S2: Cal excess depn = RS / RUL

NBV of NCA in SOFP = Bal in RS =

Year 5 Year of disposal


Cal Gain/ [Loss] on disposal = SP – NBV at disposal date

Bal in RS pertaining to NCA disposed-off – transfer RS to RE =

86
ACCA Financial Reporting (FR)
CHAPTER 4: IAS 16 PROPERTY, PLANT AND EQUIPMENT

Extract of SOPL & OCI fye 31.12 X1 X2 X3 X4 X5


Other income – Gain on disposal
Other expenses – Depn p.a.

OCI - Gain / (Loss) on Revl

Extract of SOFP as at 31.12 X1 X2 X3 X4 X5


NCA: HC / Revl Amount
AD/ Subsq AD
NBV

Equity : RS

87
ACCA Financial Reporting (FR)
CHAPTER 4: IAS 16 PROPERTY, PLANT AND EQUIPMENT

Exercise 4.6.3 (e) (Revaluation DURING the year)

Co M bought NCA at $10,000 in 01.01.X1 and decided to revalue it on 01.07.X2 to $9,000, before
selling it for $5,000 at the beginning of X4.

Required: Show the extracts of SOFP, SOPL & OCI and SOCIE for the relevant years.

Workings:
Year 1 HC model
HC at 01.01
EUL 5 years
HC depn pa
NBV 31.12

Year 2 Revaluation at 01.07.x2


Charge depn for 6 months up to the date of revl =

NBV at date of revl =

Cal Gain/(Loss) on Revl = Revl amt - NBV at the date of revl

Charge depn for remaining 6 months based on revl amt =

Cal Excess depn = RS / RUL

NBV of NCA in SOFP = Bal in RS =

Year 3 Revl model


By the year end - charge depn p.a. =

Cal Excess depn = RS / RUL

NBV of NCA in SOFP = Bal in RS =

88
ACCA Financial Reporting (FR)
CHAPTER 4: IAS 16 PROPERTY, PLANT AND EQUIPMENT

Year 4 Year of disposal


Cal G/(L) on disposal = SP - NBV at the date disposal

Bal in RS pertaining to the NCA disposed of

Transfer from RS to RE =

Extract of SOPL & OCI for the Y/E 31.12 X1 X2 X3 X4


Other exp - Depn pa
- Loss on Disp

OCI - Gain /(Loss) on Revl

Extract of SOFP as at 31.12. X1 X2 X3 X4


NCA: HC / Revl Amt
AD / Subsq AD
NBV

Eq: RS

Exercise 4.6.3 (f)


The following information is available for the year ended 31 October 2012:
$
Property
Cost as at 1 November 2011 102,000
Accumulated depreciation as at 1 November 2011 (20,400)
81,600

On 1 November 2011, the company revalued the property to $150,000.

The company’s policy is to charge depreciation on a straight-line basis over 50 years. On revaluation
there was no change to the overall useful economic life. It has also chosen to make an annual transfer
of the excess depreciation on revaluation between the revaluation reserve and retained earnings.

What should be the balance on the revaluation surplus and the depreciation charge as shown in
the financial statements for the year ended 31 October 2012?

Depreciation charge Revaluation surplus


$_______________ $______________

89
ACCA Financial Reporting (FR)
CHAPTER 4: IAS 16 PROPERTY, PLANT AND EQUIPMENT

4.7 De-recognition of PPE

Property, plant and equipment should be derecognised when it is no longer expected to generate
future economic benefit or when it is disposed of.

When property, plant and equipment is to be derecognised, a gain or loss on disposal is to be


calculated. This can be found by comparing the difference between:

Carrying value X
Disposal proceeds (Net sale proceeds) X
Profit or loss on disposal (recognised to SOPL) XX

Example 4.7 Asset write off

An entity freehold land has a cost of $22m. It has been revalued at $28m. The $6 m surplus
has been credited to the revaluation surplus within equity. It is sold for $30 m. $2 m is shown
as a gain on disposal in the profit or loss. The $6m is transferred directly from the revaluation
surplus to retained earnings, with no impact on the profit or loss.

4.8 Disclosure

The financial statement shall disclose, for each class of property, plant and equipment:

(a) The measurement bases used for determining the gross carrying amount
(b) The depreciation method used;
(c) The useful lives or the depreciation rates used;
(d) The gross carrying amount and the accumulated depreciation (aggregated with accumulated
impairment losses) at the beginning and end of the period; and
(e) A reconciliation of the carrying amount at the beginning and end of the period showing:

(i) Additions
(ii) Assets classified as held for sale or included in a disposal group classified as held for sale
in accordance with IFRS 5 and other disposals;
(iii) Acquisitions through business combinations;
(iv) Increase or decreases resulting from revaluations and from impairment losses recognised
or reversed in other comprehensive income in accordance with IAS 36;
(v) Impairment losses recognised in profit or loss in accordance with IAS 36;
(vi) Impairment losses reversed in profit or loss in accordance with IAS36;
(vii) Depreciation
(viii) Other changes.

90
ACCA Financial Reporting (FR)
CHAPTER 4: IAS 16 PROPERTY, PLANT AND EQUIPMENT

Selection of the depreciation method and estimation of the useful life of assets are matters of
judgment. Therefore, disclosure of the method adopted and the estimated useful lives or
depreciation rates provides users of financial statements with information that allows them to
review the policies selected by management and enables comparison to be made with other
entities.

For similar reasons, it is necessary to disclose:

(i) Depreciation, whether recognised in profit or loss or as a part of the cost of other assets,
during a period; and
(ii) Accumulated depreciation at the end of the period.

In accordance with IAS 8 an entity discloses the nature and effect of a change in an accounting
estimate that has an effect in the current period or is expected to have an effect in subsequent
periods.

For property, plant and equipment, such disclosure may arise from changes in estimates with
respect to:

(i) Residual values;


(ii) The estimated costs of dismantling, removing or restoring items of property, plant and
equipment
(iii) Useful lives; and
(iv) Depreciation methods.

If items of property, plant and equipment are stated at revalued amounts, the following shall
be disclosed:

(i) the effect date of revaluation;


(ii) whether an independent valuer was involved;
(iii) the methods and significant assumptions applied in estimating the items’ fair values;
(iv) the extent to which the items’ fair values were determined directly by reference to
observable prices in an active market or recent market transactions on arm’s length terms
or were estimated using other valuation techniques;
(v) for each revalued class of property, plant and equipment, the carrying amount that would
have been recognised had the assets been carried under the cost models; and
(vi) the revaluation surplus, indication the change for the period and nay restrictions on the
distribution of the balance to shareholders.

Users of financial statements may also find the following information relevant to their needs:

(i) the carrying amount of temporarily idle property, plant and equipment;
(ii) the gross carrying amount of any fully depreciated property, plant and equipment that is
still in use;

91
ACCA Financial Reporting (FR)
CHAPTER 4: IAS 16 PROPERTY, PLANT AND EQUIPMENT

(iii) the carrying amount of property, plant and equipment retired from active use and not
classified as held for sale in accordance with IFRS 5; and
(iv) when the cost model is used, the fair value of property, plant and equipment when this is
materially different from the carrying amount.

The following format (with notional figures) is commonly used to disclose non-current asset
movements.

Total Land & buildings Plant & equipment


$ $ $
Cost/valuation
At 1 Jan 20X4 5,000 40,000 10,000
Revaluation surplus 12,000 12,000 -
Additions in year 4,000 - 4,000
Disposals in year -1,000 - -1,000
At 31 December 20X4 20,000 52,000 13,000

Accumulated Depreciation
At 1 Jan 20X4 16,000 10,000 6,000
Charge for the year 4,000 1,000 3,000
Eliminated on disposals -500 - -500
At 31 December 20X4 19,500 11,000 8,500

Net book values/CV


At 31 December 20X4 45,500 41,000 4,500
At 1 Jan 20X4 34,000 30,000 4,000

92
ACCA Financial Reporting (FR)
CHAPTER 4: IAS 16 PROPERTY, PLANT AND EQUIPMENT

4.9 Check Understanding

Question 1 (Extract of Dec'05):


Elite Leisure is a private limited liability company that operates a single cruise ship. The ship was
acquired on 1 October 19W5. Details of the cost of the ship's components and their EUL are:

Component Original cost ($'m) Depreciation basis


Ship's fabric (hull, decks etc.) 300 25 years SLM
Cabins and entertainment area fittings 150 12 years SLM
Propulsion system 100 EUL of 40,000 hours

At 30 September 20X3 no further capital expenditure had been incurred on the ship.

In the year ended 30 September 20X3 the ship had experienced a high level of engine trouble which
had cost the company considerable lost revenue and compensation costs. The measured expired life
of the propulsion system at 30 September 20X3 was 30,000 hours. Due to the unreliability of the
engines, a decision was taken in early October 20X3 to replace the whole of the propulsion system at
a cost of $140 million. The expected life of the new propulsion system was 50,000 hours and in the
year ended 30 September 20X4 the ship had used its engines for 5,000 hours.

At the same time as the propulsion system replacement, the company took the opportunity to do a
limited upgrade to the cabin and entertainment facilities at a cost of $60 million and repaint the ship's
fabric at a cost of $20 million. After the upgrade of the cabin and entertainment area fittings it was
estimated that their remaining life was five years (from the date of the upgrade). For the purpose of
calculating depreciation, all the work on the ship can be assumed to have been completed on 1
October 20X3. All residual values can be taken as nil.

Calculate the carrying amount of Elite Leisure's cruise ship at 30 September 20X3 and prepare
extracts in respect of it from Elite Leisure's SOPL & OCI fye 30 September 20X4 and its SOFP.

93
ACCA Financial Reporting (FR)
CHAPTER 4: IAS 16 PROPERTY, PLANT AND EQUIPMENT
Solution:
Extract of SOPL & OCI for the year ended 30 September 20X4 $'m
Expenses:
Depn - ship's fabric
- cabin & entertainment fittings
- propulsion system
Write off of propulsion system
Repainting ship's fabric

Extract of SOFP at at 30 September 20X4


NCA: Cruise ship (W1)

Cabin & entertainment Propulsion


W1 Ship's fabric fittings system
Cost 300 150 100
Less: Op Acc depn -96 -100 -75
Carrying amount @ 30/9/X3 204 50 25
Write off - - -25
Upgrading/ Addition - 60 140
Depn charge -12 -22 -14
Carrying amount @ 30/9/X4 192 88 126

94
ACCA Financial Reporting (FR)
CHAPTER 4: IAS 16 PROPERTY, PLANT AND EQUIPMENT

Question 2 (Extract of Dec'01):

Kent acquired a 12-year lease on a property on 1 October 20X0 at a cost of $240,000. The company
policy is to revalue its properties to their market values at the end of each year and make a transfer to
realised profit in respect of excess amortisation on revalued properties. The market values of the
property on 30 September 20X1 and 20X2 were $231,000 and $175,000 respectively. The existing
balance on the revaluation surplus at 1 October 20X0 was $50,000. This related to some non -
depreciable land whose value had not changed significantly since 1 October 20X0.

Prepare extracts of the FS of Kent (including the movement on the revaluation surplus) for the years
to 30 September 20X1 and 20X2 in respect of the leasehold property.

Solution:

Extract of SOPL for the year ended 30 Sep 20X1 20X2


Expenses - amortisation
- revaluation loss

OCI - revaluation gain / (loss)

Extract of SOFP as at 30 Sep 20X1 20X2


NCA: leasehold property

Equity: RS (W1)

W1: Movement of RS $'000


Balance at 1 October 20X0
Revaluation gain

Transfer to retained earnings


Revaluation loss

95
ACCA Financial Reporting (FR)
CHAPTER 4: IAS 16 PROPERTY, PLANT AND EQUIPMENT

Question 3 (Extract of Dec'07)

The following extract of trial balance relates to Llama, a listed company, at 30 September 2007:

$'000 $'000
Land and buildings - at valuation 1 October 2006 (note (i)) 130,000
Plant - at cost (note (i)) 128,000
Accumulated depreciation of plant at 1 October 2006 32,000
Investments in equity instruments 26,500
Investment income 2,200
Cost of sales 89,200
Distribution costs 11,000
Administrative expenses 12,500
Loan interest paid 800
Inventory at 30 September 2007 37,900
Trade receivables 35,100
Revenue 180,400
Equity shares of 50 cents each fully paid 60,000
Retained earnings at 1 October 2006 25,500
2% loan note 80,000
Trade payables 46,300
Revaluation reserve (arising from land and buildings) 14,000
Suspense account (note (ii)) 24,000
Bank 6,600
471,000 471,000

Notes:

i. Llama has a policy of revaluing its land and buildings at each year end. The valuation in the trial
balance includes a land element of $30 million. The estimated remaining life of the buildings at that
date (1 Oct 2006) was 20 years. On 30 September 2007, a professional valuer valued the buildings
at $92 million with no change in the value of the land. Depreciation of buildings is charged 60% to
cost of sales and 20% each to distribution costs and administrative expenses.

96
ACCA Financial Reporting (FR)
CHAPTER 4: IAS 16 PROPERTY, PLANT AND EQUIPMENT

During the year Llama manufactured an item of plant that it is using as part of its own operating
capacity. The details of its cost, which is included in cost of sales in the trial balance, are:
$'000
Materials cost 6,000
Direct labour cost 4,000
Machine time cost 8,000
Directly attributable overheads 6,000

The manufacture of the plant was completed on 31 March 2007 and the plant was brought into
immediate use, but its cost has not yet been capitalised.
All plant is depreciated at 12.5% per annum (time apportioned where relevant) using the reducing
balance method and charged to cost of sales. No non-current assets were sold during the year.

ii. The suspense account contains the corresponding credit entry for the proceeds of a rights issue on
1 July 2007. The terms of the issue were one share for every four held at 80 cents per share

iii. The estimated income tax liability for the year ended 30 September 2007 is $18·7 million.

Required:
Prepare SOPL & OCI, SOFP and SOCIE for the year ended 30 Sep 2007.

Solution:

97
ACCA Financial Reporting (FR)
CHAPTER 4: IAS 16 PROPERTY, PLANT AND EQUIPMENT

4.10 Chapter 4 Summary

Diagram 4.10.: Summary of IAS 16 Property, Plant & Equipment

98
ACCA Financial Reporting (FR)
CHAPTER 5: IAS 38 INTANGIBLE ASSETS

CHAPTER 5: IAS 38 INTANGIBLE ASSETS


LEARNING OUTCOME
At the end of the chapter, you should be able to:
TLO B2a. Discuss the nature and accounting treatment of internally generated and purchased
intangibles.
TLO B2b. Distinguish between goodwill and other intangible assets.

TLO B2c. Describe the criteria for the initial recognition and measurement of intangible assets

TLO B2d. Describe the subsequent accounting treatment, including the principle of impairment
tests in relation to goodwill.

TLO B2e. Indicate why the value of purchase consideration for an investment may be less than
the value of the acquired identifiable net assets and how the difference should be
accounted for

TLO B2f. Describe and apply the requirements of relevant accounting standards to research and
development expenditure.

99
ACCA Financial Reporting (FR)
CHAPTER 5: IAS 38 INTANGIBLE ASSETS

5.1 Overview

5.1.0 Rational Mapping

IAS 38 in relation to other standards in context of Non-Current Asset

Diagram 5.1 Relational Mapping

Historically, corporate success has been built on physical assets and improving manufacturing efficiency.
However, although the most important assets for many new economy businesses are now brands, market
positions, knowledge capital and people, these are rarely recognised in financial statements.

Intangible assets, including Goodwill proved to be contentious areas for accounting standard setters.
Many companies (especially service sectors) have recognized major resources in the form of intangible
assets (eg brand name, customer loyalty, skills of staff, etc.) as compared to Tangible assets.

As measurement of their cost is not reliable, accounting practice do not recognize intangible assets unless
there is a market transaction (like a separate acquisition / business combination) to obtain a reliable
value for intangibles

For example, Goodwill on acquisition is recognized but Goodwill that is internally generated in the books
of subsidiary is not recognized in its own individual books

100
ACCA Financial Reporting (FR)
CHAPTER 5: IAS 38 INTANGIBLE ASSETS

5.1.1 Definition of Intangible Assets

Asset: Resource that is controlled by the entity as a result of past events (for example, purchase or self-
creation) and from which future economic benefits (inflows of cash or other assets) are expected.

Intangible asset: an identifiable non-monetary asset without physical substance.

Common Examples of Intangible assets include:


• computer software,
• patents,
• copyrights,
• motion picture films,
• client lists/customer lists,
• mortgage-servicing rights,
• licenses,
• import quotas,
• franchises,
• customer or supplier relationships/customer loyalty,
• market share and marketing rights.
• Trademarks (including brand names & publishing titles)
• Intellectual property

Items not meeting the definition of an intangible asset: identifiably, control and future benefits are
recorded as an expense, when they are incurred.

However, if the item is acquired in an acquisition (that is the acquisition of a subsidiary entity by its
parent entity) and does not meet the definition and recognition criteria, it forms part of the goodwill.

101
ACCA Financial Reporting (FR)
CHAPTER 5: IAS 38 INTANGIBLE ASSETS

5.2 Scope of IAS 38

IAS 38 applies to all intangible assets (example listed in topic 5.1.1) with certain exceptions.

Examples of assets specifically excluded from IAS 38 include:


(a) intangible assets held for sale, in the ordinary course of business (IAS 2 Inventories and Construction
Contracts)
(b) deferred tax assets (IAS 12 Income Taxes)
(c) leases that fall within the scope of IFRS 16 Leases
(d) assets arising from employee benefits (IAS 19 Employee Benefits)
(e) goodwill arising on a business combination (see IFRS 3 Business Combinations)
(f) financial assets, as defined in IAS 27, IAS 28, IAS 31, IAS 39
(g) the recognition and measurement of exploration and evaluation assets (IFRS 6) mineral rights and
expenditure on the exploration for, or development and extraction of, minerals, oil, natural gas and
similar non-regenerative resources
(h) non-current intangible assets classified as held for sale (or included in a disposal group that is
classified as held for sale) in accordance with IFRS 5
(i) deferred acquisition costs, and intangible assets arising in insurance, from contracts with
policyholders.

5.3 Recognition

Learning Outcome (ACCA Study Guide Area B)


B2a : Discuss the nature and accounting treatment of internally generated and purchased intangibles.
B2b : Distinguish between goodwill and other intangible assets.

An entity shall apply the recognition criteria in determining whether to recognise an intangible asset. The
entity shall recognise an intangible asset as an asset if, and only if:
(i) it is probable that the expected future economic benefits that are attributable to the asset will flow
to the entity;
(ii) the cost or value of the asset can be measured reliably; and
(iii) the asset does not result from expenditure incurred internally on an intangible item.

An entity shall assess the probability of expected future economic benefits using reasonable and
supportable assumptions that represent management’s best estimate (judgements) of the economic
conditions that will exist over the useful life of the asset.

102
ACCA Financial Reporting (FR)
CHAPTER 5: IAS 38 INTANGIBLE ASSETS

5.3.1 Recognition criteria

The recognition criteria are derived from the definition of intangible asset including:

The three critical attributes of an intangible asset are:


• Identifiably
• control (power to obtain benefits from the asset)
• future economic benefits (such as revenues or reduced future costs)

It is important to note that the definition of asset in this standard was not revised following the revision
of the definition of an asset in the Conceptual Framework for Financial Reporting issued in 2018.

Diagram 5.3.1 Definition and Recognition Criteria for Intangible Asset

Separate Acquisition

An asset is identifiable if it is either:


(i) separable (capable of being disposed of on their own, with the remainder of the business being
retained). OR
(ii) it arises from *contractual or other legal rights.

An asset is separable if it can be sold, transferred, exchanged, licensed or rented to another party on its
own rather than as part of a business.

*Even if an asset is not separable eg. company purchased a government grant that is not transferable and
it is not separable, it is still identifiable as it arises from contractual rights.

103
ACCA Financial Reporting (FR)
CHAPTER 5: IAS 38 INTANGIBLE ASSETS

Note:
IAS 38 goes further than the Conceptual Framework definition of an asset in that it requires that an
intangible asset must be “identifiable” so that it is “distinguishable from goodwill

An identifiable intangible asset which has a finite useful life is subject to annual amortization, whereas
an item which is not identifiable is either not recognized at all or, if it is subsumed in goodwill acquired
in a business combination, not amortised but subject to annual impairment reviews.

Control

According to the IASB Conceptual Framework definition of an asset, one of the characteristics of an asset
is that it is under the control of the entity, that is the entity has:

▪ Power to obtain the future economic benefits


▪ To restrict the access of others to those benefits

Usually, such control would come from legal rights that are enforceable in a court of law, for example
through licences, patents or trademarks.

In the absence of legal rights, it is more difficult to demonstrate control. However, legal enforceability of
a right is not a necessary condition for control because an entity may be able to control the future
economic benefits in some other way, for example through keeping the resource a secret from others.

Future economic benefit

The future economic benefits flowing from an intangible asset may include

• revenue from the sale of products or services,


• cost savings or other benefits resulting from the use of the asset by the entity.

For example, the use of intellectual property in a production process may reduce future production costs
rather than increase future revenues.

104
ACCA Financial Reporting (FR)
CHAPTER 5: IAS 38 INTANGIBLE ASSETS

5.3.2 Subsequent Expenditures

Subsequent expenditure should rarely be recognised in the carrying amount of an asset. This is because
in most cases the expenditure is incurred to maintain the expected future economic benefits embodied
in an existing asset.

In addition, it is often difficult to attribute subsequent expenditure directly to a particular intangible asset
rather than to the business as a whole.

Most subsequent costs are incurred to maintain the benefits arising from the existing asset (and not
acquire future benefits), so it fails the ‘attributable future benefits’ test. It is often not possible to reliably
measure the costs in relation to a particular intangible as separate from the cost of maintaining the
business as a whole, such as:

- Start-up costs
- staff training costs
- advertising costs
- business relocation costs
- Internally generated brands, mastheads, publishing tittles, customer lists- these cannot be
capitalized, although capitalized if acquired externally
- Internally generated Goodwill

These and all other expenditure related to intangible asset which do not meet the criteria as an
Identifiable Intangible Assets or as Goodwill arising on a business combination (acquired) should be
expensed as incurred.

Reinstatement is prohibited

Example 5.3.2: Subsequent Expenditure

Subsequent expenditure on brands, mastheads, publishing titles, customer lists and items similar in
substance is charged as expenses to Statement of Profit or Loss.

105
ACCA Financial Reporting (FR)
CHAPTER 5: IAS 38 INTANGIBLE ASSETS

5.4 Measurement

Learning Outcome (ACCA Study Guide Area B)


B2c : Describe the criteria for the initial recognition and measurement of intangible assets
B2d : Describe the subsequent accounting treatment, including the principle of impairment tests in
relation to goodwill.

As stated in the recognition criteria the cost of the asset can be recognised if it can be measured reliably.
This requirement applies to costs incurred initially to acquire or internally generate an intangible asset
and those incurred subsequently to add to, replace part of, or service it.

Diagram 5.4: Measurement of Intangible Asset

106
ACCA Financial Reporting (FR)
CHAPTER 5: IAS 38 INTANGIBLE ASSETS

5.4.1 Initial Measurement

Diagram 5.4.1: Initial Measurement

Separate Acquisition

In most cases, separately acquired intangibles satisfy the IAS 38 recognition criteria because:
(a) price an entity pays will reflect expectations about the probability that the expected future economic
benefits embodied in the asset will flow to the entity.
(b) The probability recognition criterion in is always considered to be satisfied for separately acquired
intangible assets.
(c) the cost of a separately acquired IA can usually be measured reliably.
The cost (ie fair value consideration paid or transferred) of an intangible asset comprises:
⚫ purchase price
⚫ import duties
⚫ non-refundable purchase taxes
⚫ professional and legal fees
⚫ directly-attributable expenditure including staff pay and benefits on preparing the asset for its
intended use including testing.
⚫ Deductions, such as trade discounts and rebates.

107
ACCA Financial Reporting (FR)
CHAPTER 5: IAS 38 INTANGIBLE ASSETS

Recognition of costs in the carrying amount of an intangible asset ceases when the asset is in the condition
necessary for it to be capable of operating in the manner intended by management, irrespective of whether it
is put into use immediately or not.
Therefore, costs incurred in using or redeploying an intangible asset are not included in the carrying amount
of that asset.

The following costs are not included in the carrying amount of an intangible asset:
a. costs incurred while an asset capable of operating in the manner intended by management has yet to be
brought into use; and
b. initial operating losses, such as those incurred while demand for the asset’s output builds up.
c. Administration and other general overhead cost, cost of conducting business in a new location or with a
new class of customer (including staff training).

Exercise 5.4.1 (b) – Initial cost of Intangible Asset


Data Ltd acquires new technology that will revolutionise its current manufacturing process. Costs incurred
are as follows:
$’000
Original cost of new technology 1,200
Discount received/rebate 120
Staff training incurred in operating the new process 60
Testing of the new manufacturing process 12
Losses incurred while other parts of the plant stood 24
idle.

Required: Find the initial cost of Intangible Asset

Answer:

108
ACCA Financial Reporting (FR)
CHAPTER 5: IAS 38 INTANGIBLE ASSETS

5.4.2 Subsequent Measurement


Cost Model

After initial recognition, an intangible asset should be carried at its cost, less any accumulated
amortisation and impairment losses.
(i) With Indefinite useful life
(ii) With Finite useful life

Revaluation Model

After initial recognition, an asset should be carried at its fair value at the date of revaluation less any
subsequent accumulated amortisation and accumulated impairment losses. Revaluations should be made
with sufficient regularity, so that the carrying amount does not differ materially from the fair value. The
revaluation treatment is applied after an asset has been initially recorded at cost.
The frequency of revaluations depends on the volatility of the fair values of the intangible assets being
revalued. If the fair value of a revalued asset differs materially from its carrying amount, a further
revaluation is necessary.
Revalued amount (*FV at date of revaluation) - subsequent accumulated amortisation –Accumulated
Impairment Loss
IAS 38 states that companies can only chose a revaluation model if there is an active market.

109
ACCA Financial Reporting (FR)
CHAPTER 5: IAS 38 INTANGIBLE ASSETS

Table [Link] Fair Value with reference to an Active Market

* If the fair value of the asset can be determined by reference to an active market later, the revaluation
is applied from that date.

ACTIVE MARKET MAY EXIST ACTIVE MARKET CANNOT EXIST

CAN APPLY REVALUATION MODEL CANNOT APPLY REVALUATION MODEL


It is uncommon for an active market to exist Brands, newspaper mastheads, music and film
for an intangible asset, although this may publishing rights, patents or trademarks
happen due to: because:
• The items traded are homogeneous • such assets are unique.
• Willing buyers and sellers can normally be • Transactions are infrequent,
found at any time • price paid for one asset may not provide
• Prices are available to the public. sufficient evidence of the fair value of
another,
For example, in some jurisdictions, an active • prices are not available to the public,
market may exist for freely transferable taxi contracts are negotiated between
licences, fishing licences or production quotas. individual buyers and sellers

Therefore, in practice the revaluation model will usually not be available


If an asset is revalued, all the other assets in its entire class should also be revalued.

Illustration

An entity owns various franchises. They may be all held at cost, or to revalue all of them. It is not a llowed
to revalue a single franchise.

A class of intangible assets is a grouping of assets of a similar nature and use. These are revalued
simultaneously to avoid selective revaluation of assets that would distort the disclosed portfolio value

110
ACCA Financial Reporting (FR)
CHAPTER 5: IAS 38 INTANGIBLE ASSETS

5.5 Different ways of obtaining intangible assets

Diagram 5.5: Acquisition of intangible assets

5.5.1 Separate acquired intangible assets

In most cases, separately acquired intangibles satisfy the IAS 38 recognition criteria.
Examples of separately acquired Intangible Assets (IA) include:

• Brands
• Mastheads
• Publishing
• Titles
• Licences
• computer software
• copyrights
• patents
• airport landing slots

111
ACCA Financial Reporting (FR)
CHAPTER 5: IAS 38 INTANGIBLE ASSETS

Government Grant

In some cases, an Intangible Asset may be acquired free of charge, or for nominal consideration, by way
of a Government Grant. This may happen when a government transfers or allocates to an entity IA such
as airport landing rights, licences to operate radio or television stations, import licences or quotas or rights
to access other restricted resources.

Examples of Government Grants include:

⚫ airport landing rights,


⚫ licences to operate radio, or television, stations,
⚫ import licences, or quotas or
⚫ rights to access other restricted resources.

In accordance with IAS 20 Accounting for Government Grants and Disclosure of Government Assistance,
an entity may choose to recognise both the intangible asset and the grant initially at fair value.

If an entity chooses not to recognise the asset initially at fair value, the entity recognises the asset initially
at a nominal amount (the other treatment permitted by IAS 20) plus any expenditure that is directly
attributable to preparing the asset for its intended use.

Example 5.5.1 (a)


Company A has been granted an exclusive five-year licence to manufacture and distribute a new
product. Although the licence had no direct cost to the Co, its directors feel its granting is a
reflection of the company's standing and have asked to value the licence. Accordingly, they have
placed a value of $100,000 on it.

Answer:
lAS 38 states that assets acquired as a result of a Government Grant may be capitalised at Fair
Value, along with a corresponding credit for the value of the grant.

112
ACCA Financial Reporting (FR)
CHAPTER 5: IAS 38 INTANGIBLE ASSETS

Asset exchange

Intangible assets may be acquired in exchange for a non-monetary asset or asset. The cost is measured
at fair value. If the acquired asset is not measured at fair value, its cost is measured at the carrying amount
of the asset given up.

An entity shall measure the cost of such an intangible asset at fair value unless
(a) the exchange transaction lacks commercial substance; or
(b) the fair value of neither the asset received nor the asset given up is reliably measurable.
In such cases, the asset’s cost is measured at the carrying amount of the asset given up.

Examples 5.5.1(b) – measurement of intangible assets acquired in an exchange of assets

Illustration 1:

On 1 January 20X1 an entity received landing rights at a local airport in exchange for 100 ounces of gold,
when gold was trading at CU1,000 per ounce.

The landing rights received (the intangible asset acquired in the exchange transaction) must be measured
at CU100,000 (their fair value) on initial recognition. Because there is an active market for gold, the fair
value of the landing rights received is most easily determined by reference to the fair value of the gold
given up in the exchange transaction (ie 100 ounces × CU1,000 per ounce).

Illustration 2:

On 1 January 20X1 an entity received landing rights at a local airport in exchange for 90,000 litres of
aviation fuel and CU10,000 cash. Aviation fuel costs CU1 per litre.

The landing rights received (the intangible asset acquired in the exchange transaction) must be measured
at CU100,000 (their fair value) on initial recognition. The fair value of the landing rights is determined by
reference to the fair values of the aviation fuel CU90,000 (ie 90,000 litres × CU1 per litre) plus CU10,000
cash given up in the exchange transaction.

(Source from: [Link]/content/.../1/.../Module18_version20109_IntangibleAssets_.pdf)

113
ACCA Financial Reporting (FR)
CHAPTER 5: IAS 38 INTANGIBLE ASSETS

Business combination

Intangible assets acquired as part of a business combination are normally considered to meet the
recognition criteria of IAS 38. The cost of an intangible asset acquired as part of a business combination
should be measured at its fair value at the date it was acquired in accordance with IFRS 13 Hierarchy
Level.

However, an intangible asset acquired in a business combination is not recognised when it arises from
legal or other contractual rights and its fair value cannot be measured reliably because the asset either

(a) is not separable from goodwill, or


(b) is separable from goodwill but there is no history or evidence of exchange transactions for the same
or similar assets, and otherwise estimating fair value would be dependent on immeasurable variables.

In accordance to IFRS 3 Business Combinations, if an Intangible asset is acquired in a business


combination, the cost of that Intangible Asset is at its fair value at the acquisition date if meets the
following recognition criteria:

• meets the definition of an Intangible Asset [IAS38] and


• fair value can be measured reliably

An acquirer shall recognise the Intangible Asset separately from Goodwill at the acquired date if the Fair
Value of the Intangible Asset at Date of Acquisition can be measured with sufficient reliability. If the
criteria are not met, then the cost should be subsumed within Goodwill. [IFRS3 override IAS38]

Exercise 5.5.1 (b)


H acquired 100% ordinary share capital of S for $150m. At the DOA, the FV of NA of S was $100m. In
addition, S has an internally generated brand name which valued at $30m by a reputable valuer just prior
to its acquisition by H.

SOFP H ($'m) S ($'m) CSOFP $'m


PPE 100 100

Invst in S 150

OSC 250 100

114
ACCA Financial Reporting (FR)
CHAPTER 5: IAS 38 INTANGIBLE ASSETS

5.5.2 Internally generated intangible assets

Learning Outcome (ACCA Study Guide Area B)


B2a : Discuss the nature and accounting treatment of internally generated and purchased intangibles.
B2f : Describe and apply the requirements of relevant accounting standards to research and
development expenditure.

IAS 38 prohibits recognition of internally generated goodwill and any internally generated intangibles
[those that cannot meet recognised criteria]. The only exception to this rule is development expenditure
that meet the recognition criteria.

Example 5.5.2:
An enterprise may have a portfolio of customers or market share and expect that, due to its efforts in
building customer relationships and loyalty, the customers will continue to trade with the enterprise.
Therefore, the entity wants to capitalise the expenditures on these items

Solution:
However, in the absence of legal rights to protect, or other ways to control, the relationships with
customers or the loyalty of the customers to the enterprise, the enterprise usually has insufficient control
over the economic benefits from customer relationships and loyalty and therefore fails the basic definition
of an asset. The expenditures on these items cannot be capitalised.

Diagram 5.5.2: Internally generated Intangible Assets

115
ACCA Financial Reporting (FR)
CHAPTER 5: IAS 38 INTANGIBLE ASSETS

Research and development

To assess whether an internally-generated research and development expenditure meets the criteria for
recognition as IA, an entity splits the generation of the asset into:
(i) RESEARCH PHASE

As research is the original planned investigation undertaken with the prospect of gaining new
scientific or technical knowledge and understanding, there is insufficient certainty that the
expenditure will generate future economic benefits. Thus the Accounting treatment for Research
Phase expenditure will result in:

Accounting treatment

• NO intangible asset is created in this phase


▪ to be written off as expense when it is incurred.
Dr Research Expenses [SOPL]
Cr Bank/ Payable

(ii) DEVELOPMENT PHASE

Development is the application of research findings or other knowledge to a plan or design for the
production of new or substantially improved materials, devices, products, processes or services
before the start of commercial production.

Accounting treatment for Development

MUST recognize as intangible assets if met the following criteria [IT-MAP]

i. Technical feasibility of completing intangible assets


ii. Intention to complete it
iii. Ability to use/ sell it
iv. Probable inflow of future eco. benefits
v. Cost can be measured reliably

If MET ALL criteria If DO NOT MEET ALL criteria

Dr Intangible Asset [SOFP] Dr Development Exp [SOPL]


Cr Bank / Payables Cr Bank / Payables

Costs – that CAN be capitalised Costs – that CANNOT be capitalised


▪ Materials & services consumed ▪ Selling, Administration & general
▪ Direct staff costs to develop the assets overheads
– staff salaries ▪ Inefficiencies & initial operating losses
▪ Direct expenses – patent costs ▪ Staff training for the use of the asset

116
ACCA Financial Reporting (FR)
CHAPTER 5: IAS 38 INTANGIBLE ASSETS

Exercise 5.5.2 (a)


Which one of the following could be classified as deferred development expenditure in M’s statement
of financial position as at 31 March 2010 according to IAS 38 Intangible assets?

A. $120,000 spent on developing a prototype and testing a new type of propulsion system for trains.
The project needs further work on it as the propulsion system is currently not viable.
B. A payment of $50,000 to a local university’s engineering faculty to research new environmentally
friendly building techniques
C. $35,000 spent on consumer testing a new type of electric bicycle. The project is near completion and
the product will probably be launched in the next twelve months. As this project is the first of its kind
for M it is expected to make a loss.
D. $65,000 spent on developing a special type of new packaging for a new energy efficient light bulb.
The packaging is expected to be used by M for many years and is expected to reduce M’s distribution
costs by $35,000 a year.

Goodwill

Goodwill is created by good relationship between a business and its customer by building up a reputation
for high quality products/ services. Goodwill are characterised by:

(a) Not an intangible asset


(b) It is not identifiable resource (not separable nor does it arise from contractual or other legal rights)
(c) CANNOT be measured reliably at cost.
(d) Market value of an entity less carrying amount of its identifiable net assets = range of factors that
affect the value of the entity doesn’t represent the cost of Intangible Assets controlled by the entity.

As the key issues affecting such products or services as intangible assets are:

• Recognition – whether it satisfied the definition of assets


• Measurement – whether the cost or value can be measured reliably.

However, If, such types of expenditure satisfied the recognition criteria as an asset and the amount can
be measured reliably then it should be recognized as an asset. IAS 38 Intangible Assets provides guidance
in this area including:

• Only goodwill arising from a business combination is recognized (Purchased GW - IFRS 3)


• Goodwill cannot be separated from the business that it belongs to and therefore cannot be
purchased separately from other assets. Hence, internally generated goodwill must not be
capitalized.

117
ACCA Financial Reporting (FR)
CHAPTER 5: IAS 38 INTANGIBLE ASSETS

5.6 Amortisation

Amortisation is a systematic allocation of the cost or revalued amount less any residual value
recognised as an expense over the asset’s useful life. As the benefits embodied in an intangible asset
are consumed over time, the carrying amount of the asset is reduced to reflect that consumption.
Diagram 5.6.1 Determination of Useful Life

118
ACCA Financial Reporting (FR)
CHAPTER 5: IAS 38 INTANGIBLE ASSETS

5.6.1 Amortisation Period

The depreciable amount of an asset should be allocated, over its useful life. Amortisation should
commence when the asset is available for use.

Amortisation is recorded whether, or not, there has been an increase in the asset’s fair value or
recoverable amount. Many factors need to be considered in determining the useful life of an intangible
asset including:

(a) the expected usage of the asset and whether the asset could be efficiently managed by another
management team
(b) typical product life cycles for the asset
(c) technical, technological or other types of obsolescence
(d) the stability of the industry in which the asset operates and changes in the market demand for the
products, or services, produced by the asset
(e) expected actions by competitors
(f) the cost of maintenance required to obtain the future benefits from the asset and the company’s
ability and intent to reach such a level
(g) the period of control over the asset and legal or similar limits on the use of the asset, such as the
expiry dates of related leases
(h) whether the useful life of the asset is dependent on the useful life of other assets of the entity.

Example 5.6.1

An entity purchased an exclusive right to generate hydro-electric power for forty years at a
cost of $80 m. The costs of this power are much lower than those from alternative sources.
It is expected that there will be sufficient demand for at least forty years. Therefore, amortise
the right to generate power, over forty years.

Determination of Useful Life

If control over the benefits from an intangible asset is achieved through legal rights, that have been
granted for a finite period, the useful life of the intangible asset should not exceed the period of the
legal rights unless:
a. the legal rights are renewable
b. Renewal is virtually certain.

119
ACCA Financial Reporting (FR)
CHAPTER 5: IAS 38 INTANGIBLE ASSETS

5.6.2 Amortisation methods

While amortisation shall begin when the asset is available for use, it shall cease at the earlier of the
date that the asset is classified as held for sale (or included in a disposal group that is classified as held
for sale) in accordance with IFRS 5 and the date that the asset is derecognised.

The amortisation method used should reflect the pattern in which the asset’s benefits are consumed.
The amortisation charge for each period should be recorded as an expense.

Amortisation methods include:

(i) The straight-line method


(ii) The diminishing balance method
(iii) The unit of production method

By default, the straight-line method should be used. The method selected is based on the likely
consumption of benefits, and is consistently applied from period to period. The amortization charge
for each period should normally be recognized in profit or loss.

NOTE: Review of amortisation period and amortisation method

The amortisation period and the amortisation method for an intangible asset with a finite useful life
shall be reviewed at least at each financial year-end. If the expected useful life of the asset is different
from previous estimates, the amortisation period shall be changed accordingly.

If there has been a change in the expected pattern of consumption of the future economic benefits
embodied in the asset, the amortisation method shall be changed to reflect the changed pattern. Such
changes shall be accounted for as changes in accounting estimates in accordance with IAS 8.

5.6.3 Residual Value

Residual values can only be taken into account in very limited circumstances, as so few markets exist
for intangible assets,

The residual value of an intangible asset should be assumed to be zero unless:

(a) there is a commitment by a third party to purchase the asset at the end of its useful life or

(b) there is an active market for the asset and:

(c) residual value can be determined by reference to that market and it is probable that such a
market will exist at the end of the asset’s useful life.

The depreciable amount of an asset is determined after deducting its residual value. Any residual value
implies that an entity expects to dispose of the intangible asset before the end of its economic life.

120
ACCA Financial Reporting (FR)
CHAPTER 5: IAS 38 INTANGIBLE ASSETS

5.7 De-recognition

An intangible asset shall be de-recognised (eliminated from the statement of financial position):

a) on disposal; or
b) when no future economic benefits are expected from its use or disposal.
Gains or losses arising from the retirement or disposal are the difference between the net disposal
proceeds and the carrying amount of the asset. They should be recorded as income or expense in the
SOPL.

Gains or Losses on disposal in the profit or loss as Gains or Losses = SP – NBV

Amortisation of an intangible asset with a finite useful life does not cease when the intangible asset is
no longer used, unless the asset has been fully depreciated or is classified as held for sale (or included
in a disposal group that is classified as held for sale) in accordance with IFRS 5.

5.8 Disclosure

The following for each class of intangible assets should be disclosed, distinguishing between internally-
generated intangible assets (development cost) and other intangible assets:
(a) useful lives, or the amortisation rates used
(b) amortisation methods used
(c) gross carrying amount and the accumulated amortisation (aggregated with accumulated
impairment losses) at the beginning and end, of the period
(d) line item(s) of the statement of profit or loss in which the amortisation of intangible assets is
included
(e) reconciliation of the carrying amount at the beginning and end, of the period showing:
(i) additions, indicating separately, those from internal development and through business
combinations
(ii) assets classified as held for sale or included in a disposal group classified as held for sale in
accordance with IFRS 5 and other disposals;
(iii) increases, or decreases, during the period resulting from revaluations and from impairment
losses recorded, or reversed directly in equity

121
ACCA Financial Reporting (FR)
CHAPTER 5: IAS 38 INTANGIBLE ASSETS

5.9 Check Understanding

Question 1

At 1 Jan 2018, DaBoss acquires copyrights to original recording of Ed’s Perfect. The agreement with
the singer allows the co. to record the singer for a period of 5 years. During the initial 6-month period
of the agreement, the singer is very sick and could not consequently record and the studio time at that
period was blocked has to be paid even the singer could not sing. The costs incurred by the co was:
$’000
Legal cost of acquiring the copyrights 10,000
Operational loss [studio time loss etc.] during start-up 2,000
Massive advertising campaign to launch artist 1,000

Prepare extract of SOPL & SOFP as at 31 Dec 2018.

122
ACCA Financial Reporting (FR)
CHAPTER 5: IAS 38 INTANGIBLE ASSETS

Question 2

Extract of trial balance as at 30 Sep 2008 $'000 $'000


Capitalised development expenditure at 1 October 2007 20,000
Accumulated amortisation at 1 October 2007 6,000

In addition to the capitalised development expenditure (of $20 million), further research and
development costs were incurred on a new project which commenced on 1 October 2007. The
research stage of the new project lasted until 31 December 2007 and incurred $1·4 million of costs.
From that date the project incurred development costs of $800,000 per month. On 1 April 2008 the
directors became confident that the project would be successful and yield a profit well in excess of its
costs. The project is still in development at 30 September 2008.

Capitalised development expenditure is amortised at 20% per annum using the straight-line method.

Required: Show SOPL and SOFP extracts for the year ended 30 Sep 2008.

123
ACCA Financial Reporting (FR)
CHAPTER 5: IAS 38 INTANGIBLE ASSETS

Question 3

The following trial balance relates to Candy at 30 September 20X8:

$'000 $'000
Leasehold property - at valuation 1 October 20X7 [note (i)] 50,000
Plant and equipment - at cost [note (i)] 76,600
Plant and equipment - accumulated depreciation at 1 October 20X7 24,600
Capitalised development expenditure - at 1 October 20X7 [note (ii)] 20,000
Development expenditure - accumulated amortisation at 1 October 20X7 6,000
Closing inventory at 30 September 20X8 20,000
Trade receivables 43,100
Bank 1,300
Trade payables and provisions 23,800
Revenue [note (i)] 305,800
Cost of sales 204,000
Distribution costs 14,500
Administrative expenses 22,200
Preference dividend paid 800
Interest on bank borrowings 200
Equity dividend paid 6,000
Research and development costs (note (ii)) 8,600
Equity shares of 25 cents each 50,000
8% redeemable preference shares of $1 each 20,000
Retained earnings at 1 October 20X7 24,500
Leasehold property revaluation reserve 10,000
466,000 466,000

The following notes are relevant:

I. Non-current assets - tangible:


The leasehold property had a remaining life of 20 years at 1 October 20X7. The company's policy is
to revalue its property at each year end and at 30 September 20X8 it was valued at $43 million.
Ignore deferred tax on the revaluation.

On 1 October 20X7 an item of plant was disposed of for $2.5 million cash. The proceeds have been
treated as sales revenue by Candy. The plant is still included in the above trial balance figures at its
cost of $8 million and accumulated depreciation of $4 million (to the date of disposal). All plant is
depreciated at 20% per annum using the reducing balance method. Depreciation and amortisation
of all non-current assets is charged to cost of sales.

124
ACCA Financial Reporting (FR)
CHAPTER 5: IAS 38 INTANGIBLE ASSETS

II. Non-current assets - intangible:


In addition to the capitalised development expenditure (of $20 million), further research and
development costs were incurred on a new project which commenced on 1 October 20X7. The
research stage of the new project lasted until 31 December 20X7 and incurred $1.4 million of costs.
From that date the project incurred development costs of $800,000 per month. On 1 April 20X8 the
directors became confident that the project would be successful and yield a profit well in excess of
its costs. The project is still in development at 30 September 20X8.
Capitalised development expenditure is amortised at 20% per annum using the straight-line
method. All expensed research and development is charged to cost of sales.

III. The directors have estimated the provision for income tax for the year ended 30 September 20X8
at $11.4 million.

Required: Prepare the SOPL & OCI, SOCIE & SOFP for the year ended 30 September 20X8.

125
ACCA Financial Reporting (FR)
CHAPTER 5: IAS 38 INTANGIBLE ASSETS

5.10 Chapter 5 Summary

Diagram 5.10.: Summary of IAS 38 Intangible Assets

126
ACCA Financial Reporting (FR)
CHAPTER 6: IAS 36 IMPAIRMENT OF ASSETS

CHAPTER 6: IAS 36 IMPAIRMENT OF


ASSETS
LEARNING OUTCOME
At the end of the chapter, you should be able to:
TLO B3a. Define, calculate and account for an impairment loss

TLO B3b. Account for the reversal of an impairment loss on an individual asset

TLO B3c. Identify the circumstances that may indicate the impairment to assets

TLO B3d. Describe what is meant by a cash generating unit

TLO B3e. State the basis on which impairment losses should be allocated, and allocate an
impairment loss to the assets of a cash generating unit

127
ACCA Financial Reporting (FR)
CHAPTER 6: IAS 36 IMPAIRMENT OF ASSETS

6.1 Overview

Whenever an asset’s recoverable amount (RA) falls to an amount less than its carrying amount (CA), it is
said to be impaired. Its carrying amount in the statement of financial position is therefore reduced to this
recoverable amount and, in most cases, the impairment loss is recognized as an expense and charged to
profit or loss

This is consistent with the definition of an asset / expenses in the conceptual framework as that part of
asset which has no future economic benefit and is not recoverable is written off as an expense (Conceptual
Framework defined as decrease in economic benefits in the form of asset decrease/liability increase not
resulting from distribution to equity participants).

IAS 36 puts in place a detailed method for carrying out impairment reviews and related accounting
treatments and disclosures. IAS 36 measures an impairment loss as the amount by which an asset’s
carrying amount exceeds its ‘recoverable amount’ defined as the higher of:

• Fair value less cost to sell; and


• Its value in use, which is the present value of the discounted estimated future cash flows arising
from the asset’s use and its ultimate disposal

(Source from: [Link]

128
ACCA Financial Reporting (FR)
CHAPTER 6: IAS 36 IMPAIRMENT OF ASSETS

6.1.0 Relational Mapping

Diagram 6.1 Relational Mapping (IAS 36 in relation to other standards in context of impairment of Asset)

6.1.1 Definition

Diagram 6.1.1 – Impairment of assets

Asset is impaired when:

Carrying amount (‘CA’)

(Historical Cost
> Recoverable amount (‘RA’)
– Accumulated Depreciation
– Accumulated Amortization
– Accumulated Impairment Loss)

Higher of asset’s / Cash Generating Unit’s

Cost Model Revaluation


(Historical Cost Model
– Accumulated (Revaluation
FV-cost to sell Value-in-use (‘VIU)
Depreciation amount
(cost of PV of the future
– Accumulated - subsequent
disposal) cash flows
Impairment Accumulated
Price received expected to be
Loss) Depreciation
to sell an asset derived from an
- subsequent
or CGU asset or CGU
Accumulated
Impairment Loss)

129
ACCA Financial Reporting (FR)
CHAPTER 6: IAS 36 IMPAIRMENT OF ASSETS

6.2 Identifying an asset that may be impaired

Learning Outcome (ACCA Study Guide Area B)


B3c : Identify the circumstances that may indicate the impairment to assets
At each reporting date an entity shall assess whether there is any indication that an asset may be
impaired
Basic rule: If there is indication that an asset may be impaired, the RA of the asset shall be estimated

Types of Indicator

External sources of information Internal sources of Information

• During the period, an asset's market value has • Evidence is available of obsolescence or
declined significantly more than would be physical damage of an asset
expected as a result of the passage of time or
normal use • Significant changes with an adverse effect on
the entity have taken place during the period,
• Significant changes with an adverse effect on or are expected to take place in the near future
the entity have taken place during the period, (ie: the asset becoming idle, plans to
or will take place in the near future, in the discontinue or restructure the operation to
technological, market, economic or legal which an asset belongs, plans to dispose of an
environment in which the entity operates or in asset before the previously expected date, and
the market to which an asset is dedicated reassessing the useful life of an asset as finite
rather than indefinite
• Market interest rates or other market rates of
return on investments have increased during • Evidence is available from internal reporting
the period, and those increases are likely to that indicates that the economic performance
affect the discount rate used in calculating an of an asset is, or will be, worse than expected
asset's value in use and decrease the asset's
recoverable amount materially

• Carrying amount of the net assets of the entity


is more than its market capitalisation

Exception: Irrespective of whether there is any indication, impairment test must be conducted annually
for:

i. Intangible asset with an indefinite useful life (IAS 38)


ii. Intangible asset not yet available for use
iii. Goodwill acquired in a business combination (IFRS 3)

130
ACCA Financial Reporting (FR)
CHAPTER 6: IAS 36 IMPAIRMENT OF ASSETS

6.3 Measuring recoverable amount

Basic rule: higher of an asset's or Cash Generating Unit's Fair Value - costs to sell and or its value in use
Exception:

• Not necessary to determine both Fair Value - costs to sell and value in use
If either one > CA (means no impairment), then not need to estimate the other
• If not possible to determine FV - costs to sell (ie: no basis for making a reliable estimate of the
price at which an orderly transaction to sell the asset would take place between market
participants)
RA = value in use
• If no reason to believe that value in use materially > FV - costs to sell
RA = FV - costs to sell
often be the case for an asset classify as held for sale (IFRS 5) as the future Cash Flow from
continuing use of the asset until its disposal are likely to be negligible

6.3.1 Fair Value – Cost to sell


15.1.33
Fair value (FV)

The price that would be received to sell an asset or paid to transfer a liability in an orderly transaction
between market participants at the measurement date (ie an exit price). (refer to IFRS 13 Fair Value
measurement)

Costs of Sell

Example of such costs are legal costs, stamp duty and similar transaction taxes, costs of removing the
asset and direct incremental costs to bring an asset into sale condition.

However, termination benefits like redundancy costs (as defined in IAS 19) and reorganisation cos ts
following the disposal of an asset are not direct incremental costs to dispose of the asset.

Note: FV - costs to sell does not reflect a 'forced sale' value

131
ACCA Financial Reporting (FR)
CHAPTER 6: IAS 36 IMPAIRMENT OF ASSETS

Exercise 6.3.1

ABC Co wishes to estimate the FV - costs to sell of a group of assets. There is no active market for these
assets, so ABC Co engages the valuation services of the commercial agents who will arrange the sale
transactions carried out each year in such group of assets. The agents provide the following estimates:

Sales price 800,000


Agent's commission 55,000
Legal cost 30,000
Cost of dismantling the asset so that they can be removed 35,000
Cost of creating access to the group of assets for removal 20,000
Termination cost for staff who would become redundant on the sale 100,000
Cost of retraining staff who would be transferred to new activities 34,000

What is the FV less cost to sell of this group of assets?

Solution: $660,000

6.3.2 Value in use (VIU)


15.1.34
In order to calculate the VIU of an asset, an entity has to estimate the future cash flows that it expects to
derive from the asset, keeping in mind the time value of money and the possible variations in the amount
or timing of those future cash flows. This means the entity would need to estimate future cash flows and
then discount them in order to obtain their value at today’s date.

The time value of money, represented by the pre-tax current market rate of interest

VIU – PV of future cash flows expected to derive from an asset or cash generating unit

Estimated future cash flows includes:

- Projection of cash inflow from continuing use of the asset.


- Projection of cash outflows that are necessarily incurred to generate cash flow from the continuing
use of the asset and can be directly attributed, or allocated on a reasonable and consistent basis, to
the asset
- Net cash flows, if any, to be received or paid for the disposal of the asset at the end of its useful life.
- CF projections made based on most recent financial budgets/forecasts approved by management for
a maximum 5 years, unless a longer period can be justified.
- CF projections beyond 5 years should be based on a steady/declining growth rate.

132
ACCA Financial Reporting (FR)
CHAPTER 6: IAS 36 IMPAIRMENT OF ASSETS

However, the cash flow projections exclude any estimated cash inflows or outflows expected to arise
from:

- Future restructurings to which the entity is not yet committed


- Improving or enhancing the asset’s performance.

Exercise 6.3.2

Years Net future CF DF @ 10% PV of future CF


(N1) (N2) (N5)

1 100 0.9091 90.91


2 150 0.8264 123.97
3 250 (N3) 0.7513 187.83
4 330 0.6830 225.39
5 420 0.6209 260.79

6 410 0.5645 231.43


7 390 (N4) 0.5132 200.13
8 350 0.4665 163.28
ERV (N2) 50 0.4665 23.33
Value in use
Notes:

N1 Relates to the RUL of the asset

N2 Future CF includes cash inflows and outflows to be derived from continuing use of the asset in its
current condition and from its ultimate disposal (at the end of UL)
Future CF shall exclude cash inflows/outflows arises from future restructuring or improving the
asset's performance, financing activity or income tax payments or receipts

N3 CF projections based on most recent fin budgets/forecasts approved by management (ie. normally
for a max of 5 yrs unless a longer period can be justified)

N4 For CF projected beyond 5 yrs, a steady / declining growth rate is to be used from yr 6 onwards to
extrapolate the projected CF of yr 5

N5 Based on pre-tax discount rate reflecting current mkt rates of interest

133
ACCA Financial Reporting (FR)
CHAPTER 6: IAS 36 IMPAIRMENT OF ASSETS

6.4 Measuring an impairment loss


Learning Outcome (ACCA Study Guide Area B)
B3a : Define, calculate and account for an impairment loss

In measuring impairment loss. The recoverable amount need to identify by comparing fair value and
value in use. Then, the carrying amount must be compared with recoverable amount.

Basic rule: impairment loss = Carrying Amount > Recoverable Amount

Example 6.4:

Co has an asset purchased on 1.1.01, End of 02, there is an evidence for impairment and
Co calculated:
Historical Cost $100 - Present Value of net future cash flow =$20 (value
Residual Value=$ 0 in use)
Deprecation=Straight Line Method - Market price of asset =$25
Estimated Useful Life 5 years - Related cost of disposal =$2

Required:
Calculate impairment loss & show extract of final accounts

Solution:
Carrying amount $ Recoverable amount
HC higher of
AD
CA @31.12.02 FV – cost to sell Value in use
$25-$2 = 23 $ 20
Impairment loss (charge to SOPL) = $60-$23 = $37

Extract of SOPL Year 01 Year 02 Year 03


Depreciation
Impairment loss

Extract of SOFP
NCA
HC
AD
AIL
NBV

Subsequent measurement after impairment loss:


- continue to charge depreciation after impairment loss (RA/RUL =$23/3yrs)
- reviewed for impairment by assessing at the end of each reporting period whether there is any
indication that it may be impaired.

134
ACCA Financial Reporting (FR)
CHAPTER 6: IAS 36 IMPAIRMENT OF ASSETS

6.5 Recognising an impairment loss

Learning Outcome (ACCA Study Guide Area B)


B3a : Define, calculate and account for an impairment loss
B3e : State the basis on which impairment losses should be allocated, and allocate an impairment loss
to the assets of a cash generating unit

Exercise 6.5

An entity owns a property which was originally purchased for $300,000. The property has been revalued
to $500,000 previously. At 31 Dec 2011, the property has a current carrying value of $460,000 but the
recoverable amount of the property has just been estimated at only $200,000.

Solutions:

Impairment loss:

Dr OCI & RS
Dr Imp loss (SOPL)
Cr AIL/NBV

135
ACCA Financial Reporting (FR)
CHAPTER 6: IAS 36 IMPAIRMENT OF ASSETS

Extract of P/L & OCI stmt $


Other exp:
impairment loss

OCI:
Loss on Revaluation

Extract of SOFP $
NCA
Revaluation amount
Subs AD
Subs IL
NBV

6.6 Cash-generating units

Learning Outcome (ACCA Study Guide Area B)


B3d : Describe what is meant by a cash generating unit

A cash generating unit (CGU) is the smallest identifiable group of assets that generates cash inflows that
are largely independent of the cash inflows from other assets or groups of assets.
• Various factors determine whether cash inflows from an asset / group of assets are largely
independent of the cash inflows from other asset / groups of assets, for eg. How management
monitors the entity’s operations (such as by product lines, businesses, individual locations,
districts or regional areas)
• How management makes decisions about continuing or disposing of the entity’s assets and
operations.

136
ACCA Financial Reporting (FR)
CHAPTER 6: IAS 36 IMPAIRMENT OF ASSETS

Example 6.6 (a)

A mining entity owns a private railway to support its mining activities. The private railway could be sold
only for scrap value and it does not generate cash inflows that are largely independent of the cash inflows
from the other assets of the mine.

Solution:

It is not possible to estimate the recoverable amount of the private railway because its value in use cannot
be determined and is probably different from scrap value. Therefore, the entity estimates the recoverable
amount of the CGU to which the private railway belongs, ie the mine as a whole.

Example 6.6 (b)

A bus company provides services under contract with a municipality that requires minimum service on
each of five separate routes. Assets devoted to each route and the cash flows from each route can be
identified separately. One of the routes operates at a significant loss.

Solution:

Because the entity does not have the option to curtail any one bus route, the lowest level of identifiable
cash inflows that are largely independent of the cash inflows from other assets or groups of assets is the
cash inflows generated by the five routes together. The CGU for each route is the bus company as a whole.

6.6.1 Determine the CA & RA of a CGU


15.1.35
Learning Outcome (ACCA Study Guide Area B)
B3e : State the basis on which impairment losses should be allocated, and allocate an impairment loss
to the assets of a cash generating unit

It may be necessary to consider some recognise liability to determine the RA of a CGU. This may occur if
the disposal of a CGU would require the buyer to assume the liability.

FV - costs to sell Value in use Carrying amount


= Estimated Selling Price (including = Present Value of = Carrying amount
consideration of liability) future Cash Flow - liability
- costs of disposal - liability

137
ACCA Financial Reporting (FR)
CHAPTER 6: IAS 36 IMPAIRMENT OF ASSETS

Example 6.6.1(a)

A company operates a mine in a country where legislation requires that the owner must restore the site
on completion of its mining operations. The cost of restoration includes the replacement of the
overburden, which must be removed before mining operations commence. A provision for the costs to
replace the overburden was recognised as soon as the overburden was removed.

The amount provided was recognised as part of the cost of the mine and is being depreciated over the
mine's useful life. The carrying amount of the provision for restoration costs is $500, which is equal to
the present value of the restoration costs.

The entity is testing the mine for impairment. The cash-generating unit for the mine is the mine as a
whole. The entity has received various offers to buy the mine at a price of around $800. This price reflects
the fact that the buyer will assume the obligation to restore the overburden. Disposal costs for the mine
are negligible. The value in use of the mine is approximately $1,200, excluding restoration costs. The
carrying amount of the mine is $1,000.

The cash-generating unit's fair value less costs to sell is $800. This amount considers restoration costs
that have already been provided for. As a consequence, the value in use for the cash-generating unit is
determined after consideration of the restoration costs and is estimated to be $700 ($1,200 less $500).
The carrying amount of the cash-generating unit is $500, which is the carrying amount of the mine
($1,000) less the carrying amount of the provision for restoration costs ($500).

Therefore, the recoverable amount of the cash-generating unit exceeds its carrying amount, indicating no
impairment loss.

138
ACCA Financial Reporting (FR)
CHAPTER 6: IAS 36 IMPAIRMENT OF ASSETS

Example 6.6.1 (b)

CGU excluding GW

Company X

Co X conducted the following impairment test:


a. Individual asset I - no impairment
b. For CGU II - no impairment
c. For individual asset J - evidence - conduct impairment test on individual asset basis
CA = $500 RA = higher of
FV - costs to sell = $400
Value in use = $300

Therefore, impairment loss = ____________ Dr


Cr
d. For CGU I - evidence - all are dependent assets - conduct impairment test for CGU

CA of individual asset in CGU: RA of CGU:


A $400 Assume FV-costs to sell = N/A
B $300 Therefore, RA = Value in use of $800
C $200
D $100
$1,000

Therefore, impairment loss = ______________

Asset Before IL IL (pro-rata) After IL


A 400
B 300
C 200
D 100
1000

Rule 
Impairment loss should be allocated to assets on a pro rata basis

139
ACCA Financial Reporting (FR)
CHAPTER 6: IAS 36 IMPAIRMENT OF ASSETS

Example 6.6.1(c)

CGU including purchased GW

H purchased 100% shares in S on 1.1.01 at cost of $200,000. S has manufacturing plants in 3 different
cities with a CA (FV) at date of acquisition:

Plant A $80,000 City A made up of asset 1, 2 & 3


Plant B $50,000 City B
Plant C $40,000 City C

Each plant is identified as a CGU for conducting impairment test purpose.


Assume that the EUL of the plant is 10 yrs. At the end of 02, there is evidence that plant A is impaired,
the RA of plant A is $55,000.
Date of acquisition (1.1.01)
IFRS 3 requires that GW must be calculated in a biz combination:
$
Cost of investment 200,000
Less: FV of identifiable NA acquired x H% 170,000
GW on acquisition 30,000

Note 
No amortisation is allowed for GW, but must test for annual impairment regardless of evidence.

A B C Total
FV of NA 80,000 50,000 40,000 170,000
GW allocated 14,000 9,000 7,000 30,000
CA of CGU 94,000 59,000 47,000 200,000

For year 1, no impairment for GW - RA of CGU > CA of CGU

End of year 2, the CA of identifiable assets of plant A is $64,000. Assume it is made up of:
Asset 1 $20,000
Asset 2 $20,000
Asset 3 $24,000.

140
ACCA Financial Reporting (FR)
CHAPTER 6: IAS 36 IMPAIRMENT OF ASSETS

Solution:

Impairment test for CGU A $


CA as at 1.1.01 80,000
Depn (80/10 x 2)
CA as at 31.12.02 RA = $55,000
Add: GW allocated
CA of CGU

Therefore, impairment loss = $_____________

Assets Before IL IL (pro-rata) After IL


GW
1
2
3
CA of CGU

Rule 1 
The impairment loss shall be allocated to reduce CA of CGU in the following order:

i. to any assets that are obviously damaged or destroyed


ii. of any GW allocated to the CGU
iii. then to other assets on pro-rata basis

Rule 2 
DO NOT reduce the CA of an asset below the highest of

FV - costs to sell Value in use


Zero
(if measurable) (if determinable)

ie. For asset 3, the FV - costs to sell is $23,000, value in use is N/A.

Assets Before IL IL (pro-rata) After IL


GW
1
2
3
CA of CGU

Note:
The excess IL for asset 3 should be written off against other assets of CGU on a pro-rata basis.

141
ACCA Financial Reporting (FR)
CHAPTER 6: IAS 36 IMPAIRMENT OF ASSETS

6.7 Reversal of Impairment Loss

Learning Outcome (ACCA Study Guide Area B)


B3b : Account for the reversal of an impairment loss on an individual asset

IAS36 requires that an entity shall assess, in the case of an impaired asset other than goodw ill, at each
SOFP date whether there is an indication that the impairment loss has decreased or that it no longer exists
eg of indications are:
• The Market Value of the asset has increased substantially.
• The economic performance of the asset is, or will be, better than expected

If there is an indication, the entity shall estimate the RA of that asset.

If the Recoverable Amount is higher than the Carrying Value, the impairment loss may be reversed, subject
to the following condition and limit:

Condition:

An impairment loss recognized in prior periods for an asset other that goodwill shall be reversed if there
has been a change in the estimates used to determine the asset’s Recoverable Amount since the last
impairment loss was recognized. If this is the case, the Carrying Value of the asset shall be increased to its
Recoverable Amount (reversal of an impairment loss)

Limit:

The increased CV of an asset other than goodwill attributable to a reversal of an impairment loss shall not
exceed the CV that would have been determined net of amortization or depreciation had no impairment
loss been recognized for the asset in prior years.

6.7.1 Accounting for reversal of impairment loss


15.1.36
1. If the revaluation model is followed:

To the extend the earlier impairment loss was recognized in the SOPL, the present reversal of
impairment should also be recognized in the SOPL.

The balance of the reversal will be directly credited to the revaluation surplus.

2. If the revaluation model is not followed:

The reversal of impairment loss shall be immediately recognized in the SOPL/IS as an item of income.

After the impairment loss is reversed, the CV and hence the depreciable amount would change.
Therefore, the amount of depreciation is to be revised.

142
ACCA Financial Reporting (FR)
CHAPTER 6: IAS 36 IMPAIRMENT OF ASSETS

6.8 Check Understanding

Question 1

Entity B has a single substantial asset which is uses to manufacture a product. The CA of the asset after 4
years is $5m (cost $7m, accumulated depreciation on a SLM of $2m). There is no ERV. Due to a
breakthrough in technology, entity B now expects the machine to produce 30% less in revenue terms
than expected over the rest of its EUL of 10 yrs. Net future cash flows for the next 5 years, based on
management's best estimate after taking the 30% cut into account are:

Year 1 2 3 4 5
Future cash flows ($'000) 600 660 710 755 790

The expected growth rates for the following years (extrapolated)

Year 6 7 8 9 10
Future cash flows 2% -1% -7% -16% -30%

If the machine was sold now it would realize $3.2m, net of selling costs. The discount rate to be applied
to the future cash flows is 10%

Calculation Value in Use ($'000)

Year Long term Future cash PV factor at Discounted


growth rate flows ($'000) 10% future cash
($'000)
1 600 0.909 545
2 660 0.826 545
3 710 0.751 533
4 755 0.683 516
5 790 0.621 491
6 2% 806 0.564 454
7 -1% 798 0.513 409
8 -7% 742 0.466 346
9 -16% 623 0.424 264
10 -30% 436 0.385 168
NPV = 4272 (Value in use)

143
ACCA Financial Reporting (FR)
CHAPTER 6: IAS 36 IMPAIRMENT OF ASSETS

Solution:
Based on IAS 36, if there is evidence for impairment test (breakthrough of technology reduction 30% in
revenue), company must conduct impairment test.
End of year 4 - Compare CA with
RA

Higher of

Value in Use = FV less Cost to Sell =

For year 4, company will charge an impairment loss of ______________________

Dr Imp loss (SOPL) Subq meas: NCA in SOFP at the end of year 4 at its RA $4272k
Cr NBV ▪ Charge depn based on IAS 16 [RA $4272k / RUL 10 years]
▪ Reviewed for imp by assessing at the end of each RP
whether there is any indication that it may be impaired

144
ACCA Financial Reporting (FR)
CHAPTER 6: IAS 36 IMPAIRMENT OF ASSETS

Question 2

NCA carried at Revaluation amount

A NCA costs $3m and has a EUL of 10 years with nil ERV. It is depreciated on a SLM and revalued
annually. At the end of year 1 and 2 the asset has a market value of $3.6 m and $2.1m respectively.
(assuming RA same as the mkt value)

Show the accounting treatment assuming:


(a) Company carried the asset at HC - AD [IAS 16 cost model]
(b) Company revalued the asset annually [IAS 16 revaluation model]. Company has a policy to transfer
excess depreciation to RE.

Solution (a): $000


Beginning of year 1 HC
Depn (3000/10 years) IAS 16
Closing NBV
Imp Loss IAS 36
RA

Beginning of year 2 Op NBV


Depn IAS 16
Closing NBV
Dr P/L Imp Loss IAS 36
Cr NBV RA

Solution (b): $'000


Beginning of year 1 HC
Depn (3000/10 years) IAS 16
Closing NBV
Dr NCA Revl surplus
Cr OCI & RS Revl amt

Beginning of year 2 Op NBV


Depn IAS 16 (Revl amt/RUL)
Closing NBV
Dr OCI & RS Revl deficit IAS 36
Dr SOPL RA
Cr NBV

SOCIE for the end of Yr 2 RS ($'000)


Op bal
EDA
Bal available in RS
Imp loss/ Revl loss
Cl bal

145
ACCA Financial Reporting (FR)
CHAPTER 6: IAS 36 IMPAIRMENT OF ASSETS

Question 3

On 1 January 20X0 Multiplex acquired Steamdays, a company that operates a scenic railway along the
coast of a popular tourist area. The summarised SOFP of Steamdays on 1 January 20X0, reflecting the
terms of the acquisition was:
$'000
Goodwill 200
Operating licence 1,200
Property - train stations and land 300
Rail track and coaches 300
Two steam engines 1,000
Purchase consideration 3,000

The operating licence is for ten years. It was renewed on 1 January 20X0 by the transport authority and
is stated at the cost of its renewal. The engines are already valued at their fair values less costs to sell
(net realisable values).

On 1 February 20X0 the boiler of one of the steam engines exploded, completely destroying the whole
engine. Fortunately, no one was injured, but the engine was beyond repair. Due to its age a replacement
could not be obtained. Because of the reduced passenger capacity, the estimated recoverable amount
of the whole of the business after the accident was assessed at $2 million.

Passenger numbers after the accident were below expectations even allowing for the reduced capacity.
A market research report concluded that tourists were not using the railway because of their fear of a
similar accident occurring to the remaining engine. In the light of this the recoverable amount of the
business was re-assessed on 31 March 20X0 at $1.8 million. On this date Multiplex received an offer of
$900,000 in respect of the operating licence (it is transferable).

Calculate the carrying value of the assets of Steamdays (in Multiplex's consolidated SOFP) at 1
February 20X0 and 31 March 20X0 after recognising the impairment losses.

Solution:

CA Feb 20X0 Mar 20X0


before Imp loss CA after Imp loss CA after
Imp loss Imp loss Imp loss
Goodwill
Operating licence
Property - train stations and land
Rail track and coaches
Two steam engines
Total

146
ACCA Financial Reporting (FR)
CHAPTER 6: IAS 36 IMPAIRMENT OF ASSETS

Question 4

a) The objective of IAS 36 Impairment of assets is to prescribe the procedures that an entity applies to
ensure that its assets are not impaired.

Required:
Explain what is meant by an impairment review and how frequently assets should be tested for
impairment. You answer should include reference to assets that may form a cash generating unit.
Note: you are NOT required to describe the indicators of an impairment or how impairment losses
are allocated against assets.

Solution:
Impairment loss

An impairment occurs when the carrying value of an asset exceeds its recoverable amount.
Recoverable amount represents the higher of the amount of cash that an asset will generate either
through use (value in use) or through disposal (fair value less costs to sell).

Value in use

The value in use is the present value of all future cash flows derived from an asset, including any
disposal proceeds at the end of the asset's life. The present value of future cash flows will be
affected by the timing, volatility and uncertainty of the cash flows. This can be reflected in the
forecasted cash flows or the discount rate used.

Cash generating units

Very few business assets generate their own cash flows, and so assets are often grouped together
into cash generating units for impairment purposes. A cash generating is the smallest group of
identifiable assets generating independent cash flows from other assets.

Fair value - costs to sell

Fair value less costs to sell is the price that would be received to sell an asset in an orderly
transaction between market participants, less the cost of disposal. The fair value of used assets with
no active market will have to be estimated. Valuations are based on willing parties, and so a 'forced
sale' value would not normally be used.

Impairment reviews

At each reporting date an entity shall assess whether there are any indications that an impairment
has occurred; if there are such indications then the recoverable amount of the asset must be
estimated.

147
ACCA Financial Reporting (FR)
CHAPTER 6: IAS 36 IMPAIRMENT OF ASSETS

b) Explain how an impairment loss is accounted for after loss has been calculated

Solution:
Impairment losses should be recognised immediately to profit or loss alongside depreciation, but the
impairment of a revalued asset should be taken directly to the revaluation surplus (until the balance
on the revaluation surplus is reduced to zero) and any excess only charged to profit or loss. In the
SOFP, the carrying value of the asset will be stated at cost - accumulated depreciation - accumulated
impairment loss. Future depreciation charges will be based on the impaired value and the remaining
useful life at the date of the impairment.

Impairments of CGU must be apportioned to the individual assets within that unit. The impairment is
firstly allocated to goodwill, and then it is apportioned to all other assets (both tangible and intangible)
on a pro rata basis (rule 1). However, individual assets are not impaired below their own realisable
value; any unused impairment being re-apportioned to the other assets (rule 2)

c) Tenji acquired an item of plant at a cost of $900,000 on 1 April 2010. The plant had an estimated
residual value of $50,000 and estimated life of five years, neither of which has changed. Tenji uses
straight-line depreciation. On 31 March 2012, Tenji had been having difficulty of selling products
produced by this plant. It now estimated that net cash inflows earned from the plant for the next
three years will be:

$’000
Year ended: 31 March 2013 320
31 March 2014 190
31 March 2015 150

On 31 March 2015, the plant is still expected to be sold for its estimated realisable value. Tenji has
confirmed that there is no market in which to sell the plants at 31 March 2012. Tenji’s cost of capital
is 10% and the following values should be used:

Values of $1 at: $
End of year 1 0.91
End of year 2 0.83
End of year 3 0.75

Required:
Calculate the carrying amounts of the assets above at 31 March 2012 after applying any impairment
losses

148
ACCA Financial Reporting (FR)
CHAPTER 6: IAS 36 IMPAIRMENT OF ASSETS

Solution:

Carrying amount =
Recoverable amount =
Hence, no impairment loss. Asset will be carrying at _______

d) Telepath owned a 100% subsidiary, Tilda, that is treated as a cash generating unit. On 31 March 2012,
there was an industrial accident (a gas explosion) that caused damage to some of Tilda’s plant. The
assets of Tilda immediately before the accident were:

$’000
Goodwill 1,800
Patents 1,200
Factory building 4,000
Plant 3,500
Receivables and cash 1,500
12,000

As a result of the accident, the recoverable amount of Tilda is $6.7 million.


The explosion destroyed (to the point of no further use) an item of plant that had a carrying amount
of $500,000.
Tilde has an open offer from a competitor of $1 million for its patent. The receivables and cash are
already stated at their fair values less costs to sell (net realisable values).

Calculate the carrying amount of Tilda’s asset after taking into account the impairment loss.

Solution:

CA before Imp loss Imp loss CA after Imp loss

Goodwill
Patent
Factory building
Plant
Receivables and cash
Total

149
ACCA Financial Reporting (FR)
CHAPTER 6: IAS 36 IMPAIRMENT OF ASSETS

6.9 Chapter 6 Summary

Diagram 6.9.: Summary of IAS 36 Impairment of Assets

150
ACCA Financial Reporting (FR)
CHAPTER 7: IAS 20 GOVERNMENT GRANT & DISCLOSURE OF GOVERNMENT ASSISTANCE

CHAPTER 7: IAS 20 GOVERNMENT


GRANT & DISCLOSURE OF
GOVERNMENT ASSISTANCE
LEARNING OUTCOME
At the end of the chapter, you should be able to:
TLO B11a. Apply the provisions of relevant accounting standards in relation to accounting for
government grants

151
ACCA Financial Reporting (FR)
CHAPTER 7: IAS 20 GOVERNMENT GRANT & DISCLOSURE OF GOVERNMENT ASSISTANCE

7.1 Overview

Government grant is a form of government assistance provided to entities that meets the criteria and
conditions attached to the grant.

In the context of financial reporting, it is important to disclose adequate information in relation to


government assistance, to ensure that an entity’s performance is accurately interpreted. The
identification of government assistance allows a fair comparison to be made with other entities in a similar
industry that have not received such assistance.

The objective of IAS 20 is to prescribe the accounting for, and disclosure of, government grants and other
forms of government assistance. It is common for entities to receive government assistance for various
purposes.

Government assistance does not include benefits provided indirectly to an entity, for example the
provision of infrastructure in development areas.

7.1.0 Relational Mapping

Diagram 7.1 Relational Mapping (IAS 20 in relation to other standards in context of Government Grant)

152
ACCA Financial Reporting (FR)
CHAPTER 7: IAS 20 GOVERNMENT GRANT & DISCLOSURE OF GOVERNMENT ASSISTANCE

7.1.1 Definition
15.1.37
Definitions of Government assistance and Government Grants:

(i) Government Assistance

Government assistance is direct action to provide economic benefits to a qualifying firm, or groups of
firms. It does not include indirect help, such as improving local infrastructure, or imposing import
tariffs, free technical or marketing advice, the provision of guarantees etc.

(a) Aimed at encouragement/long term support of business activities either in certain


regions/industry sectors

(b) Provides economic benefit specific to an entity by subsidizing entities that will provide jobs,
services or goods that might not otherwise be either available or available at a desired cost

(c) Does not include indirect benefits such as provision of infrastructure in development areas or
imposition of trading constraints on competitors

(d) To encourage entity to embark on a course of action which it would not normally have taken if
the assistance was not provided

(ii) Government Grants

Government grants are assistance by government in the form of transfers of resources to an entity in
return for past or future compliance with certain conditions relating to the operating activities of the
entity.

Some forms of government assistance are excluded from the definition of government grants like:

(a) Government assistance that cannot reasonably have a value placed on them.
Example: Free Technical or marketing advice, provision of guarantee

(b) Transactions with government which cannot be distinguished from the entity's normal trading
transactions.

Example: If government procurement policy results in a portion of the entity's sales, any
segregation would be arbitrary.

153
ACCA Financial Reporting (FR)
CHAPTER 7: IAS 20 GOVERNMENT GRANT & DISCLOSURE OF GOVERNMENT ASSISTANCE

7.1.2 Scope of IAS 20


15.1.38
Learning Outcome (ACCA Study Guide Area B)
B11a : Apply the provisions of relevant accounting standards in relation to accounting for government
grants
This Standard shall be applied in accounting for, and in the disclosure of, government grants and in the
disclosure of other forms of government assistance.
IAS 20 does not apply to:
• Government assistance given in the form of ‘tax break’, such as accelerated depreciation and
reduced rate of tax.
• Government acting as part owner of the entity.

7.2 Purpose of government assistance

Government is responsible for social and economic policy. One of the means it has at its disposal to
influence society, or the economy, is by providing financial incentives that will move society, or an
economic sector, in the desired direction.

Common desires of government include:

Economic Objective Incentive


Reduce Unemployment Subsidise new jobs and training

Grants for new factories / job creation

Increase Tourism Offer airlines a period of free of landing fees, subsidise hotel
building

Improve health Grants towards removal of hazardous materials, such as


asbestos

7.3 Accounting treatment of Government Grants

Government grants, including non-monetary grants (e.g. PPE, license, etc.) at fair value, shall not be
recognised until there is reasonable assurance that:

(a) The entity will comply with the conditions attaching to the grant; and
(b) The entity will actually receive the grant.

Government grants shall be recognised as income over the periods necessary to match them with the
related costs which they are intended to compensate, on a systematic basis. They shall NOT be credited
directly to shareholder’s interests.

154
ACCA Financial Reporting (FR)
CHAPTER 7: IAS 20 GOVERNMENT GRANT & DISCLOSURE OF GOVERNMENT ASSISTANCE

Diagram 7.3 Accounting Treatment for Government Grants

7.3.1 Non-monetary Government Grants


15.1.39

A government grant may take the form of a transfer of a non-monetary asset, such as land or other
resources, for the use of the entity. In these circumstances it is usual to assess the fair value of the non -
monetary asset and to account for both grant and asset at that fair value. An alternative course that is
sometimes followed is to record both asset and grant at a nominal amount.

7.3.2 Government Grants Related to Assets


15.1.40
Grants related to assets are used to acquire or construct specific long term assets.

Government grants related to assets, including non-monetary grants at fair value should be presented in
the statement of financial position either:

• By setting up the grant as deferred income, or


• By deducting the grant in arriving at the carrying amount of the asset (that is netting off).

Deferred income method Netting-off method

The deferred income method sets up the grant as The netting-off method deducts the grant in
deferred income in the statement of financial arriving at the carrying amount of the asset to
position, which is recognised in profit or loss on a which it relates.
systematic and rational basis over the useful life
of the asset. The grant is recognised in profit or loss over the
life of a depreciable asset by way of a reduced
Normally this corresponds to the method of depreciation charge.
depreciation on the related asset.

155
ACCA Financial Reporting (FR)
CHAPTER 7: IAS 20 GOVERNMENT GRANT & DISCLOSURE OF GOVERNMENT ASSISTANCE

Exercise 7.3.2 [CBG]


Rose Co receives a 20% grant towards the cost of a new PPE, which cost $100,000. The PPE has an EUL
of 4 years and ERV is nil. The expected profit before depreciation or grant of Rose Co amount to $50,000
p.a.

Solution:
Method 1 (Treat as deferred income)

Extract of Statement of Profit or Loss ($’000) Year 1 Year 2 Year 3 Year 4


Profit before depreciation
Depreciation
Grant
Profit after depreciation

Extract of Statement of Financial Position ($’000)


Non-current asset
Property, plant and equipment - cost
Accumulated depreciation
Net book value

Non-current liability : Deferred income


Current Liability : Deferred income

Method 2 (Reducing the cost of PPE)

Extract of Statement of Profit or Loss ($’000) Year 1 Year 2 Year 3 Year 4


Profit before depreciation
Depreciation
Profit
Extract of Statement of Financial Position ($’000)
Non-current asset
Property, plant and equipment – cost
(net of government grant)
Accumulated depreciation
Carrying value

NOTE
On balance both the methods give the same net profit but Method 1 gives a clear indication of the cost
of the non-current asset and reports the grant income and depreciation expenses separately so that the
users can have better information on the grants received by the company.

156
ACCA Financial Reporting (FR)
CHAPTER 7: IAS 20 GOVERNMENT GRANT & DISCLOSURE OF GOVERNMENT ASSISTANCE

7.3.3 Presentation of grants related to income


15.1.41
Government grants related to income are defined as those not related to assets and can be presented in
two ways:
• A credit in profit or loss (either separately, or under a general heading such as 'other income') or
• A deduction from the related expense.

Note: Treating the grant as a deduction from the related expense (the net treatment) results in the
statement of profit or loss being less comparable with those of similar entities that have not received
such grants.

This treatment may also lead to the particular category of expenditure being excessively low in one year,
or in comparison with other categories of expenditure during that period.

Disclosure of grants received will therefore be important to assist comparison and understanding.

Example 7.3.3 – Revenue Grant


Hermos Co received a grant of $15m to compensate it for costs it incurred in planting trees over a
period of 5 years. Hermos Co will incur such costs in this manner:
Year Cost
($'m)
1 2
2 4
3 6
4 8
5 10
30

Required:

How will the grant be treated in the books of Hermos Co in accordance with IAS 20?

157
ACCA Financial Reporting (FR)
CHAPTER 7: IAS 20 GOVERNMENT GRANT & DISCLOSURE OF GOVERNMENT ASSISTANCE

Solution:
Based on IAS 20, Government grants shall be recognised as income over the periods necessary to
match with the related costs for which the grants are intended to compensate, on a systematic basis.

(i) Present as separate income Year 1 Year 2 Year 3 Year 4 Year 5


Extract of Statement of Profit or Loss for the year ended ($'m)
Expense: Planting trees cost
Other Income: Government grant

(ii) Present as deduction from related expenses


Extract of Statement of Profit or Loss for the year ended ($'m)
Expenses: Planting trees cost (net)

Extract of SOFP for the year ended ($'m)


Non-current liability: Deferred
income (government grant)
Current liability: Deferred income
(government grant)

158
ACCA Financial Reporting (FR)
CHAPTER 7: IAS 20 GOVERNMENT GRANT & DISCLOSURE OF GOVERNMENT ASSISTANCE

7.4 Repayment of Government Grants

A government grant that becomes repayable shall be accounted for as a revision to an accounting
estimate (IAS 8 Accounting Policies, Changes in Accounting Estimates and Errors)

(a) Repayment of a grant related to income shall be applied first against any unamortized deferred credit
set up in respect of the grant. To the extent that the repayment exceeds any such deferred credit, or
when no deferred credit exists, the repayment shall be recognised immediately as an expense and
charged to P/L.
(b) Repayment of a grant related to an asset shall be recorded by increasing the carrying amount of the
asset or reducing the deferred income balance by the amount repayable. The cumulative additional
depreciation that would have been recognised to date as an expense in the absence of the grant shall
be recognised immediately as an expense.

Exercise 7.4.1(a) (Repayment of RBG)


ABC Co have received grants of $60,000 for staff recruitment. The Co spread the grant over 2 years. At
the beg of year 2, a government audit reveals that some of the conditions have not been met, and the
company must repay $40,000 of the grant.

Extract of SOPL for the Y/E ($) Year 1 Year 2


Exp: Repayment of GG
Other income: GG

Extract of SOFP as at ($) Year 1 Year 2


CL: deferred income (GG)

Exercise 7.4.1(b) (Repayment of CBG):


At beginning of year 3, Co. A defaults one condition. The govt demands immediate repayment of the
grant in full.

Method I: As deferred income Method II: Reducing cost of NCA


Extract of SOPL Year 1 Year 2 Year 3 Extract of SOPL Year 1 Year 2 Year 3
Profit be4 depn 50000 50000 Profit be4 depn 50000 50000
Depn (25000) (25000) Depn (20000) (20000)
Grant 5000 5000 Profit 30000 30000
Profit 30000 30000

Extract of SOFP Year 1 Year 2 Year 3 Extract of SOFP Year 1 Year 2 Year 3
NCA NCA
P&M - Cost 100000 100000 P&M - Cost 80000 80000
- AD (25000) (50000) - AD (20000) (40000)
NBV 75000 50000 NBV 60000 40000

NCL: def inc 10000 5000


CL: def inc 5000 5000

159
ACCA Financial Reporting (FR)
CHAPTER 7: IAS 20 GOVERNMENT GRANT & DISCLOSURE OF GOVERNMENT ASSISTANCE

Example 7.4 – Grant Recognition

Determine if the following grants should be recognised and, if so, the period over which they should be
recognised in profit or loss:

(1) A cash grant is available to private child nurseries to spend on toys in urban regeneration areas
that qualify for such support. The only condition attaching to the grant is that the money should
be spent Immediately.

(2) A manufacturing entity receives a grant of $1 million when it creates 50 jobs. $0.5 million is payable
when the figure is reached with the remaining $0.5 million payable at the end of four years should
the 50 jobs still be in existence. There is reasonable assurance that the employment levels will be
maintained when reached.

(3) Free testing equipment is available to new motor emission businesses being set up in a region that
qualifies for special government support.

(4) A government department offers a grant in the form of free technical advice to entities setting-up
businesses overseas to help export growth.

Solution:
(1) Nursery toys

This is a cash grant, so it ranks as a government grant which should be recognised. As it is given as
immediate financial support to the entity, it should be recognised in profit or loss immediately.

(2) Job creation

This is a government grant which should be recognised. $0.25 million should be recognised in profit or
loss for each year from the date the grant becomes receivable (when 50 jobs have been created). This
matches the grant with the related costs.
Because this does not match the cash receipts, deferred income and a receivable will appear in Years 1
and 3 respectively.

(3) Testing equipment

This is a grant of a non-monetary asset. The usual treatment would be to account for both grant and the
asset at the fair value of the equipment and recognise the grant in profit or loss over the period the asset
is depreciated.

(4) Technical advice

Free technical advice is likely to be a grant which cannot reasonably have a value placed upon it. As a
result, it should not be recognised.

160
ACCA Financial Reporting (FR)
CHAPTER 7: IAS 20 GOVERNMENT GRANT & DISCLOSURE OF GOVERNMENT ASSISTANCE

7.5 Disclosure
The following matters shall be disclosed:

(a) The accounting policy adopted for government grants, including the methods of presentation adopted
in the financial statements

(b) The nature and extent of government grants recognised in the financial statements and an indication
of other forms of government assistance from which the entity has directly benefited.

7.6 Check Understanding

Question 1 (adopted from June ’03)

Derringdo acquired an item of plant at gross cost of $800,000 on 1 October 20X2. The plant has an EUL
of 10 years with a ERV equal to 15% of its gross cost. Derringdo uses straight line depreciation on a time
apportioned basis. The Co received a government grant of 30% of its cost price at the time of purchase.
The terms of the grant are that is the Co retains the asset for 4 years or more, then no repayment
liability will be incurred. If the plant is sold within 4 years, a repayment on a sliding scale would be
applicable. The repayment is 75% if sold within the first year of purchase and this amount decreases by
25% p.a. Derringdo has no intention to sell the plant within the first 4 years.

Derringdo's AP for capital grant is to treat them as deferred credits and release them to income over the
life of the asset to which they relate.

Required

I. Discuss whether the company’s policy for the treatment of govt grants meets the definition of a
liability in the conceptual framework.

II. Prepare extracts of Derringdo's FS for the year to 31 March 20X3 in respect of the plant and the
related grant in:
▪ applying the company’s policy
▪ compliance with the definition of liability in the framework

161
ACCA Financial Reporting (FR)
CHAPTER 7: IAS 20 GOVERNMENT GRANT & DISCLOSURE OF GOVERNMENT ASSISTANCE

Solution:

Accounting issues here are:

(i) Should a capital grant be treated as deferred income (liability) in the FS, OR
(ii) Should a liability be recog for the potential repayment of GG

Based on the conceptual framework, liability is defined as a present obligation of the entity arising from
past events, the settlement of which is expected to result in an outflow of eco benefits from the entity.
A liability should only recog if there is probable outflow of eco benefits. Derringdo has credited
$240,000 ($800,000 x 30%) grant to a deferred income which is shown as a liability in the SOFP. The
grant will not have to be paid under normal circumstances (not outflow of eco benefits) and so a liability
does not exist.

This example is complicated by the possibility of having to repay the grant if the asset is sold within 4
years. At the SOFP's date, the asset has not been sold. Therefore, there is no past event that give rise to
a present obligation to pay.

Following from the above, the framework will not permit the grant to be shown as a liability. Instead the
grant would be claimed as income in the year that it was received (provided that there was no intention
to sell the asset within 4 years’ claw-back period. However, the treatment of the grants as deferred
income is in accordance with IAS 20.

Extract of SOPL for the Y/E 31 March 20X3 ($) IAS20 CF


Exp: depn
Other income: govt grant

Extract of SOFP as at 31 March 20X3


NCA: PPE

NCL: deferred income (govt grant)

CL: deferred income (govt grant)

162
ACCA Financial Reporting (FR)
CHAPTER 7: IAS 20 GOVERNMENT GRANT & DISCLOSURE OF GOVERNMENT ASSISTANCE

7.7 Chapter 7 Summary

Diagram 7.7.: Summary of IAS 20 Government Grant & Disclosure of Government Assistance

163
ACCA Financial Reporting (FR)
CHAPTER 8: IAS 23 BORROWING COST

CHAPTER 8: IAS 23 BORROWING COST


LEARNING OUTCOME
At the end of the chapter, you should be able to:

TLO B1a. Define and compute the initial measurement of a non-current asset (including
borrowing costs and an asset that has been self-constructed).

164
ACCA Financial Reporting (FR)
CHAPTER 8: IAS 23 BORROWING COST

8.1 Overview

Companies as part of their normal; activities construct or acquire different types of assets. If an entity
constructs a substantial asset either for use itself or for resale, it is likely that additional funds in the form
of loan capital will be required in order to finance the construction. The finance cost of these additional
funds are costs of the construction of the asset, similar to materials and labour that are costs of the
construction of the asset.

However, such finance costs incurred need to be:


• recognised as an expense in profit or loss, or
• recognised as part of the cost of the asset carried in the statement of financial position.

Whereby when borrowings that have been acquired specifically to finance the construction of a
substantial asset, a direct cost is incurred that would have been avoided had the construction not taken
place.

Therefore, it is reasonable that the financing cost associated with such borrowings should form part of
the cost of the asset, as are other costs incurred in the construction process. International standards now
require this treatment for all ‘directly attributable’ borrowing costs.

• Borrowing costs that are directly attributable to the acquisition, construction or production of a
qualifying asset form part of the cost of that asset.
• Other borrowing costs are recognised as an expense.

8.1.0 Relational Mapping

Diagram 8.1 Relational Mapping (IAS 23 in relation to other standards in context of Non-Current Assets)

165
ACCA Financial Reporting (FR)
CHAPTER 8: IAS 23 BORROWING COST

8.1.1 Scope
15.1.42
Learning Outcome (ACCA Study Guide Area B)
B1a : Define and compute the initial measurement of a non-current asset (including borrowing costs
and an asset that has been self-constructed).

IAS23 is used as a standard to prescribe the accounting treatment for borrowing costs:

Borrowing Cost

Borrowing costs are interest and other costs that an entity incurs in connection with the borrowing of
funds.
Examples include:
• Interest on bank overdrafts and short term and long term borrowings,
• Amortisation of discounts or premiums relating to borrowings (IFRS 9 FI – amortised cost method)
• Amortisation of ancillary costs incurred in connection with the arrangement of borrowings;
• Finance charge in respect of finance leases (IFRS 16 Leases)
• Exchange difference arising from foreign currency borrowings to the extent that they are regarded
as an adjustment to interest costs (IAS 21 The Effects of Changes in Foreign Exchange Rates)
• Dividends on preference shares (IAS 32 Financial Instruments: Presentation - if treated as a liability
in the Statement of financial position)

Qualifying Asset

A qualifying asset is an asset that necessarily takes a substantial period of time to get ready for its
intended use or sale. Depending on the circumstances, any of the following may be qualifying assets:

(i) Inventories (eg construction of an aeroplane for sale).


(ii) Manufacturing plants.
(iii) Power generation facilities.
(iv) Intangible assets.
(v) Investment properties.

166
ACCA Financial Reporting (FR)
CHAPTER 8: IAS 23 BORROWING COST

Financial assets, and inventories that are manufactured, or otherwise produced, over a short period of
time, are not qualifying assets. Assets that are ready for their intended use or sale when acquired are not
qualifying assets.

8.1.2 What is excluded in IAS 23


15.1.43

(a) The Standard does not deal with the actual or imputed cost of equity, including preferred capital not
classified as a liability.
(b) An entity is not required to apply the Standard to borrowing costs directly attributable to the
acquisition, construction or production of:
(i) A qualifying asset measured at fair value, for example a biological asset, or
(ii) Inventories that are manufactured or otherwise produced, in large quantities on a repetitive
behaviour.

8.2 Recognition

Learning Outcome (ACCA Study Guide Area B)


B1a : Define and compute the initial measurement of a non-current asset (including borrowing costs
and an asset that has been self-constructed).

The objective of IAS 23 is to prescribe the accounting treatment for borrowing costs. IAS 23 requires the
capitalization of borrowing costs that are directly attributable to the acquisition, construction or
production of a qualifying asset when:
• It is probable that costs will results in future economic benefits and the costs can be reliably
measured.
• The costs are directly attributable and they would have been avoided if the asset was not bought,
constructed or produced.

Borrowing Costs may include:

(i) interest expense calculated using the effective interest method as described in IFRS 9 Financial
Instruments: Recognition and Measurement;
(ii) Finance charges in respect of long term leases recognized in accordance with IFRS 16 Leases; and
(iii) Exchange differences arising from foreign currency borrowings to the extent that they are
regarded as an adjustment to interest costs.

167
ACCA Financial Reporting (FR)
CHAPTER 8: IAS 23 BORROWING COST

8.3 Measurement

Learning Outcome (ACCA Study Guide Area B)


B1a : Define and compute the initial measurement of a non-current asset (including borrowing costs
and an asset that has been self-constructed).

8.3.1 Specific borrowing cost


15.1.44
The borrowed funds used specifically for the acquisition, construction or production of a qualifying asset.
The amount of borrowing costs eligible for capitalisation will be:

Actual borrowing costs incurred on that borrowing during the period less any investment
income on the temporary investment of those borrowings pending their expenditure on the
qualifying asset.

Exercise 8.3.1 Specific Borrowing costs:

An entity borrowed $1m at 7.5% pa in order to finance the construction of a new building which
would take 12 months to complete. As stage payments were to be made in respect of the
construction costs, surplus funds were invested and during the 12 months’ period interest income
of $35,000 was earned.

Calculate the borrowing costs that is capitalise as part of the cost of the new building.

Solution:

168
ACCA Financial Reporting (FR)
CHAPTER 8: IAS 23 BORROWING COST

8.3.2 General borrowings


15.1.45
To the extent that an entity borrows funds generally and uses it for obtaining a qualifying asset, the entity
shall determine the amount of BC eligible for capitalisation by applying a capitalisation rate to the
expenditures on that asset.
BC incurred during the year
Weighted average of BC = Total borrowings outstanding X 100
during the period

The amount of BC capitalises during a period shall not exceed the amount of BC incurred during that
period.

Exercise 8.3.2 – Weighted average cost:


An entity has the following loan finance in place during the year:
$1 million of 6% pa loan finance
$2 million of 8% pa loan finance

It constructed a new factory which cost $600,000 and this was funded out of the existing loan
finance.

The factory took eight months to complete.

What borrowing costs should be capitalised?

Solution:

169
ACCA Financial Reporting (FR)
CHAPTER 8: IAS 23 BORROWING COST

8.4 Period of Capitalisation

Borrowing Cost should be capitalised only during the period of construction (from commencement to the
cessation) or production of the qualifying asset to get ready for its intended use or sale.

Borrowing Costs incurred before and after the construction period should be expensed to the
SOPL as incurred.

8.4.1 Commencement of Capitalisation


15.1.46

An entity shall begin capitalisation of borrowing costs as part of the cost of a qualifying asset on the
commencement date. The commencement date for capitalisation is the date when the entity first meets
all of the following conditions:

(i) It incurs expenditure for the asset;


(ii) It incurs borrowing costs; and

It undertakes activities that are necessary to prepare the asset for its intended use or sale

Example 8.4.1 - Commencement of capitalisation

The following events take place:

• An entity buys some land on 1 December.


• Work is undertaken in December and January in preparing a planning application.
• Planning permission is obtained on 31 January.
• Payment for the land is deferred until 1 February.
• The entity takes out a loan to cover the cost of the land and the construction of the building on 1
February.

When should capitalisation of borrowing costs commence?

Solution:

In this scenario the key dates are as follows:


• Expenditure on the acquisition is incurred on 1 December.
• Borrowing costs start to be incurred from 1 February.
• Activities to prepare the building for intended use/sale (work on planning permission) were carried
out during December and January.
The earliest date when all three of these conditions were met is 1 February.

170
ACCA Financial Reporting (FR)
CHAPTER 8: IAS 23 BORROWING COST

8.4.2 Suspension of Capitalisation


15.1.47

An entity shall suspend capitalisation of borrowing costs during extended periods in which it suspends
active development of a qualifying asset.

An entity does not suspend capitalising borrowing costs when a temporary delay is necessary part of the
process of getting an asset ready for its intended use or sale.

Example 8.4.2:

Capitalisation continues during the extended period that high water levels delay construction of a bridge,
if such high water levels are common during the construction period in the geographical region involved.

8.4.3 Cessation of Capitalisation


15.1.48

An entity shall cease capitalising borrowing costs when substantially all the activities necessary to prepare
the qualifying asset for its intended use or sale are complete.

An asset is normally ready for its intended use or sale when the physical construction of the asset is
complete even though routine administrative work might still continue. If minor modifications, such as
the decoration of a property to the purchaser’s or user’s specification, are all that are outstanding, this
indicates that substantially all the activities are complete.

When an entity completes the construction of a qualifying asset in parts and each part is capable of being
used while construction continues on other parts, the entity shall cease capitalising borrowing costs when
it completes substantially all the activities necessary to prepare that part for its intended use or sale.

Example 8.4.3 (a)


Capitalisation of borrowing cost can cease on the completion of the individual asset

A business park comprising several buildings, each of which can be used individually, is an example of a
qualifying asset for which each part is capable of being usable while construction continues on other parts.

Example 8.4.3 (b)


Capitalisation of borrowing cost can cease only on the completion of the whole asset

An example of a qualifying asset that needs to be completed before any part can be used is an industrial
plant involving several processes which are carried out in sequence at different parts of the plant within
the same site, such as a steel mill.

171
ACCA Financial Reporting (FR)
CHAPTER 8: IAS 23 BORROWING COST

8.5 Disclosure

An entity shall disclose:

(a) The amount of borrowing costs capitalised during the period; and
(b) The capitalisation rate used to determine the amount of borrowing costs eligible for capitalisation

172
ACCA Financial Reporting (FR)
CHAPTER 8: IAS 23 BORROWING COST

8.6 Check Understanding

Question 1

Amex issued a $10 million unsecured loan with a coupon (nominal) interest rate of 6% on 1 April 2009.
The loan has an effective finance cost of 7.5% per annum. The loan was specifically issued to finance the
building of the new store which meets the definition of a qualifying asset in IAS 23.

Construction of the store commences on 1 May 2009 and it was completed and ready for use on 28
February 2010, but did not open for trading until 1 April 2010.

During the year, trading at Amex’s other stores was below expectations so Amex suspended the
construction of the new store for a two-month period during July and August 2009. The proceeds of the
loan were temporarily invested for the month of April 2009 and earned interest of $40,000.

Required:

Calculate the net borrowing cost that should be capitalised as part of the cost of the new store and
the finance cost that should be reported in the SOPL for the year ended 31 March 2010.

Solution:

173
ACCA Financial Reporting (FR)
CHAPTER 8: IAS 23 BORROWING COST

8.7 Chapter 8 Summary

Diagram 8.7.: Summary of IAS 23 Borrowing cost

174
ACCA Financial Reporting (FR)
CHAPTER 9: IAS 40 INVESTMENT PROPERTY

CHAPTER 9: IAS 40 INVESTMENT


PROPERTY

LEARNING OUTCOME
At the end of the chapter, you should be able to:

TLO B1f. Discuss why the treatment of investment properties should be differ from other properties.

TLO B1g. Apply the requirements of relevant accounting standards to an investment property.

175
ACCA Financial Reporting (FR)
CHAPTER 9: IAS 40 INVESTMENT PROPERTY

Overview

An entity may own land or a building as an investment, rather than for use in the business. It may therefore
generate cash flows largely independently of other assets, which the entity holds.

Investment property (IP) is held to earn rentals or for capital appreciation or both. Therefore, an
investment property generates cash flows largely independently of the other assets held by an entity. This
distinguishes investment property from owner-occupied property

IAS 40 prescribe the accounting treatment for investment property and related disclosure requirements.

9.1 Relational Mapping

Diagram 9.1 Relational Mapping

9.2.1 Scope

Examples of items that are NOT investment properties.

Types of non-investment property Applicable IAS


Property held to sell in the ordinary course of business IAS 2 Inventories
rd
Property being constructed or developed on behalf of 3 parties IFRS 15 Revenue from contract with
customers
Owner-occupied property IAS 16 Property, plant and equipment
Property leased to another entity under a Finance Lease (Lessor IFRS 16 Leases
Books)

176
ACCA Financial Reporting (FR)
CHAPTER 9: IAS 40 INVESTMENT PROPERTY

Buildings whichare leased out to persons who are under contract to the company (property occupied
by employees is not an IP whether or not the employees pay rent at market rates) Note: If the
employees pay less than the market rate, the rental income cannot be grossed up to the market rate as
it would not be a fair representation of the financial performance.

9.2 Definition

Learning Outcome (ACCA Study Guide Area B)


B1f : Discuss why the treatment of investment properties should be differ from other properties.

9.2.1 Investment Property vs Owner-occupied


Property

9.2.2 Investment Property

Property (land or a building—or part of a building—or both) held (by the owner or by the lessee under a
lease) to earn rentals or for capital appreciation or both, rather than for:

(i) use in the production or supply of goods or services or for administrative purposes; or
(ii) sale in the ordinary course of business.

Example of Investment property:

• land held for long-term capital appreciation, not for short-term sale.
• land held for a currently undetermined future use.
• a building owned and leased out under short term leases.
• a building that is vacant but is held to be leased out under short term leases.
• Property that is being constructed or developed for future use as investment property.

177
ACCA Financial Reporting (FR)
CHAPTER 9: IAS 40 INVESTMENT PROPERTY

9.2.3 Owner-occupied Property

Owner –Occupied Property is property held (by the owner) for use in the production or supply of goods
or services or for administrative purposes.

9.3 Classification of Property

9.3.1 Partly owner occupied & partly leased/rented out

Diagram 9.3.1 Partly owner occupied and partly leased out

9.3.2 Property leased to Parent or Subsidiary

1. Individual book of Parent - follow IAS 40, can use fair value model
2. Consolidated FS - follow IAS 16, treat as owner occupied property, may use cost or revaluation
model

9.3.3 Fair value of Investment Property


The price that would be received to sell an asset or paid to transfer a liability in an orderly transaction
between market participants at the measurement date (ie an exit price). (refer to IFRS 13 Fair Value
measurement)

178
ACCA Financial Reporting (FR)
CHAPTER 9: IAS 40 INVESTMENT PROPERTY

9.4 Recognition Criteria

Investment property should be recognised as an asset when 2 conditions are met:

a. probable that inflow of future economic benefits


b. cost can be measured reliably

Application of recognition principle: Applied to initial cost and subsequent cost

9.5 Conceptual Framework

Learning Outcome (ACCA Study Guide Area B)


B1g : Apply the requirements of relevant accounting standards to an investment property.

(i) Definition of asset

Conceptual Framework 2010 Conceptual Framework 2018

Resources controlled by the entity as a result of A present economic resource controlled by


past events and from which future economic the entity as a result of past events.
benefits are expected to flow into the entity
An economic resource is a right that has the
potential to produce economic benefits

It is important to note that the definition of asset in this standard was not revised following the revision of
the definition of an asset in the Conceptual Framework for Financial Reporting issued in 2018.

(ii) Recognition criteria

An asset in IAS40 is recognized if

It is probable that the future economic benefits will flow to the entity and the asset has a
cost or value that can be measured reliably).

First, one should determine whether the item is an asset of the reporting entity (Conceptual Framework)
and then determine whether that asset is an item of IP (IAS 40).

An investment property is still classified as an asset in the SOFP and as such is inconsistent with the
Conceptual Framework definition of Asset being:
• resource controlled by the entity
• as a result of past event
• which will result in an inflow of economic benefits

179
ACCA Financial Reporting (FR)
CHAPTER 9: IAS 40 INVESTMENT PROPERTY

9.6 Measurement Criteria

9.6.1 Initial Measurement (at the date of purchase) = cost

An investment property should be measured initially at its cost.


The cost of investment property includes:
• Its purchase price and
• Any directly attributable expenditure, such as legal fees or professional fees, property taxes, and
other transaction cost etc.

Cost should NOT include:


• Start-up expenses whatsoever.
• However, if these start-up expenses are directly attributable to the item of investment property,
then it can include them. But do NOT include any general start-up expenses.
• Operating losses that is incurred before planned occupancy level is achieved, and
• Abnormal waste of material, labour or other resources incurred at construction.

When payment for investment property is deferred, then it must be discounted to its present value in
order to set the cash price equivalent.

Example 9.6.1:
On 30th June 20X4, an entity acquired for $3 million a site on which it planned to build a large office
building which it would then let out on a floor by floor basis. Construction commenced on 1st September
20X4 and was substantially completed on 30 th June 20X5 at a cost of $5 million at which point tenants
could move in.
Due to a misunderstanding the letting agents only commenced the marketing of the office space on 1 st
July 20X5 with the first leases being signed by tenants with effect from 1 st October 20X5. The building was
not fully let until July 20X6

Solution:
The property should be recognised as an investment property on 1st July20X5 when the offices were
ready to be occupied.
Costs incurred after this date should be recognised as an expense in profit or loss even though the entity
did not start to receive rentals until 1st October 20X5.
Any losses incurred during this ‘empty period’ and then up to the point the building is fully let are part of
the entity’s normal business operations and do not form part of the cost of the investment property.

180
ACCA Financial Reporting (FR)
CHAPTER 9: IAS 40 INVESTMENT PROPERTY

9.6.2 Subsequent measurement (at year end)

IAS 40 requires an entity to choose between 2 models.


(i) The fair value model
(ii) The cost model

FAIR VALUE MODEL COST MODEL

• All of its IP are measured at FV at the end of each • All of IP with IAS 16 requirement (cost less
reporting period provided fair value can be accumulated depreciation less accumulated
measured reliably. impairment)
• the changes in fair value (gain or loss) is shown • Even if this model is adopted the FV of IP should
directly in the profit or loss for the period in be disclosed in Financial Statements.
which they arise.
• no depreciation is charged on the asset.

Once chosen the FV / cost model, should apply it to all Investment property. Change of model is permitted
only if change resulted in more appropriate presentation:

From cost model to FV model Possible


From FV model to cost model Highly unlikely (not appropriate presentation)

181
ACCA Financial Reporting (FR)
CHAPTER 9: IAS 40 INVESTMENT PROPERTY

9.6.3 Fair Value model

Key terms
(1) After initial recognition, an entity that chooses the fair value model should measure all of its
investment property at fair value, except in the extremely rare cases N1, where this cannot
be measured reliably. In such cases it should apply the IAS 16 cost model must be applied
until the asset is disposed. The Residual Value is assumed to be zero
(2) The fair value of investment property should be measured following the FV Hierarchy in IFRS
13.

9.6.4 Difference between FV model and Revaluation model

The fair value model allowed for non-financial assets like IP and Biological asset is not the same as a
revaluation, where increases in carrying amount above a cost-based measure are recognised as
revaluation surplus.

Under the fair value model, all changes in fair value are recognised in the statement of profit or loss.

Exercise 9.6.4

Celine, a manufacturing company, purchases a property for $1 million on 1 January 20X1 for its investment
potential. The land element of the cost is believed to be $400,000, and the buildings element is expected
to have a useful life of 50 years. At 31 December 20X1, local property indices suggest that the fair va lue
of the property has risen to $1.1 million.

Show how the property would be presented in the financial statements as at 31 December 20X1 if Celine
adopts:

(a) the cost model


(b) the fair value model

Solution:

FV model Cost model


Extract of SOPL $'000 Extract of SOPL $'000
Invst prop surpluses Depn expenses

Extract of SOFP Extract of SOFP


NCA $'000 NCA $'000
Invst prop Invst prop

182
ACCA Financial Reporting (FR)
CHAPTER 9: IAS 40 INVESTMENT PROPERTY

9.7 Transfer

Transfers to, or from, investment property shall be made when, and only when, there is a change in use,
evidenced by:

For Transfer to/From Investment Property which carried at Fair Value:

183
ACCA Financial Reporting (FR)
CHAPTER 9: IAS 40 INVESTMENT PROPERTY

9.8 De-recognition

Derecognise (eliminate from the SOFP) an investment property


• on disposal or when it is permanently withdrawn from use and
• no future economic benefits are expected from its disposal.
• On Transfer

When disposed of / when permanently withdrawn from use / no further future economic benefits:

To recognised G/(L) on disposal in the profit or loss G/(L) = SP - NBV

9.8.1 Disposal of Investment Property

Company will need to calculate gain or loss on disposal (IAS 40) as a difference between:

• Net disposal proceeds, and


• Asset’s carrying amount.

Gain or loss on disposal is recognized in profit or loss.

Compensation from 3rd parties for investment property that was impaired, lost or given up shall be
recognised in profit or loss when the compensation becomes receivable.

9.9 Disclosures

IAS 40 Investment property prescribes a lot of disclosures to be presented in the financial statements,
including the description of selected model, how the fair value was derived, what the classification
criteria for investment property are, movements in investment property during the reporting period.

Expertise on comparability, accounting choices, and fair value is among the most controversial an d
poorly discussed themes in the current accounting scenario.

The IASB, when issuing The Conceptual Framework for Financial Reporting, pointed out there are
accounting choices in the standards so that the phenomenon they describe are faithfully represented

Comparability is among the qualitative characteristics of improving the accounting information utility that
allows users to identify and grasp the similarities between the items and the differences between them.
As a result, accounting information will be more useful if they can be compared through similar
information from other entities, or from the same entity, within another period (IASB Framework, 2011)

Accounting choices are needed in order to guarantee a reliable representation, at the same time their
discretionary use may decrease the comparability level expected by the International Accounting
Standards Board (IASB).

184
ACCA Financial Reporting (FR)
CHAPTER 9: IAS 40 INVESTMENT PROPERTY

9.10 Check Understanding

Question 1

Co. A owns a building which it has been used as a head office. In order to reduce costs, on 31 Dec 2009 it
moved its head office functions to one of its production center and is now letting out its head office
building. Company policy is to use FV model for Invst prop. The building had an original cost $250,000 on
1 Jan 2000 and was depreciated over 50 yrs. At 31 Dec 2009, its FV was judged to be $350,000.

How will the building present in the FS as at 31 Dec 2009?

Solution:

Extract of SOPL & OCI for the YE 31 Dec 2009 $'000


Depn exp

OCI:
Gain on revl

Extract of SOFP as at 31 Dec 2009


NCA $'000
Invst prop
(no depn to be charged)

Equity: Revaluation surplus

Note:
The building will be subjected to FV exercise at each year end and the G / (L) will go to profit or loss.

185
ACCA Financial Reporting (FR)
CHAPTER 9: IAS 40 INVESTMENT PROPERTY

Question 2

Speculate owns the following properties at 1 April 2012:

Property A:
An office building used by Speculate for administrative purposes with a depreciated historical cost of $2
million. At 1 April 2012 it had a remaining life of 20 years. After a reorganisation on 1 October 2012, the
property was let to a third party and reclassified as an investment property applying Speculate’s policy
of the fair value model. An independent valuer assessed the property to have a fair value of $2.3 million
at 1 October 2012, which had risen to $2.34 million at 31 March 2013

Property B:
Another office building sub-let to a subsidiary of Speculate. At 1 April 2012, it had a fair value of $1.5
million, which had risen to $1.65 million at 31 March 2013.

Prepare extracts from Speculate’s entity statement of profit or loss and other comprehensive income
and statement of financial position for the year ended 31 March 2013 in respect of the above
properties. In the case of property B only, state how it would be classified in Speculate’s consolidated
statement of financial position."

Solution:

Extracts from Speculate’s financial statements for the year ended 31 March 2013
(workings in brackets in $’000)
$’000
Statement of profit or loss and other comprehensive
income
Depreciation of office building
Gain on investment properties

OCI:
Statement of financial position
Non-current assets
Investment properties
Equity
Revaluation surplus

186
ACCA Financial Reporting (FR)
CHAPTER 9: IAS 40 INVESTMENT PROPERTY

Question 3

Speculator owns a number of properties. An independent surveyor has assessed their market values as:
Valuation 30 June 2000 Valuation 30 June 2001
Property Cost 1 July 1999 ($) ($) ($)
A 41,000 52,000 73,000
B 76,000 82,000 66,000
C 80,000 70,000 90,000
197,000 204,000 229,000

All properties had an estimated life of 50 years when they were acquired. They are all let on short leases
under commercial terms, however, property C is let to a fellow subsidiary of Speculator. The group
policy (applied to all members of the group) is to adopt the fair value model in IAS 40 for investment
properties and to treat owner-occupied properties under the cost model in IAS 16.

(a) Describe the possible accounting treatments of accounting for investment properties in IAS 40 and
explain why they may require a different accounting treatment to owner-occupied properties.

(b) Prepare extracts of the group FS of Speculator in respect of the above properties for the years to
30 June 2000 and 2001. How would this differ if the question asked for the entity statements of
Speculator?

Solution Question (a):


It is argued that the principal reasons for holding investment properties are that the owner expects to
receive rental income from them and benefit from capital appreciation. They are not held for
'consumption' in the normal course of business (they are not used as part of a company's operations in
the production or supply of goods and services or administrative purposes). As they are held as an
investment, it is often considered that the current values of the investments and the changes in them
are more important than their original costs. IAS 40 takes this into account by permitting a choice of
either 'cost model' or a 'fair value' model as a subsequent measurement.

Cost model
Under cost model (as in IAS 16), investment properties are measured at depreciated historical cost less
any accumulated impairment. This treatment is similar to owner-occupied properties. Where cost
model is adopted, the fair values of the investment properties must be disclosed.

Fair value model


This model requires investment properties to be measured at fair values on the SOFP with changes in
fair values (gain or loss) to be recognised in profit or loss in the period which arises and no
depreciation is to be charged. This differs from a revaluation model in IAS 16 where changes are
generally treated as movement on revaluation reserve (unrealised).

187
ACCA Financial Reporting (FR)
CHAPTER 9: IAS 40 INVESTMENT PROPERTY

Solution Question (b):


In the consolidated FS, property C would have to be classified as an owner-occupied property and
treated under IAS 16 cost model. This mean it would be carried at depreciated historical cost.

CSOPL for the year ended 30 June 2000 ($) 2001 ($)
Other income: Investment property surplus -A
-B

Exp: depreciation
Investment property deficits – B

CSOFP as at 30 June 2000 ($) 2001 ($)


NCA: PPE
Investment properties

In the entity FS of Speculator, property C would be classed as an investment property. The property
would not be depreciated. Instead a deficit of $10,000 in year 2000 and a surplus of $20,000 in year
2001 would be reported as well as the above surpluses and deficits on properties A & B.

SOPL for the year ended 30 June 2000 ($) 2001 ($)
Other income: Investment property surplus

Expenses
Depreciation
Investment property deficits

SOFP as at 30 June 2000 ($) 2001 ($)


NCA
PPE
Investment properties

188
ACCA Financial Reporting (FR)
CHAPTER 9: IAS 40 INVESTMENT PROPERTY

Question 4

(IAS 2, IAS 16, IAS 36, IAS 40):


The following trial balance relates to Jacky at 31 December 20X2:
$'m $'m
Land and buildings: cost (including $60m land) (note (ii & vii)) 380
Land and buildings - accumulated depreciation at 1 Jan 20X2 64
Plant and equipment - at cost (note (iii, vi & vii)) 258
Plant and equipment - accumulated depreciation at 1 Jan 20X2 126
Investment property at 1 Jan 20X2 (note (ix)) 548
Inventory at 1 Jan 20X2 444
Trade receivables 545
Bank 28
Trade payables 434
Revenue 2,648
Purchases 1,669
Distribution costs 519
Administrative expenses 345
Interim dividend paid 6
Interest on Loan note paid 3
Equity shares of 50 cents each 100
Share premium 244
General reserve 570
Retained earnings at 1 Jan 20X2 349
4% loan note repayable 20X8 (issued 20X0) 150
Taxation: over provision in year to 31 Dec 20X1 (note (iv)) 5
Rental income 48
Proceed from sale of equipment (note (iv)) 7
4,745 4,745

The following notes are relevant:

i. Closing inventories amounted to $358m at cost. However, shortly after the year end,
obsolescence inventories with a cost of $15m were sold for $8m.

ii. Jacky decided to changes its accounting policy with respect to its 10-year-old land and buildings
from cost model to revaluation model. The revalued amount at 1 Jan 20X2 were $800m (including
$100m for land). No further revaluation was necessary at 31 Dec 20X2. Jacky wishes to transfer a
realised profit in respect of excess depreciation on revalued assets.

189
ACCA Financial Reporting (FR)
CHAPTER 9: IAS 40 INVESTMENT PROPERTY

During the year, Jacky disposed of some equipment for $7m. The equipment had cost $15m and
had accumulated depreciation brought forward at 1 Jan 20X2 of $3m. There were no other
additions or disposal of PPE in the year.

Jacky charges depreciation on plant and equipment to cost of sales and land and building to
administrative costs. Depreciation rates are as follows:
Buildings SLM over 50 years
Plant and equipment 20% RBM

Jacky's accounting policy is to charge a full year's depreciation in the year of an asset's purchase
and none in the year of disposal.

iii. Due to a change in the company's product portfolio plans, an item of plant with carrying amount
of $22m at 31 Dec 20X1 may be impaired due to a change in use. An impairment test conducted
at 1 Jan 20X2 revealed its fair value less costs to sell to be $16m. The asset is now expected to
generate an annual net income stream of $3.8m for the next 5 years at which point the asset
would be disposed for $4.2m. An appropriate discount rate is 8%. 5 years’ discount factors at 8%
are (calculate to the nearest million):

Year DF @ 8%
1 0.93
2 0.86
3 0.79
4 0.74
5 0.68

iv. The income tax liability for the year is estimated at $27m. Ignore deferred tax.
v. An interim dividend of 3c per share was paid on 30 June 20X2. A final dividend of 2c per share was
declared by the directors on 31 Jan 20X3.
vi. During the year on 1 July 20X2, Jacky made a 1 for 4 bonus issue, capitalizing its general reserve.
This transaction had not been accounted for. The fair value of the company's shares on the date
of the bonus issue was $6.50 each.
vii. Jacky uses the fair value model of IAS 40. The fair value of the investment property at 31 Dec 20X2
was $586m.

Required
(a) Prepare the SOPL & OCI for the year ended 31 December 20X2.
(b) Prepare the statement of changes in equity for the year ended 31 December 20X2.
(c) Prepare the statement of financial position as at 31 December 20X2.

190
ACCA Financial Reporting (FR)
CHAPTER 9: IAS 40 INVESTMENT PROPERTY

9.11 Chapter 9 Summary

Diagram 9.11.: Summary of IAS 40 Investment Property

191
ACCA Financial Reporting (FR)
CHAPTER 10: IAS 37 PROVISIONS, CONTINGENT LIABILITIES & CONTINGENT ASSETS

CHAPTER 10: IAS 37 PROVISIONS,


CONTINGENT LIABILITIES &
CONTINGENT ASSETS
LEARNING OUTCOME
At the end of the chapter, you should be able to:

TLO B7a. Explain why an accounting standard on provision is necessary.

TLO B7b. Distinguish between legal and constructive obligations.

TLO B7c. State when provisions may and may not be made and Demonstrate how they should be
accounted for.

TLO B7d. Explain how the provisions should be measured.

TLO B7e. Define contingent assets and liabilities and Describe their accounting treatment and required
disclosure.

192
ACCA Financial Reporting (FR)
CHAPTER 10: IAS 37 PROVISIONS, CONTINGENT LIABILITIES & CONTINGENT ASSETS

10.1 Overview

Learning Outcome (ACCA Study Guide Area B)


B7a : Explain why an accounting standard on provision is necessary.

Preparers of financial statements are required to exercise judgement when preparing and presenting
financial information. Accounting for provisions and contingencies is particularly problematic due to the
increased level of uncertainty. The key issues include:
• Whether a provision or contingency should be recognised
• If it is recognised at what amount it should be recorded

The objective of this Standard is to ensure that appropriate recognition criteria and measurement bases
are applied to provisions, contingent liabilities and contingent assets and that sufficient information is
disclosed in the notes to enable users to understand their nature, timing and amount.

Prior to the introduction of IAS 37, there was no accounting standard dealing with provisions. Companies
wanting to present their year-on-year results in the most favourable light would make large one-off
provisions in years where a high level of profits was generated. These provisions were then available to
shield expenditure in years where profits were lower. This created the misleading impression that profits
were smoothly rising from year to year, with no peaks and troughs.

IAS 37 was introduced to end such abuses and the principle that provisions are only made where there
are valid grounds for them must be maintained. This applies equally to any changes made to provisions.
The integrity of management as a factor in financial statement preparation can never be overlooked.

10.1.1 Scope

This Standard shall be applied by all entities in accounting for provisions, contingent liabilities and
contingent assets, except:

(a) those resulting from executory contracts, except where the contract is onerous; and
(b) those covered by another Standard

Executory contracts are contracts under which neither party has performed any of its obligations or both
parties have partially performed their obligations to an equal extent. This Standard does not apply to
executory contracts unless they are onerous.

When another Standard deals with a specific type of provisions, contingent liability or contingent asset,
an entity applies that Standard instead of this Standard. For example, some types of provisions are
addressed in Standards on:

(i) Construction contracts;


(ii) Income taxes (IAS 12 Income Taxes);

193
ACCA Financial Reporting (FR)
CHAPTER 10: IAS 37 PROVISIONS, CONTINGENT LIABILITIES & CONTINGENT ASSETS

(iii) Leases (IFRS 16 Leases). However, IFRS 16 contains no specific requirements to deal with operating
leases that have become onerous, this Standard applies to such cases.
(iv) Employee benefits (IAS 19 Employee Benefits); and
(v) Insurance contracts (see IFRS 4 Insurance Contracts); However, this Standard applies to provisions,
contingent liability or contingent asset of insurer, other than those arising from its contractual
obligations and rights under insurance contracts within the scope of IFRS 4.

10.2 Definition

Learning Outcome (ACCA Study Guide Area B)


B7b : Distinguish between legal and constructive obligations.

Provision is defined as liabilities of uncertain timing or amount.

Provisions can be distinguished from other liabilities such as trade payables and accruals because there is
uncertainty about the timing or amount of the future expenditure required in settlement. By contrast:

a) trade payables are liabilities to pay for goods or services that have been received or supplied and
have been invoiced or formally agreed with the supplier; and

b) accruals are liabilities to pay for goods or services that have been received or supplied but have not
been paid, invoiced or formally agreed with the supplier. Although it is sometimes necessary to
estimate the amount or timing of accruals, the uncertainty is generally much less than for
provisions.

Accruals are often reported as part of trade and other payables, whereas provisions are reported
separately.

Relationship between provisions and contingent liabilities

All provisions are generally contingent because they are uncertain in timing or amount. The term ‘contingent
liability’ is used for liabilities that do not meet ALL the recognition criteria.

This Standard distinguishes between:

a) Provisions - which are recognized as liabilities (assuming that a reliable estimate can be made) because
they are present obligations and it is probable that an outflow of resources embodying economic
benefits will be required to settle the obligations; and
b) Contingent liabilities - which are not recognized as liabilities because they are either:

(i) Possible obligations; as it has not yet confirmed whether the entity has a present obligation that
could lead to an outflow of resources embodying economic benefits; or

(ii) Present obligation that do not meet the recognition criteria in this Standard (because either it is
not probable that an outflow of resources embodying economic benefits will be required to settle
the obligation, or a sufficiently reliable estimate of the amount of the obligation cannot be made).

194
ACCA Financial Reporting (FR)
CHAPTER 10: IAS 37 PROVISIONS, CONTINGENT LIABILITIES & CONTINGENT ASSETS

10.3 Recognition

Learning Outcome (ACCA Study Guide Area B)


B7c : State when provisions may and may not be made and Demonstrate how they should be
accounted for.
A provision should be recognised when:
(a) an entity has a present obligation (legal or constructive) as a result of a past event;
(b) it is probable that an outflow of resources embodying economic benefits will be required to settle
the obligation; and
(c) a reliable estimate can be made of the amount of the obligation.

If ALL these conditions are not met, no provision shall be recognised.

It is important to note that the definition of a liability in this standard was not revised following the revision
of the definition of a liability in the Conceptual Framework for Financial Reporting issued in 2018.

10.3.1 Present Obligation

A past event is deemed to give rise to a present obligation if, taking account of all available evidence,
including any additional evidence provided by events after the reporting period (IAS 10), it is more likely
than not that a present obligation exists at the end of the reporting period.

After taking into account such evidences:


(a) If it is more likely than not that a present obligation exists at the end of the reporting period, the entity
recognises a provision (if the recognition criteria are met); and
(b) Where it is more likely that no present obligation exists at the end of the reporting period (<50%
chance), entity discloses a contingent liability, unless the possibility of an outflow of resources
embodying economic benefits is remote.

Legal and constructive obligations

Legal obligation - An obligation that derives from:

• A contract (through explicit or implicit terms);


• Legislation; or
• Other operation of law.
An example of a legal obligation would be a warranty provided at the time of sale to undertake necessary
repairs for a specified period of time.

Constructive obligation - An obligation that derives from an entity's actions where:

195
ACCA Financial Reporting (FR)
CHAPTER 10: IAS 37 PROVISIONS, CONTINGENT LIABILITIES & CONTINGENT ASSETS

• By an established pattern of past practice, published policies or a sufficiently specific current


statement, the entity has indicated to other parties that it will accept certain responsibilities; and

• As a result, the entity has created a valid expectation on the part of those other parties that it will
discharge those responsibilities.

Constructive obligations are more difficult to identify with certainty than legal obligations. In practice they
are recognised where the situation has much the same commercial effect as a legal obligation. In other
words, in practice, the entity cannot avoid settling the obligation.

For example, there is likely to be a constructive obligation where failure to do something would result in
unacceptable damage to an entity's reputation or future business.

For example:

A retail store operates a policy of giving refunds to customers that goes beyond the company's legal
obligations. The policy is long established and widely known. It is likely that this policy creates a
constructive obligation, as a significant breach of the policy would damage the company's reputation
considerably.

10.3.2 Past Events

As Financial statements deal with the financial position of an entity at the end of its reporting period and
not its possible position in the future. Therefore, no provision is recognised for costs that need to be
incurred to operate in the future. The only liabilities recognised in an entity’s statement of financial
position are those that exist at the end of the reporting period.

196
ACCA Financial Reporting (FR)
CHAPTER 10: IAS 37 PROVISIONS, CONTINGENT LIABILITIES & CONTINGENT ASSETS

10.3.3 Probable outflows of resources embodying economic


benefits

For a liability to qualify for recognition there must be not only a present obligation but also the probability
of an outflow of resources embodying economic benefits to settle that obligation. For the purpose of this
Standard, * an outflow of resources or other event is regarded as probable if the event is more likely than
not to occur, i.e. the probability that the event will occur is greater than the probability that it will not.
Where it is not probable that a present obligation exists, an entity discloses a contingent liability, unless
the possibility of an outflow of resources embodying economic benefits is remote.

10.3.4 Reliable estimate of the obligation

The use of estimates is an essential part of the preparation of financial statements and does not
undermine their reliability. Provisions are liabilities of uncertain timing and amount. Therefore, if an entity
is able to determine a range of possible outcomes and can therefore make an estimate of the obligation
that is sufficiently reliable to use in recognising a provision.

If no reliable estimate can be made, (extreme rare case), that a liability exists then the liability cannot be
recognised. That liability is disclosed as a contingent liability.

Points to note:

1. The event must be past, i.e., it must have occurred at the end of the reporting period. No provision is
made for costs that may be incurred in the future but where no obligation yet exists.

2. Only obligations arising from past events existing independently of an entity's future actions (i.e.,
the future conduct of its business) are recognised as provisions. If management can avoid incurring
expenditure by changing the entity's future operations, no provision arises.

3. An obligation always involves another party to whom the obligation is owed. However, the exact
identity of that other party need not be known, e.g., the obligation may be to the public at large.

4. A board or management decision does not give rise to an obligation unless it has been communicated
before the end of the reporting period to those affected by it so as to raise a valid expectation that
the entity will discharge its responsibilities. In the absence of such communication, the board could
change its mind and hence would be under no obligation.

5. Sometimes, the existence of an obligation will be uncertain, e.g., where there is a legal dispute. In
these cases, IAS 37 deems a past event to give rise to a present obligation if it is more likely than not
that an obligation exists at the end of the reporting period. However, if it is possible rather than
probable that an obligation exists, a contingent liability will exist, not a provision (see below).

197
ACCA Financial Reporting (FR)
CHAPTER 10: IAS 37 PROVISIONS, CONTINGENT LIABILITIES & CONTINGENT ASSETS

Example 10.3.4 - Present obligation as a result of a past event

Company A carries out quarrying activities. A condition of the planning consent is that environmental
damage caused by quarrying must be remedied on completion of the quarrying. In this case, an obligation
exists independently of the company's future conduct in relation to damage already caused at the end of
the reporting period, because the company cannot avoid having to pay for remedial action. By contrast,
no obligation exists in relation to expected further damage from continued quarrying because the
company could decide not to quarry in the future.

Company B operates aircraft that need periodic overhauls if they are to continue in operation. No
obligation exists in relation to future overhauls because the company could decide to sell or scrap the
aircraft rather than overhaul them.

10.4 Contingent Liabilities

Learning Outcome (ACCA Study Guide Area B)


B7e : Define contingent assets and liabilities and Describe their accounting treatment and required
disclosure.

10.4.1 Introduction

Contingent liability is either:

• A possible obligation that arises from past events and whose existence will be confirmed only by the
occurrence or non-occurrence of one or more uncertain future events not wholly within the control
of the entity, OR

• A present obligation that arises from past events but is not recognised because:
- It is not probable that an outflow of resources embodying economic benefits will be required to
settle the obligation; OR
- The amount of the obligation cannot be measured with sufficient reliability.

Points to note:

1. Note the distinction between a provision and a contingent liability. A contingent liability arises when
some, but not all, of the criteria for recognizing a provision are met.
2. If an obligation is probable it is not a contingent liability – instead a provision is needed, assuming a
reliable estimate can be made of the amount of the obligation.

198
ACCA Financial Reporting (FR)
CHAPTER 10: IAS 37 PROVISIONS, CONTINGENT LIABILITIES & CONTINGENT ASSETS

10.4.2 Treatment of Contingent Liabilities


Contingent liabilities should not be recognized in the financial statements, but may require disclosure.
Because contingent liabilities are inherently uncertain, they should be assessed continually to identify
whether the criteria for recognising a provision have been met. If this occurs, a provision should be
recognized in the period in which the criteria are met. This would represent a change of accounting
estimate regarding the likely outcome of an uncertain situation.

10.4.3 Disclosure of Contingent Liabilities

Unless the possibility of any outflow in settlement is remote, the following disclosures should be made
for each class of contingent liability at the end of the reporting period:

• A brief description of its nature; and


• Where practicable:

– An estimate of the financial effect (measured in the same way as a provision)


– An indication of the uncertainties; and
– The possibility of any reimbursement.

No specific guidance is provided in IAS 37 on the meaning of ‘remote' but it should be interpreted as
meaning extremely unlikely. This means that the probability of an event occurring should be so small that
it can be ignored.

Example 10.4.3 (a) Contingent Liability

A company has provided a guarantee to a third party which, if it were to be called on to honour it, would
undermine the going concern basis. In such a situation, even a 5% or 10% chance that the guarantee will
be enforced should not be considered remote as this could potentially destroy the entire company.

Example 10.4.3 (b)

During 2010, Gaga Co gives a guarantee of certain borrowings of Lady Co, whose financial condition at
that time is sound. During 2011, the financial condition of Lady Co deteriorates and at 30 June 2011,
Lady Co files for protection from its creditors.

What accounting treatment is required in the book of Gaga Co:

(a) At 31 Dec 2010 Disclose as Contingent Liability


(b) At 31 Dec 2011 Recognised as Provision

199
ACCA Financial Reporting (FR)
CHAPTER 10: IAS 37 PROVISIONS, CONTINGENT LIABILITIES & CONTINGENT ASSETS

10.5 Contingent Assets

Learning Outcome (ACCA Study Guide Area B)


B7e : Define contingent assets and liabilities and Describe their accounting treatment and required
disclosure.

10.5.1 Introduction

A contingent asset is a possible asset that arises from past events and whose existence will be confirmed
only by the occurrence or non-occurrence of one or more uncertain future events not wholly within the
control of the entity. An example of a contingent asset is the possible gain arising from a pending legal
action or other claim.

10.5.2 Treatment of Contingent Assets

A contingent asset must NOT be recognized in the financial statements. Only when the realisation of the
related economic benefits is virtually certain should recognition take place because, at that point, the
asset is no longer contingent.

Contingent assets should be assessed continually to identify whether the uncertainty has been removed.
If events confirm the existence of an asset, it should be recognised provided that it can be measured
reliably.

10.5.3 Disclosure of Contingent Assets

Where an inflow of economic benefits is probable, i.e. more likely than not, the contingent asset must be
disclosed.

The following information is required:


• A brief description of the nature of the contingent asset.
• An estimate of the financial effect.

As for contingent liabilities, these disclosures may be avoided on the grounds that it is impractical to
provide the information or would be seriously prejudicial to the entity.

200
ACCA Financial Reporting (FR)
CHAPTER 10: IAS 37 PROVISIONS, CONTINGENT LIABILITIES & CONTINGENT ASSETS

Example 10.5.3

As a result of new banking regulations, International plc will need to retrain a large proportion of its
financial services division in order to ensure continued compliance with banking regulations. At the end
of the reporting period no retraining of staff has taken place. However, the head of the financial services
division has announced that he is committed to a completion of the retraining programme by the end of
the following year.
Carefree plc, is also subject to the same banking regulations, by the end of the reporting period Carefree
plc has contracted a training organisation to undertake the retraining programme with a start date of 15
January, two weeks after the reporting period. Staff have been notified of their training session dates.
In accordance with IAS 37 how should each company account for the cost of retraining their staff?

Solution:
At the end of the reporting period neither company has an obligation to pay for training as no training has
been carried out. Therefore, no provision can be provided as the cost of retraining is a future cost, as such
no past event.

10.6 Measurement

Learning Outcome (ACCA Study Guide Area B)


B7d : Explain how the provisions should be measured.

The amount recognised as a provision shall be the best estimate of the expenditure required to settle the
present obligation at the end of the reporting period.

➢ Obligation is estimated by weighting


all possible outcomes by their
Large Population of Items
associated probabilities.
➢ Usage of expected value.

➢ Individual most likely outcome may


Single Obligation
be the best estimate of the liability.

201
ACCA Financial Reporting (FR)
CHAPTER 10: IAS 37 PROVISIONS, CONTINGENT LIABILITIES & CONTINGENT ASSETS

Example 10.6 Single Obligation

If the expenditure for a single obligation is estimated at $10,000 and there is a 55% chance of the expenditure being
incurred, then $10,000 is provided for. The process of estimating the amount involves two separate steps:

Step 1
Is it probable that there will be an outflow of economic resources (arising from a present obligation)?
Yes, there is in this case, as there is a 55% probability.

Step 2
What reliable estimate can be made? $10,000 in this case.

Points to note:
1. An expected value calculation (see next example) is not relevant for a single obligation.
2. Where a single obligation is being measured, the individual most likely outcome may be the best estimate
of the liability. However, even in this case, the entity should consider other outcomes. Where other
possible outcomes are either mostly higher or mostly lower than the most likely outcome, the best
estimate will be a higher or lower amount.

10.6.1 Expected Values

Where there is a large population of items, the obligation is estimated by weighting all possible outcomes
by their associated probabilities, to arrive at the expected value.

Exercise 10.6.1 (a) Large population of items using expected value

An entity sells goods with a warranty under which customers are covered for the cost of repairs of any
manufacturing defects that become apparent within the first six months after purchase. The entity’s past
experience and future expectations indicate that, for the coming year, 75 per cent of the goods sold will have
no defects, 20 per cent of the goods sold will have minor defects and 5 per cent of the goods sold will have
major defects.
Defects Repair Cost
$m

Minor 1
Major 4
Solution
An entity assesses the probability of an outflow for the warranty obligations as a whole. The expected value of
the cost of repairs are:
No Defects Minor Defects Major Defects

(75% of Nil) + (20% of $ 1m) + (5% of $ 4m) = $ 400,000

Amount of provision for warranty cost.


Debit Warranty cost (expense)
Credit Provision for warranty (liability)

202
ACCA Financial Reporting (FR)
CHAPTER 10: IAS 37 PROVISIONS, CONTINGENT LIABILITIES & CONTINGENT ASSETS

The provision is measured before tax, as the tax consequences of the provision, and changes in it, are
dealt with under IAS 12.

Exercise 10.6.1 (b)

An entity sells goods with warranty to repair any defects that are discovered within the first 2 months
after purchasing. Past experience suggests that 88% of the goods sold will have no defects, 7% will have
minor defects and 5% will have major defects. If minor defects were detected in all products sold, the
cost of repairs would be $24,000; if major defects were detected, the cost would be $200,000.

Amount of provision made =

Exercise 10.6.1 (c)

Mikel has been contacted by a customer because a part he supplied has damaged one of the
customer’s machines. The customer has asked Mikel to pay costs of $5,500. Mikel’s offer to pay $3,000
has been refused by the customer, leading to legal action. Mikel’s solicitor has told him that the court
case will not take place until late 2011, and that it is almost certain that he will be required to pay
$4,000.

Amount of provision made =

Exercise 10.6.1 (d)


Arena sells a line of goods under a six-month warranty. Any defect arising during that period is repaired
free of charge. Arena has calculated that if all the goods sold in the last six months of the year required
repairs the cost would be $4 million. If all of these goods had more serious faults and had to be
replaced the cost would be $8 million. The normal pattern is that 60% of goods sold will be fault-free,
25% will require repairs and 15% will have to be replaced.
What is the amount of the provision required?

Solution:

203
ACCA Financial Reporting (FR)
CHAPTER 10: IAS 37 PROVISIONS, CONTINGENT LIABILITIES & CONTINGENT ASSETS

10.6.2 Risk and Uncertainties

The risks and uncertainties that inevitably surround many events and circumstances shall b e taken into
account in reaching the best estimate of a provision.

10.6.3 Future Events

Future events such as changes in technologies, efficiency improvements and changes in legislation may
have a significant impact on the measurement of provisions. These should be taken into account where
there is sufficient objective evidence that they will occur.

10.6.4 Expected disposal of assets

Gains from the expected disposal of assets should not be taken into account in measuring a provision even
if the expected disposal is closely linked to the event giving rise to the provision. Instead, such gains are
accounted for under the relevant IFRS, i.e., IAS 16 Property, Plant and Equipment and IFRS 5 Non-current
Assets Held for Sale and Discontinued Operations for PPE.

10.6.5 Discounting

Where the effect of the time value of money is material, the amount of the provision should be
discounted. In other words, it should be recorded at the present value of the expenditure required to
settle the obligation. This is likely to be an issue when there is a significant period of time between the
end of the reporting period and settlement of the obligation.

The discount rate used should be the pre-tax rate that reflects current market assessments of the time
value of money and the risks specific to the liability.

Over the period between the recognition of a provision and its ultimate settlement, the provision should
be increased each year by the discount rate. The increase should be recognised as a finance cost in profit
or loss, not as a further expense under the line item where the original provision was charged.

The double-entry will be:

DR Finance costs (profit or loss)


CR Provision

204
ACCA Financial Reporting (FR)
CHAPTER 10: IAS 37 PROVISIONS, CONTINGENT LIABILITIES & CONTINGENT ASSETS

Points to note:

1. The entity’s average borrowing rate should not be used as the discount rate. The discount rate should
be pre-tax and reflect current market rates of the time value of money and the risks specific to the
liability.

2. The discount rate should not be risk-adjusted if the cash flows already take account of this. This is
relevant when risk is being incorporated directly in cash flows (for example a potential outflow may
be increased to reflect its greater risk). When this has been done, adding a risk premium to the
discount rate would be double-counting the risk effect

3. Disclosure should be made of the increase during the period in the discounted amount arising from
the passage of time and the effect of any change in the discount rate.

Exercise 10.6.5 (a) – Discounting

A company has a present obligation at 31 December 20X6, which it expects to settle in three years' time
for $100,000. The rate which reflects the time value of money and the risks specific to the liability is 10%.

Required:
- At what amount should the provision be measured at 31 December 20X6?
- How much should be recognised as a finance charge in each of the three years ending 31 December
20X7, 20X8 and 20X9?
- What accounting entry is required to record the finance charge in the year ending 31 December 20X7?

Solution:
The finance charge in each of the three years is calculated as:

Balance b/f Finance Cost Balance c/f


(Opening) @ 10% 31 Dec (Closing)
$ $ $
20X7
20X8
20X9

The accounting entry to record the finance charge in 20X7 is:

DR Finance costs in profit or loss


CR Provisions

205
ACCA Financial Reporting (FR)
CHAPTER 10: IAS 37 PROVISIONS, CONTINGENT LIABILITIES & CONTINGENT ASSETS

Exercise 10.6 (a)


During the year Davis acquired an iron ore mine at a cost of $10 million. In addition, when all the ore has
been extracted (estimated ten years' time) the company will face estimated costs for landscaping the area
affected by the mining that have a present value of $4 million. These costs would still have to be incurred
even if no further ore was extracted.

How should this $4 million future cost be recognised in the financial statements?
A. Accrual $400,000 per annum for next ten years
B. Provision $4 million and $4 million charged to operating costs
C. Provision $4 million and $4 million capitalised as part of cost of mine
D. Should not be recognised as no cost has yet arisen

Exercise 10.6 (b)


Moses Co has constructed a building on a piece of land at 1/1/2016 that required the Co to demolish the
building at the end of 10 years and restore the site to its original condition. Estimation of site restoration
is $50,000 and relevant interest rate in this case is 10%. Cost incurred to build the building is $500,000.

Double Entry @ 1/1/2016


DR NCA
CR Bank
CR Provision for site restoration

The increase of provision at year end will be charged to profit or loss as finance cost. It is referred to as
the unwinding of discount.

Double Entry
DR Finance Cost
CR Provision

Extract of SOPL for the YE 31/12/2016


$
Depreciation
Finance Cost

Extract of SOFP as at YE 31/12/2016


$
NCA: PPE
NCL: Provision for site restoration

206
ACCA Financial Reporting (FR)
CHAPTER 10: IAS 37 PROVISIONS, CONTINGENT LIABILITIES & CONTINGENT ASSETS

10.7 Reimbursement

Where some or all of the expenditure required to settle a provision is expected to be reimbu rsed by
another party, the reimbursement shall be recognised when, and only when, it is virtually certain that
reimbursement will be received if the entity settles the obligation. The reimbursement shall be treated
as a separate asset. The amount recognised for the reimbursement shall not exceed the amount of the
provision.

In the statement of profit or loss, the expense relating to a provision may be presented net of the amount
recognised for a reimbursement.

Example 10.7 (a) - Reimbursements

Construction plc was awarded a contract to build a tunnel. Construction plc, delegated some aspects of the
contract to other companies. One of the sub-contractors, Underwater Ltd, was negligent in the performance
of its contract with Construction plc, which caused delay in the completion of the tunnel.

As a result of the delay, the customer is claiming damages of $10m against Construction plc. In turn,
Construction plc has commenced proceedings against Underwater Ltd. The lawyers have advised Construction
Ltd that both actions are likely to be successful.

Required:
In accordance with IAS 37 how should Construction plc account for the legal claims?

Solution:
The claim against Construction plc represents a probable loss and a provision should be provided in the
financial statement. The counter claim against Underwater Ltd represents a contingent asset which is
probable and should be disclosed.

Exercise 10.8 (b)

An employee sues the Co for compensation. Lawyer advised that it is highly likely that the Co would have
to pay $50,000. Reimbursement from insurance Co is confirmed at $30,000.

Double entry for prov created: Double entry for reimbursement:

Extract of SOPL $ Extract of SOFP $


Other expenses - CA: Recv
net compensation
CL: Prov for compensation

207
ACCA Financial Reporting (FR)
CHAPTER 10: IAS 37 PROVISIONS, CONTINGENT LIABILITIES & CONTINGENT ASSETS

10.8 Changes in provisions and judgement required


Provisions are inherently uncertain and IAS 37 requires that they should be reviewed at the end of each
reporting period and adjusted to reflect the current best estimate. If a transfer of economic benefit is no
longer probable, the provision should be reversed.

The application of the principles of IAS 37 in setting up and revising the amounts of provisions requires a
great deal of estimation and judgement on the part of management. This is an area where it is vital that
clear and comprehensive policies and procedures are laid down and then applied consistently from one
period to another.

Even then, the carrying amounts of provisions are estimates of the effect of uncertain future events, so it
is entirely legitimate for different people to take different views. The important objective is to make
estimates which are neutral in terms of the information provided, rather than designed to achieve a
predetermined profit or net asset figure.

10.9 Specific applications

10.9.1 Future Operating Losses

(a) Provisions shall not be recognised for future operating losses.

(b) Future operating losses do not meet the definition of a liability and the general recognition criteria
set out for provisions.

(c) An expectation of future operating losses is an indication that certain assets of the operation may be
impaired. An entity tests these assets for impairment under IAS 36 Impairment of Assets.

10.9.2 Onerous Contract

If an entity has a contract that is onerous, the present obligation under the contract shall be recognised
and measured as a provision.

Many contracts (for example, some routine purchase orders) can be cancelled without paying
compensation to the other party, and therefore there is no obligation. Other contracts establish both
rights and obligations for each of the contracting parties. Where events make such a contract onerous,
the contract falls within the scope of this Standard and a liability exists which is recognised.

This Standard defines an onerous contract as a contract in which the unavoidable costs of meeting the
obligations under the contract exceed the economic benefits expected to be received under it.

208
ACCA Financial Reporting (FR)
CHAPTER 10: IAS 37 PROVISIONS, CONTINGENT LIABILITIES & CONTINGENT ASSETS

Example 10.9.2

You provide electricity to commercial and domestic clients. The government has instructed you to cut
domestic charges by 5% and fix the price for 2 years. This will cost you $30 million loss, as you cannot reduce
the cost of your supplies.

Solution:

This is an onerous contract, and a provision should be made for the $30 million loss.

10.9.3 Restructuring

A restructuring is a programme that is planned and controlled by management, and materially changes
either:
• The scope of a business undertaken by an entity; or
• The manner in which that business is conducted.

Examples of restructuring:

(a) Sale or termination of a line of business;


(b) The closure of business locations in a country or region or the relocation of business activities from
one country or region to another;
(c) Changes in management structure, for example, eliminating a layer of management; and
(d) Fundamental reorganisations that have a material effect on the nature and focus of the entity’s
operations.

A provision for restructuring costs is recognised only when the general recognition criteria for provisions.

1. Recognition criteria for restructuring cost:

(a) Has a detailed formal plan for the restructuring identifying at least:

(i) the business or part of a business concerned;


(ii) the principal locations affected;
(iii) the location, function, and approximate number of employees who will be compensated
for terminating their services;
(iv) the expenditures that will be undertaken; and
(v) when the plan will be implemented; and

209
ACCA Financial Reporting (FR)
CHAPTER 10: IAS 37 PROVISIONS, CONTINGENT LIABILITIES & CONTINGENT ASSETS

(b) Has raised a valid expectation in those affected that it will carry out the restructuring by starting
to implement that plan or announcing its main features to those affected by it.

Evidence that restructuring plan is implemented:

• by dismantling plant or;


• selling assets or;
• by the public announcement of the main features of the plan.

A public announcement of a detailed plan to restructure constitutes a constructive obligation to


restructure only if it is made in such a way and in sufficient detail (i.e. setting out the main features of the
plan) that it gives rise to valid expectations in other parties such as customers, suppliers and employees
(or their representatives) that the entity will carry out the restructuring.

2. Constructive obligation at the end of reporting period

A management or board decision to restructure taken before the end of the reporting period does not
give rise to a constructive obligation at the end of the reporting period unless the entity has, before the
end of the reporting period:

(a) started to implement the restructuring plan; or

(b) announced the main features of the restructuring plan to those affected by it in a sufficiently specific
manner to raise a valid expectation in them that the entity will carry out the restructuring.

3. Announcement of restructuring plan after the year end

If an entity starts to implement a restructuring plan, or announces its main features to those affected,
only after the reporting period, disclosure is required under IAS 10 Events after the Reporting Period, if
the restructuring is material and non-disclosure could influence the economic decisions that users make
on the basis of the financial statements.

4. Obligation for the sale of an operation

No obligation arises for the sale of an operation until the entity is committed to the sale, i.e. there is a
binding sale agreement.

Even when an entity has taken a decision to sell an operation and announced that decision publicly, it
cannot be committed to the sale until a purchaser has been identified and there is a binding sale
agreement. Until there is a binding sale agreement, the entity will be able to change its mind and indeed
will have to take another course of action if a purchaser cannot be found on acceptable terms. When the
sale of an operation is envisaged as part of a restructuring, the assets of the operation are reviewed for
impairment, under IAS 36. When a sale is only part of a restructuring, a constructive obligation can ar ise
for the other parts of the restructuring before a binding sale agreement exists.

210
ACCA Financial Reporting (FR)
CHAPTER 10: IAS 37 PROVISIONS, CONTINGENT LIABILITIES & CONTINGENT ASSETS

5. Expenditures included in a restructuring provision:

A restructuring provision shall include only the direct expenditures arising from the restructuring, which
are those that are both:

(a) necessarily entailed by the restructuring; and

(b) not associated with the ongoing activities of the entity.

A restructuring provision does not include such costs as:

(i) retraining or relocating continuing staff;


(ii) marketing; or
(iii) investment in new systems and distribution networks.

These expenditures relate to the future conduct of the business and are not liabilities for restructuring at
the end of the reporting period. Such expenditures are recognised on the same basis as if they arose
independently of a restructuring.

Identifiable future operating losses up to the date of a restructuring are not included in a provision, unless
they relate to an onerous contract.

Gains on the expected disposal of assets are not taken into account in measuring a restructuring provision,
even if the sale of assets is envisaged as part of the restructuring.

Example 10.9.3 (Extract of Dec’10 Q5) Provision for Restructuring

Manco has been experiencing substantial losses at its furniture making operation which is treated as a
separate operating segment. The company’s year-end is 30 September. At a meeting on 1 July 2010 the
directors decided to close down the furniture making operation on 31 January 2011 and then dispose of its
non-current assets on a piecemeal basis. Affected employees and customers were informed of the decision
and a press announcement was made immediately after the meeting. The directors have obtained the
following information in relation to the closure of the operation:

(i) On 1 July 2010, the factory had a carrying amount of $3·6 million and is expected to be sold for net
proceeds of $5 million. On the same date the plant had a carrying amount of $2·8 million, but it is
anticipated that it will only realise net proceeds of $500,000.

(ii) Of the employees affected by the closure, the majority will be made redundant at cost of $750,000, the
remainder will be retrained at a cost of $200,000 and given work in one of the company’s other
operations.

(iii) Trading losses from 1 July to 30 September 2010 are expected to be $600,000 and from this date to the
closure on 31 January 2011 a further $1 million of trading losses are expected.

Required:
Explain how the decision to close the furniture making operation should be treated in Manco’s FS for the
years ending 30 September 2010 and 2011. Your answer should quantify the amounts involved.

211
ACCA Financial Reporting (FR)
CHAPTER 10: IAS 37 PROVISIONS, CONTINGENT LIABILITIES & CONTINGENT ASSETS

Solution:

From the information in the question, the closure of the furniture making operation is a restructuring as
defined in IAS 37 and, due to the timing of the decision, a provision for the closure costs will be required in
the year ended 30 September 2010 as they have raised a valid expectation in those affected (i.e.: informing
employees, customers and a press announcement) indicate that this is an irreversible decision and that
therefore there is an obligating event.

Commenting on each element in turn for both years:

Factory

At September 2010

Factory continued to be depreciated for the 3 months ended 30 Sep 2010 as it was still in used (closed
down only in 31 Jan 2011).

The expected gain on the sale of the factory cannot be recognised and should not be taken into account
in measuring of the provision for restructuring.

At September 2011

The realised profit on the disposal of the factory will be reported in the SOPL (as gain only recognise in
the period which it arises).

Plant

At September 2010

It did appear that the plant is impaired. Based on its carrying amount of $2·8 million, an impairment charge
of $2·3 million ($2·8 million – $0·5 million) would be required. The impairment charge is not part of the
restructuring provision, but should be reported with the depreciation charge for the year.

Plant continued to be depreciated for the 3 months ended 30 Sep 2010 based on the impairment amount
of $0.5m as it was still in used (closed down only in 31 Jan 2011).

At 30 September 2011

Any further loss on the disposal of the plant will both be reported in the SOPL.

212
ACCA Financial Reporting (FR)
CHAPTER 10: IAS 37 PROVISIONS, CONTINGENT LIABILITIES & CONTINGENT ASSETS

Redundancy and retraining costs

At 30 September 2010

A provision for the redundancy costs of $750,000 should be made, but the retraining costs relate to the
ongoing actives of Manco and cannot be provided for.

At 30 September 2011

The redundancy costs incurred during the year will be offset against the provision created last year. Any
under- or over-provision will be reported in the SOPL.
The retraining costs will be written off as they are incurred.

Trading losses

At 30 September 2010

The losses of $600,000 will be reported as part of the results for the year ended 30 September 2010.

The future operating losses from 1 October 2010 to the closure on 31 January 2011 cannot be provided in
the year ended 30 September 2010.

At 30 September 2011

The actual losses incur will be reported as part of the results for the year ended 30 September 2011.

213
ACCA Financial Reporting (FR)
CHAPTER 10: IAS 37 PROVISIONS, CONTINGENT LIABILITIES & CONTINGENT ASSETS

10.10 Disclosures

For each class of provision, an entity shall disclose:

(a) the carrying amount at the beginning and end of the period;

(b) additional provisions made in the period, including increases to existing provisions;

(c) amounts used (i.e. incurred and charged against the provision) during the period;

(d) unused amounts reversed during the period; and

(e) the increase during the period in the discounted amount arising from the passage of time and the
effect of any change in the discount rate.

Comparative information is not required.

An entity shall disclose the following for each class of provision:

(a) a brief description of the nature of the obligation and the expected timing of any resulting outflows
of economic benefits;

(b) an indication of the uncertainties about the amount or timing of those outflows. Where necessary to
provide adequate information, an entity shall disclose the major assumptions made concerning future
events, as addressed and

(c) the amount of any expected reimbursement, stating the amount of any asset that has been
recognised for that expected reimbursement.

Unless the possibility of any outflow in settlement is remote, an entity shall disclose for each class of
contingent liability at the end of the reporting period a brief description of the nature of the contingent
liability and, where practicable:

(a) an estimate of its financial effect,

(b) an indication of the uncertainties relating to the amount or timing of any outflow; and

(c) the possibility of any reimbursement.

214
ACCA Financial Reporting (FR)
CHAPTER 10: IAS 37 PROVISIONS, CONTINGENT LIABILITIES & CONTINGENT ASSETS

Example 10.10 (a) – Warranties

A manufacturer gives warranties at the time of sale to purchasers of its three product lines. Under the
terms of the warranty, the manufacturer undertakes to repair or replace items that fail to perform
satisfactorily for two years from the date of sale. At the end of the reporting period, a provision of 60,000
has been recognised. The provision has not been discounted as the effect of discounting is not material.
The following information is disclosed:

A provision of 60,000 has been recognised for expected warranty claims on products sold during the last
three financial years. It is expected that the majority of this expenditure will be incurred in the next
financial year, and all will be incurred within two years after the reporting period.

Example 10.10 (b) – Decommissioning Cost

In 2000, an entity involved in nuclear activities recognizes a provision for decommissioning costs of 300
million. The provision is estimated using the assumption that decommissioning will take place in 60 – 70
years’ time. However, there is a possibility that it will not take place until 100 – 110 years’ time, in which
case the present value of the costs will be significantly reduced. The following information is disclosed:

A provision of 300 million has been recognised for decommissioning costs. These costs are expected to be
incurred between 2060 and 2070; however, there is a possibility that decommissioning will not take place
until 2100 – 2110. If the costs were measured based upon the expectation that they would not be incurred
until 2100 – 2110 the provision would be reduced to 136 million.

The provision has been estimated using existing technology, at current prices, and discounted using a real
discount rate of 2 per cent.

Example 10.10 (c) – Warranties

A manufacturer gives warranties at the time of sale to purchasers of its product. Under the terms of the
contract for sale, the manufacturer undertakes to make good, by repair or replacement, manufacturing
defects that become apparent within three years from the date of sale. On past experiences, it is
probable (i.e. more likely than not) that there will be some claims under the warranties.

Present obligation as a result of a past obligating event – The obligating event is the sale of the product
with a warranty, which gives rise to a legal obligation.

An outflow of resources embodying economic benefits in settlement – Probable for the warranties as
a whole.

Conclusion – A provision is recognized for the best estimate of the cost of making good under the
warranty produces sold before the end of the reporting period.

215
ACCA Financial Reporting (FR)
CHAPTER 10: IAS 37 PROVISIONS, CONTINGENT LIABILITIES & CONTINGENT ASSETS

Example 10.10 (d) – Contaminated land – Legislation virtually certain to be enacted

An entity in the oil industry causes contamination but cleans up only when required to do so under the
law as of the particular country in which it operates. One country in which it operates has had no
legislation requiring cleaning up, the entity has been contaminating land in that country for several
years. At 31 December 20X0, it is virtually certain that a draft law requiring a clean-up of land already
contaminated will be enacted shortly after the year-end.

Present obligation as a result of a past obligating event – The obligating event is the contamination of
the land because of the virtual certainty of legislation requiring cleaning up.

An outflow of resources embodying economic benefits in settlement – Probable.

Conclusion – A provision is recognized for the best estimate of the cost of clean-up.

Example 10.10 (e) – Contaminated land and constructive obligation

An entity in the oil industry causes contamination and operates in a country where there is no
environmental legislation. However, the entity has a widely published environmental policy in which it
undertakes to clean up all contamination that it causes. The entity has a record of honouring this
published policy.

Present obligation as a result of a past obligating event – The obligating event is the contamination of
the land which gives rise to a constructive obligation because the conduct of the entity has created a
valid expectation on the part of those affected by it that the entity will clean up contamination.

An outflow of resources embodying economic benefits in settlement – Probable.

Conclusion – A provision is recognized for the best estimate of the cost of clean-up.

216
ACCA Financial Reporting (FR)
CHAPTER 10: IAS 37 PROVISIONS, CONTINGENT LIABILITIES & CONTINGENT ASSETS

Example 10.10 (f) – Offshore Oilfield

An entity operates an offshore oilfield where its licensing agreement requires it to remove the oil rig at
the end of production and restore the seabed. 90% of the eventual costs relate to the removal of the oil
rig and restoration of damage caused by building it and 10% arise through the extraction of oil. At the
end of the reporting period, the rig has been constructed but no oil has been extracted.

Present obligation as a result of a past obligating event – The construction of the oil right creates a legal
obligation under the terms of the license to remove the rig and restore the seabed and is thus an
obligating event. At the end of the reporting period, however, there is no obligation to rectify the
damage that will be caused by extraction of the oil.

An outflow of resources embodying economic benefits in settlement – Probable.

Conclusion – A provision is recognized for the best estimate of 90% of the eventual costs that relate to
the removal of the oil rig and restoration of damage caused by building it. These costs are included as
part of the cost of the oil right. The 10% of costs that arise through the extraction of oil are recognised
as a liability when the oil is extracted.

Example 10.10 (g) Closure of a division – no implementation before end of the reporting period

On 12 December 20X0 the board of an entity has decided to close down a division. Before the end of the
reporting period (31 December 20X0) the decision was not communicated to any of those affected and
no other steps were taken to implement the decision.

Present obligation as a result of a past obligating event – There has been no obligating event and so
there is no obligation.

Conclusion – No provision is recognized.

217
ACCA Financial Reporting (FR)
CHAPTER 10: IAS 37 PROVISIONS, CONTINGENT LIABILITIES & CONTINGENT ASSETS

Example 10.10 (h) – Legal requirement to fit smoke filters

Under new legislation an entity is required to fit smoke filters to its factories by 30 June 20X1. The entity
has not fitted the smoke filters.

(a) At 31 December 20X0, the end of the reporting period

Present obligation as a result of a past obligating event – There is still no obligation for the costs of
fitting smoke filters because no obligating event has occurred (the fitting of the filters). However, an
obligation might arise to pay fines or penalties under the legislation because the obligating event has
occurred (the non-compliant operation of the factory)

Conclusion – No provision is recognized for the costs of fitting smoke filters

(b) At 31 December 20X1, the end of the reporting period

Present obligation as a result of a past obligating event – There is still no obligation for the costs of
fitting smoke filters because no obligating event has occurred (the fitting of the filters). However, an
obligation might arise to pay fines or penalties under the legislation because the obligating event has
occurred (the non-compliant operation of the factory).

An outflow of resources embodying economic benefits in settlement – Assessment of probability


of incurring fines and penalties by non-compliant operation depends on the details of the legislation
and the stringency of the enforcement regime.
Conclusion – No provision is recognized for the costs of fitting smoke filters. However, a provision is
recognised for the best estimate of any fines and penalties that are more likely than not to be
imposed.

Example 10.10 (i) – Staff retraining as a result of changes in the income tax system

The government introduces a number of changes to the income tax system. As a result of these changes,
an entity in the financial services sector will need to retrain a large proportion of its administrative and
sales workforce in order to ensure continued compliance with financial services regulation. At the end
of the reporting period, no retraining of staff has taken place.

Present obligation as a result of a past obligating event – There is no obligation because no obligating
event (retraining) has taken place.

Conclusion – No provision is recognised.

218
ACCA Financial Reporting (FR)
CHAPTER 10: IAS 37 PROVISIONS, CONTINGENT LIABILITIES & CONTINGENT ASSETS

Example 10.10 (j) – An onerous contract

An entity operates profitably from a factory that it has leased under an operating lease. During December
20X0 the entity relocated its operations to a new factory. The lease on the old factory continues for the
next four years, it cannot be cancelled and the factory cannot be re-let to another user.

Present obligation as a result of a past obligating event – The obligating event is the signing of the lease
contract, which gives rise to a legal obligation.

An outflow of resources embodying economic benefits in settlement – When the lease becomes
onerous, an outflow of resources embodying economic benefits is probable. (Until the lease becomes
onerous, the entity accounts for the lease under IA 17 Leases)
Conclusion – A provision is recognized for the best estimate of the unavoidable lease payments.

Example 10.10 (k) – Court Case

After a wedding in 20X0, ten people died, possibly as a result of food poisoning from products sold by
the entity. Legal proceedings are started seeking damages from the entity but it disputes liability. Up to
the date of authorization of the financial statements for the year to 31 December 20X0 for issue, the
entity’s lawyers advise that it is probable that the entity will not be found liable. However, when the
entity prepares the financial statements for the year to 31 December 20X1, its lawyers advice that,
owning to developments in the case, it is probable that the entity will be found liable.

(a) At 31 December 20X0

Present obligation as a result of a past obligating event – On the basis of the evidence available
when the financial statements were approved, there is no obligation as a result of past events.
Conclusion – No provision is recognised. The matter is disclosed as a contingent liability unless the
probability of any outflow is regarded as remote.

(b) At 31 December 20X1

Present obligation as a result of a past obligating event – On the basis of the evidence available,
there is a present obligation.
An outflow of resources embodying economic benefits in settlement – Probable

Conclusion – A provision is recognised for the best estimate of the amount to settle the obligation.

219
ACCA Financial Reporting (FR)
CHAPTER 10: IAS 37 PROVISIONS, CONTINGENT LIABILITIES & CONTINGENT ASSETS

Example 10.10 (l) – Repairs and Maintenance

Some asset requires, in addition to routine maintenance, substantial expenditure every few years for
major refits or refurbishment and the replacement of major components IAS 16 Property, Plant and
Equipment gives guidance on allocating expenditure on an asset to is components part where these
components have different useful lives or provide benefits in a different pattern.

Example 10.10 (m)

A furnace has a lining that needs to be replaced every five years for technical reasons. At the end of the
reporting period, the lining has been in use for three years.

Present obligation as a result of a past obligating event – There is no present obligation.

Conclusion – No provision is recognised.

The cost of replacing the lining is not recognised because, at the end of the reporting period, no
obligation to replace the lining exists independently of the company’s future actions – even the intention
to incur the expenditure depends on the company deciding to continue operating the furnace or to
replace the lining. Instead of a provision being recognised, the depreciation of the lining takes account
of the consumption, i.e. it is depreciated over five years. The re-lining costs then incurred are capitalized
with the consumption of each new lining shown by depreciation over the subsequent five years.

Example 10.11 (n) – Refurbishment costs – No legislative requirement

An airline is required by law to overhaul its aircraft once every three years.

Present obligation as a result of a past obligating event – There is no present obligation.

Conclusion – No provision is recognised.

The cost of overhauling aircraft is not recognised as a provision. Even a legal requirement to overhaul
does not make the costs of overhaul a liability, because no requirement exists to overhaul the aircraft
independently of the entity’s future actions – the entity could avoid the future expenditure by its future
actions, for example by selling the aircraft. Instead of a provision being recognised, the depreciation of
the aircraft takes account of the future incidence of maintenance costs, i.e. an amount equivalent to the
expected maintenance costs is depreciated over three years.

220
ACCA Financial Reporting (FR)
CHAPTER 10: IAS 37 PROVISIONS, CONTINGENT LIABILITIES & CONTINGENT ASSETS

10.11 Check Understanding

Question 1
Should a provision be made based on the scenario below?
(i) An oil company have an established practise of always making good any environmental damage
caused by drilling, even though it is not legally obliged to do so.
(ii) Management is planning a maintenance programme for an old office in a years' time. The cost
estimation is $100,000.
Solution:
(i) The established pattern of past practise indicated that the Co will make good of environmental
damage, therefore create a valid expectation. There is a constructive obligation.
A provision should be created based on reliable estimation.

(ii) A provision should not be recognised because there is no obligating event. The intention to incur
maintenance expenditure does not create a legal/constructive obligation. The cost can be avoided if
the entity sells of the office in the future.

Question 2
On 1 October 20X7, Promoil acquired a newly constructed oil platform at a cost of $30 million together
with the right to extract oil from an offshore oilfield under a government licence. The terms of the
licence are that Promoil will have to remove the platform (which will then have no value) and restore
the seabed to an environmentally satisfactory condition in 10 years' time when the oil reserves have
been exhausted. The estimated cost of this in 10 years’ time will be $15 million. The present value of $1
receivable in 10 years at the appropriate discount rate for Promoil of 8% is $0.46.

(a) Explain and quantify how the oil platform should be treated in the financial statements of Promoil
for the year ended 30 September 20X8.

(b) Describe how your answer to (a) would change if the government licence did not require an
environmental clean-up.

221
ACCA Financial Reporting (FR)
CHAPTER 10: IAS 37 PROVISIONS, CONTINGENT LIABILITIES & CONTINGENT ASSETS

Solution:
(a) Promoil must provide for dismantling and restoration costs at 30 September 20X8, as the liability came
into existence with the granting of the license and the cost has been reliably estimated.

Extract of SOPL for the year ended 30 Sep 20X8 $'000


Depn
Finance cost

Extract of SOFP as at 30 Sep 20X8 $'000


NCA:
Cost
- AD
Carrying amount

NCL:
Environmental provision

(b) If the government licence did not require an environmental clean-up, Promoil would have no legal
obligation. It would then be necessary to determine whether or not Promoil had a constructive
obligation. This would apply if on past performance it had established a practice of carrying out an
environmental clean-up where required, which would give rise to the expectation that it would do so
in this case. If a constructive obligation existed, the accounting would be as per the above.

If no obligation were established, there would be no liability. No provision would be made for the
clean-up. The platform would be capitalised at $30m and depreciated over 10 years. There would be
no finance costs.

Extract of SOPL for the year ended 30 Sep 20X8 $'000


Depn

Extract of SOFP as at 30 Sep 20X8


NCA:
Cost
- AD
Carrying amount

222
ACCA Financial Reporting (FR)
CHAPTER 10: IAS 37 PROVISIONS, CONTINGENT LIABILITIES & CONTINGENT ASSETS

Question 3
Raycroft operates a nuclear power station. The power station is due to be decommissioned on 31
December 20X8 but will be fully operational up to that date. It has been estimated that the cost of
decommissioning the power station and cleaning up any environmental damage, as required by
legislation, will be $60 million. Raycroft recognised a provision for the present value of this expenditure
at 31 December 20X0. A suitable discount rate for evaluating costs of this nature is 12%, equivalent to a
present value factor after eight years of 0.404. The decommissioning cost will be depreciated over eight
years.

What is the total charge to profit or loss in respect of this provision for the year ended 31 December 20X1?

A $2,880,800
B $3,030,000
C $5,938,800
D $7,500,000

Question 4

MN obtained a government licence to operate a mine from 1 April 2011. The licence requires that at the
end of the mine’s useful life, all buildings must be removed from the site and the site landscaped. MN
estimates that the cost of this decommissioning work will be $1,000,000 in ten years’ time (present
value at 1 April 2011 $463,000) using a discount factor of 8%.

According to IAS 37 Provisions, Contingent Liabilities and Contingent Assets how much should MN
include in provisions in its statement of financial position as at 31 March 2012?

A $100,000
B $463,000
C $500,000
D $1,000,000

223
ACCA Financial Reporting (FR)
CHAPTER 10: IAS 37 PROVISIONS, CONTINGENT LIABILITIES & CONTINGENT ASSETS

10.12 Chapter 10 Summary


Diagram 10.12.: Summary of IAS 37 Provisions, Contingent Liabilities & Contingent Asset

224
ACCA Financial Reporting (FR)
CHAPTER 11: IAS 10 EVENTS AFTER THE REPORTING PERIOD

CHAPTER 11: IAS 10 EVENTS AFTER THE


REPORTING PERIOD
LEARNING OUTCOME
At the end of the chapter, you should be able to:
TLO B7g. Events after the reporting period:

(i) Distinguish between and account for adjusting and non-adjusting events after
the reporting period
(ii) Identify items requiring separate disclosure, including their accounting
treatment and required disclosures

225
ACCA Financial Reporting (FR)
CHAPTER 11: IAS 10 EVENTS AFTER THE REPORTING PERIOD

11.1 Overview

Financial statements are prepared to the end of the reporting period. However, the preparation of
financial statements, will normally continue for a period after this date. During this time lag, events may
occur which provide additional information that is relevant to the preparation of the financial
statements.

The objective of this Standard is to prescribe:

• When an entity should adjust its financial statements for events after the reporting period; and
• the disclosures that an entity should give about the date when the financial statements were
authorized for issue and about events after the reporting period.

For example, the standard also requires states that entity should not prepare its financial statements on
a going concern basis if events after the reporting period indicate that the going concern assumption is
not appropriate.

11.1.1 Scope

This Standard shall be applied in the accounting for, and disclosure of, events after the reporting period.

11.2 Events after the reporting period

Learning Outcome (ACCA Study Guide Area B)


B7g : Events after the reporting period:
(i) Distinguish between and account for adjusting and non-adjusting events after the reporting
period
Events after the reporting period are those events, favourable and unfavourable, that occur between the
end of the reporting period and the date when the financial statements are authorized for issue.

Note: The date the financial statements are authorised for issue is the key cut off point. Any event which
takes place after this date is outside the scope of IAS 10.

Where an entity is required to submit its financial statements to its shareholders for approval after the
financial statements have been issued, the financial statements are authorised for issue on the date of
issue (not the date when the shareholders approve the financial statements).

226
ACCA Financial Reporting (FR)
CHAPTER 11: IAS 10 EVENTS AFTER THE REPORTING PERIOD

Example:
The management of an entity completes draft financial statement-s for the year to 31 December 20X1 on
28 February 20X2. On 18 March 20X2, the board of directors reviews the financial statements and
authorises them for issue. The entity announces its profit and selected other financial information on 19
March 20X2. The financial statements are made available to shareholders and others on 1 April 20X2. The
shareholders approve the financial statements at their annual meeting on 15 May 20X2 and the approved
financial statements are then filed with a regulatory body on 17 May 20X2.

The financial statements are authorised for issue on 18 March 20X2 (date of board authorisation for
issue)

11.3 Recognition and measurement

Learning Outcome (ACCA Study Guide Area B)


B7g : Events after the reporting period:
(i) Distinguish between and account for adjusting and non-adjusting events after the reporting
period

Two types of events after reporting period can be identified:

Adjusting Events Non-adjusting event

Those that provide evidence of conditions that Those that are indicative of conditions that arose
existed at the end of the reporting period after the reporting period (non-adjusting events
(adjusting events after the reporting period); and after the reporting period).

Requires Adjustment in financial statements by Requires additional disclosure only [if material]

▪ Adjust to amount recognised in financial


statement
▪ Recognise of items previously not recognised

11.4 Adjusting events after the reporting period

An entity shall adjust the amounts recognized in its financial statements to reflect adjusting events after
the reporting period.

The following are examples of adjusting events after the reporting period that require an entity to adjust
the amounts recognized in its financial statements, or to recognize items that were not previously
recognized.

227
ACCA Financial Reporting (FR)
CHAPTER 11: IAS 10 EVENTS AFTER THE REPORTING PERIOD

Example 11.4 (a)

The settlement after the reporting period of a court case that confirms that the entity had a present
obligation at the end of the reporting period. The entity adjusts any previously recognised provision
related to this court case in accordance with IAS 37 or recognises a new provision.

Illustration:
Your co has been sued for wrongful dismissal of a director. You made a provision of $1.2m for the lawsuit
in your FS at 31 Dec 2010, which have not been approved. On Jan 2011, the court confirms that your co
has to compensate $0.8m.

Adjustment: The provision to adjust to $0.8m

Example 11.4 (b)


The receipt of information after the reporting period indicating that an asset was impaired at the end of
the reporting period, or that the amount of a previously recognised impairment loss for that asset needs
to be adjusted.

For example:

(i) The bankruptcy of a customer that occurs after the reporting period usually confirms that a loss
existed at the end of the reporting period on a trade receivable and that the entity needs to adjust
the carrying amount of the trade receivable; and

Illustration:
You have a customer that owes you $500,000 on 31 Dec 2010. On Jan 2011, the customer goes into
liquidation. You’re informed that you will receive nothing from the liquidations.
Adjustment: If FS has not been approved, the RLCA amount in FS as at 31 Dec 2010 should reduce by
$500,000.

(ii) The sale of inventories after the reporting period may give evidence about their net realisable value
at the end of the reporting period.

Illustration:
There were 10,000 units of product A included at cost of $5 each as closing inventory in the FS as at 31
Dec 2010. In Feb 2011, sales of some of these products were made at a price of $4.50 each. Sales staff
receive a commission of 15% of the sales price on this product.

Adjustment: If the FS has not been approved, the closing inventory amount in the FS as at 31 Dec
2010 should reduce by $11,750.

228
ACCA Financial Reporting (FR)
CHAPTER 11: IAS 10 EVENTS AFTER THE REPORTING PERIOD

Example 11.4 (c)


The determination after the reporting period of the cost of assets purchased, or the proceeds from assets
sold, before the end of the reporting period.

Illustration:
Your co sold an operation for $5m on 1 Jan 2010. In addition, your co will receive $1m if the operation
that you sold reaches its profit target for the year to 31 Dec 2010. When preparing your FS to this date,
you’re told that profit target has not been met. Therefore, you produce the FS to reflect the sales proceeds
as $5m. On Jan 2011, you learn that the profit target had been met, and therefore you are owed $1m
more.

Adjustment: If the FS has not been approved, the sales proceeds of the operation sold should increase
by $1m in the FS to 31 Dec 2010.

Example 11.4 (d)


The discovery of fraud or errors that show that the financial statements are incorrect.

Illustration:
You have been preparing the FS for the year ended 31 Dec 2010. On Feb 2011, your auditor identifies
some fictitious sales invoices totaling $800,000. COS has been overstated by $600,000, as part of the
fraud.

Adjustments: If the FS has not been approved, the sales and COS should reduce by $800,000 and
$600,000 respectively in the FS to 31 Dec 2010.

Example 11.4 (e) (Adjusting Events)

A pressing machine with a budgeted carrying amount at 31 December 20X6 of $20,000 is classified as
held for sale in December 20X6. Its fair value less costs to sell is then estimated as $18,000 and it is sold
for $16,500 on 28 February 20X7. The 20X6 financial statements are authorised for issue by the board on
15 March 20X7.
The machine should be measured at $16,500 as a held for sale asset in the 20X6 financial statements.

Point to note:

As the financial statements will have been adjusted for an adjusting event there is no specific requirement
to disclose the event.

However, where the adjusting event affects an item which was not previously recognised but was
disclosed, the disclosure will need to be updated. For example, the contingent liability for damages under
a court case may need to be updated for new information.

229
ACCA Financial Reporting (FR)
CHAPTER 11: IAS 10 EVENTS AFTER THE REPORTING PERIOD

11.5 Non-adjusting events after the reporting period

An entity shall not adjust the amounts recognized in its financial statements to reflect non-adjusting
events after the reporting period.

An event after the RP that is indicative of conditions that arose after the RP. No adjustment to the
amounts recognized in FS, only disclosure required (if material events).

Non-adjusting (NA) events are:


• Announcing a plan to discontinue an operation
• Major purchases and disposals of assets
• The destruction of assets after the reporting date by fire or flood.
• Commencing a court case arising out of events after the reporting date.
• Announcement of major restructuring
• Abnormally large changes in asset prices or foreign exchange rates / tax rate
• Entering into significant guarantees
• Issue of shares after RP
• A decline in the market value of investments

An example of a non-adjusting event after the reporting period is a decline in market value of investments
between the end of the reporting period and the date when the financial statements are authorized for
issue. The decline in market value does not normally relate to the condition of the investments at the end
of the reporting period, but reflects circumstances that have arisen subsequently. Therefore, an entity
does not adjust the amounts recognized in its financial statements for investments. Similarly, the entity
does not update the amounts disclosed for the investments as the end of the reporting period, although
it may need to give additional disclosure.

11.6 Dividends

If an entity declares dividends to holders of equity instruments (as defined in IAS 32 Financial Instruments:
Presentation) after the reporting period, the entity shall not recognized those dividends as a liability at the
end of the reporting period.

If dividends are declared (i.e. the dividends are appropriately authorized and no longer at the discretion
of the entity) after the reporting period but before the financial statements are authorized for issue, the
dividends are not recognized as a liability at the end of the reporting period because they do not meet the
criteria of a present obligation in IAS 37. Such dividends are disclosed in the notes in accordance with IAS
1 Presentation of Financial Statements.

If dividends are declared before the year end, they must be shown in the statement of changes in equity
and accrued for in the statement of financial position.

230
ACCA Financial Reporting (FR)
CHAPTER 11: IAS 10 EVENTS AFTER THE REPORTING PERIOD

11.7 Going Concern

An entity shall not prepare its financial statements on a going concern basis if management determines after
the reporting period either that it intends to liquidate the entity or to cease trading, or that it has no realistic
alternatives but to do so.

Deterioration in operating results and financial position after the reporting period may indicate a need to
consider whether the going concern assumption is still appropriate. If the going concern assumption is no
longer appropriate, the effect is so pervasive that this Standard requires a fundamental change in the
basis of accounting, rather than an adjustment of the amounts recognized within the original basis of
accounting.

IAS 1 specifies required disclosure if:

• The financial statements are not prepared on a going concern basis; or

• Management is aware of material uncertainties related to events or conditions that may cast
significant doubt upon the entity’s ability to continue as a going concern. The events or conditions
requiring disclosure may arise after the reporting period.

Example 11.7 (a) Building Defects


A routine inspection of an entity's main freehold building two weeks after its year end of 30 June 20X8
and before the accounts were authorized for issue revealed substantial cracks in the walls. A more
detailed review was immediately undertaken by specialist professionals, who reported that there were
major problems with the foundations. In their view these problems must have arisen several years ago,
even though the visible evidence had only now come to light.

Required: Explain how this event should be dealt with in the financial statements for the year ended
30 June 20X8.

Solution:
The cracks in the walls are clear evidence of a change in the building’s condition.
The specialist professionals have provided their opinion that the problems must have arisen several years
ago (not since the yearend), thus providing evidence as to the building’s condition at the end of the
reporting period.
The draft financial statements should be adjusted to take account of this change in condition. A full
impairment review under IAS 36 Impairment of Assets should be carried out and any impairment loss
should be recognised in the June 20X8 financial statements.

Point to note:
It is highly likely that expenditure on repairs will be needed in the 30 June 20X9 financial year to rectify
the damage and that the cost may be significant. But no provision for these repairs should be recognised
at 30 June 20X8. There is a past event (the faults now identified) but at the end of the reporting period
there is no obligation to incur the expenditure on repairs; so no provision should be recognised.

231
ACCA Financial Reporting (FR)
CHAPTER 11: IAS 10 EVENTS AFTER THE REPORTING PERIOD

Example 11.7 (b) Inventories


At its year end of 30 June 20X8 an entity held in inventories 4,000 units of a particular product line at a
cost of $550 each. The product had been selling well, at $750 each with selling costs of $100 each.
Early in its new financial year the entity learnt that competitor action was such that it could only sell its
product for $605, with selling costs unchanged.

Required: Explain how this event should be dealt with in the financial statements for the year ended
30 June 20X8.

Solution:

11.8 Disclosure

Learning Outcome (ACCA Study Guide Area B)


B7g : Events after the reporting period:
(ii) Identify items requiring separate disclosure, including their accounting treatment and
required disclosures

11.8.1 Date of authorisation for issues

An entity shall disclosure the date when the financial statements were authorized for issue and who gave
that authorization. If the entity’s owners or others have the power to amend the financial statements
after issue, the entity shall disclose that fact.

It is important for users to know when the financial statements were authorized for issue, because the
financial statements do not reflect events after this date.

11.8.2 Non-adjusting events after the reporting period

If non-adjusting events after the reporting period are material, non-disclosure could influence the
economic decisions that users make on the basis of the financial statements. Accordingly, an entity shall
disclose the following for each material category of non-adjusting event after the reporting period:

- The nature of the event; and


- An estimate of its financial effect, or a statement that such an estimate cannot be made.

232
ACCA Financial Reporting (FR)
CHAPTER 11: IAS 10 EVENTS AFTER THE REPORTING PERIOD

11.9 Check Understanding

Question 1
Britney’s current year end is 31 March 2009. Its financial statements were authorised for issue by its
directors on 6 May 2009 and the AGM will be held on 3 June 2009. The following matters have been
brought to your attention:

(i) On 12 April 2009 a fire completely destroyed the company’s largest warehouse and the inventory
it contained. The carrying amounts of the warehouse and the inventory were $10 million and $6
million respectively. Britney’s trading operations have been severely disrupted since the fire and
it expects large trading losses for some time to come.

(ii) A single class of inventory held at another warehouse was valued at its cost of $460,000 at 31
March 2009. In April 2009 this inventory was sold for $480,000 on which Britneys’ sales staff
earned a commission of 15% of the selling price.

(iii) The notification of the bankruptcy of a customer at 30 April 2009. The balance of the trade
receivables due from the customer at 31 March 2009 was $23,000 and at the date of the
notification it was $25,000. No payment is expected from the bankruptcy proceedings.

(iv) Britney is being sued by an employee who lost a limb in an accident while at work on 15 March
2009. The company is contesting the claim as the employee was not following the safety
procedures that he had been instructed to use. Accordingly, the financial statements include a
note of a contingent liability of $500,000 for personal injury damages. In a recently decided case
where a similar injury was sustained, a settlement figure of $750,000 was awarded by the court.
Although the injury was similar, the circumstances of the accident in the decided case are
different from those of Britney's case.

Explain the required treatment of the above items by Britney in its financial statements for the year
ended 31 March 2009.

233
ACCA Financial Reporting (FR)
CHAPTER 11: IAS 10 EVENTS AFTER THE REPORTING PERIOD

Solution:

(i) This is a non-adjusting event as there was no reason to doubt that the value of warehouse and
the inventory it contained was worth less than its carrying amount at 31 March 2009. The total
loss suffered as a result of the fire is $16 million. IAS 10 would require the details of this non -
adjusting event to be disclosed as a note to the FS for the year ended 31 March 2009 as a total
loss of $16 million.

The severe disruption in Waxwork’s trading operations since the fire, together with the
expectation of large trading losses in the future, may raise the concern about the going concern
status of the company. If it is judged that Waxwork is no longer a going concern, then the fire
and its consequences become an adjusting event requiring the financial statements for the year
ended 31 March 2009 to be redrafted on the basis that the company is no longer a going
concern (i.e. they would be prepared on a liquidation basis).

(ii) Net realisable value (NRV) = $408,000 (480,000 x 85%)

This would appear to give evidence of conditions that existed at 31 March 2009 (ie: the NRV of
that class of inventory was below its cost). Based on IAS 2, Inventory is required to be valued at
the lower of cost and NRV, thus this is an adjusting event. Thus, the carrying amount of the
inventory of $460,000 should be written down by $52,000 to its NRV of $408,000.

(iii) This is an adjusting event after the reporting period within the terms of IAS 10. $23,000 was
existed at 31 March 2009, thus it should be written off to irrecoverable debts at the year end and
the trade receivables balance correspondingly reduced.

(iv) As it is not yet known whether the employee's legal action will be successful, it is correct to show
it as a contingent liability. If the case is settled before the FS are authorised for issue, this will be
an adjusting event requiring a provision for damages if Britney is found liable. However, the
circumstances of the claim are different from those of the recently settled case. So, this
settlement does not appear to have any effect on the likelihood of Britney losing the case. But, it
does affect the estimation amount of the liability. The contingent liability should be increased to
$750,000.

234
ACCA Financial Reporting (FR)
CHAPTER 11: IAS 10 EVENTS AFTER THE REPORTING PERIOD

Question 2
Triangle, a public listed company, is in the process of preparing its draft financial statements for the year
to 31 March 20X5. The following matters have been brought to your attention:

(i) On 1 April 20X4 the company brought into use a new processing plant that had cost $15 million
to construct and had an estimated life of ten years. The plant uses hazardous chemicals which are
put in containers and shipped abroad for safe disposal after processing. The chemicals have also
contaminated the plant itself which occurred as soon as the plant was used. It is a legal
requirement that the plant is decontaminated at the end of its life. The estimated present value
of this decontamination, using a discount rate of 8% per annum, is $5 million. The financial
statements have been charged with $1.5 million ($15 million/10 years) for plant depreciation and
a provision of $500,000 ($5 million/10 years) has been made towards the cost of the
decontamination.

(ii) On 15 May 20X5 the company's auditors discovered a fraud in the material requisitions
department. A senior member of staff who took up employment with Triangle in August 20X4 had
been authorising payments for goods that had never been received. The payments were made to
a fictitious company that cannot be traced. The member of staff was immediately dismissed.
Calculations show that the total amount of the fraud to the date of its discovery was $240,000 of
which $210,000 related to the year to 31 March 20X5. (Assume the fraud is material).

(iii) The company has contacted its insurers in respect of the above fraud. Triangle is insured for theft,
but the insurance company maintains that this is a commercial fraud and is not covered by the
theft clause in the insurance policy. Triangle has not yet had an opinion from its lawyers. (4 marks)

Explain how the items in (i) to (iii) above should be treated in Triangle's financial statements for the
year to 31 March 20X5 in accordance with current international accounting standards. Your answer
should quantify the amounts where possible.

235
ACCA Financial Reporting (FR)
CHAPTER 11: IAS 10 EVENTS AFTER THE REPORTING PERIOD

Solution:

(i) Contamination
• The obligation to clean up the contamination existed in full from the day that the plant was
brought into use. Therefore, the provision should be recognised in full immediately at PV
• It should not be accrued incrementally over the life of the plant.
• On initial recognition, cost of the plant should include the PV of the decontamination
• The PV will increase as the discount unwinds (Dr finance cost, Cr provision)

The plant and the provision should be reported as follows:

Extract of SOPL for the year ended 31 March 20X5


$000
Depreciation 2,000
Finance cost 400

Extract of SOFP as at 31 March 20X5


NCA $000
Cost 20,000
(-) AD (2,000)
Carrying amount 18,000

NCL
Provision for decontamination (5000 +400) = 5,400

(ii) Fraud
• The fraud means that the draft FS for the year-ended 31 March 20X5 are incorrect
• The discovery of this fraud provides new evidence about conditions existing at the end of the
reporting period, and so it is classified as an adjusting event.
• The $210,000 fraud that occurred during the year will be charged to profit or loss as an
operating expense and disclosed.
• The $30,000 fraud occurring after the year-end does not affect conditions existing at the end
of the reporting period and so it will not be adjusted for. However, it will be disclosed if it is
considered to be material in its own right.

(iii) Insurance claim


• The insurance claim gives rise to a contingent asset for $240,000.
• According to IAS 37, contingent assets are not recognised unless the realisation of income is
virtually certain
• The insurance claim here is not virtually certain because the insurers are disputing the claim.
• A contingent asset is disclosed when an inflow of economic benefits is probable, but without
legal opinion it is not possible to regard the success of the claim as probable.
• Following on from the above, the insurance claim should be ignored altogether in the
financial statements for the year-ending 31 March 20X5.

236
ACCA Financial Reporting (FR)
CHAPTER 11: IAS 10 EVENTS AFTER THE REPORTING PERIOD

11.10 Chapter 11 Summary

Diagram 11.10.: Summary of IAS 10 Events after the Reporting Period

237
ACCA Financial Reporting (FR)
CHAPTER 12: IAS 8 ACCOUNTING POLICIES, CHANGES IN ACCOUNTING ESTIMATES
AND ERRORS

CHAPTER 12: IAS 8 ACCOUNTING


POLICIES, CHANGES IN ACCOUNTING
ESTIMATES & ERRORS
LEARNING OUTCOME
At the end of the chapter, you should be able to:

TLO B9d. Account for changes in accounting estimates, changes in accounting policy and
correction of prior period errors.

238
ACCA Financial Reporting (FR)
CHAPTER 12: IAS 8 ACCOUNTING POLICIES, CHANGES IN ACCOUNTING ESTIMATES
AND ERRORS

12.1 Overview

IAS 8 prescribes the criteria for:

• Selecting and changing Accounting Policies (AP)


• Disclosure of changes in Accounting Policies
• Changes in Accounting Estimates (AE)
• Corrections of prior period errors.

This enhance the relevance, faithful representation and the comparability of financial information that is
presented by the entities to its wide range of users.

This Standard shall be applied in selecting and applying AP, and accounting for changes in AP, changes in
AE and corrections of prior period errors.

The tax effects of corrections of prior period errors and of retrospective adjustments made to apply
changes in accounting policies are accounted for and disclosed in accordance with IAS 12 Income Taxes.

239
ACCA Financial Reporting (FR)
CHAPTER 12: IAS 8 ACCOUNTING POLICIES, CHANGES IN ACCOUNTING ESTIMATES
AND ERRORS

12.2 Accounting Policies

12.2.1 Definition
15.1.49

Accounting Policies are the specific principles, bases, conventions, rules and practices applied by an
entity in preparing and presenting Financial Statement.

12.2.2 Selection of Accounting Policies


15.1.50
Are there any existing IAS / IFRS?
YES NO
The AP applied to that item shall be determined Management shall use its judgement in applying AP that results
by applying the relevant IFRS in relevant & reliable info by referring to:
I. Requirements and guidance in IFRSs dealing with similar &
related issues

II. The definition, recognition criteria & measurement


concepts for Assets, Liabilities, Income & Expenses in the
Conceptual Framework 2018

III. Consider most recent pronouncements of other standard


setting bodies, other accounting literature and accepted
industry practices, where these do not conflict with IFRS

240
ACCA Financial Reporting (FR)
CHAPTER 12: IAS 8 ACCOUNTING POLICIES, CHANGES IN ACCOUNTING ESTIMATES
AND ERRORS

12.3 Consistency of Accounting Policies

Learning Outcome (ACCA Study Guide Area B)


B9d : Account for changes in accounting estimates, changes in accounting policy and correction of
prior period errors.

An entity shall select and apply its accounting policies consistently for similar transaction, other events
and condition, unless an IFRS specifically requires or permits categorization of items for which different
policies may be appropriate. If an IFRS requires or permits such categorization, an appropriate accounting
policy shall be selected and applied consistently to each category.

12.3.1 Changes in Accounting Policies


15.1.51
General rule: AP should apply consistently for similar transactions to ensure comparability

An entity shall change an Accounting Policy only if the change:

• is required by an IFRS
• results in the financial statements providing reliable and more relevant information about the
effects of transactions, other events or conditions on the entity’s financial position, financial
performance or cash flows.

12.3.2 Applying Changes in Accounting Policies


15.1.52

When there is a change in accounting policy under IAS 8, it shall apply the change retrospectively, the
entity shall adjust the opening balance of each affected component of equity for the earliest prior period
presented and the other comparative amounts disclosed for each prior period presented as if the new
accounting policy had always been applied.

Applying changes in Accounting Policies


If required by an IFRS Voluntary change

• Refer to transitional provisions in new IFRS, in Apply the changes retrospectively


which case IAS does not apply
• If IFRS does not specify transitional provision,
apply the changes retrospectively

241
ACCA Financial Reporting (FR)
CHAPTER 12: IAS 8 ACCOUNTING POLICIES, CHANGES IN ACCOUNTING ESTIMATES
AND ERRORS

Retrospective application

Applying a new AP to transactions, other events and conditions as if that policy had always been applied,
by adjusting in both current and previous accounting periods:

▪ shall adjust the opening balance of each affected component of equity for the earliest prior period
presented (Statement of Changes in Equity), and
▪ the other comparative amounts disclosed for each prior period presented

Retrospective restatement is correcting the recognition, measurement and disclosure of amounts of


elements of FS as if a prior period error had never occurred.
The Standard refers to limitations on retrospective application when it proves impracticable.

Limitations on Retrospective Application

If it is impracticable to determine the cumulative effect of the change, the change should be applied
prospectively (only adjust in current period).

Impracticable means the entity cannot apply it after making every reasonable effort to do so.

What do NOT constitute as changes in Accounting Policies ?

• the application of an accounting policy for transactions, other events or conditions that differ in
substance from those previously occurring; and

• the application of a new accounting policy for transactions, other events or conditions that did
not occur previously or were immaterial

Example 12.3.3 (a) (differ in substance)

A construction company has been engaged in small construction contracts which take 2-3
years to complete and has adopted the "completed contract" method of accounting. In the
current year, the company enters into a large construction contract that is expected to take
8 years to complete and decides to use the ‘% of completion’ method of accounting for this
contract. This is not considered a change in accounting policy.

Example 12.3.3 (b) (new transaction)

The company is using leased asset for the first time and apply IFRS16 - Leases. This is not a
change of policy.

242
ACCA Financial Reporting (FR)
CHAPTER 12: IAS 8 ACCOUNTING POLICIES, CHANGES IN ACCOUNTING ESTIMATES
AND ERRORS

Disclosure

When initial application of an IFRS and a voluntary change in accounting policy has an effect on the current
period or any prior period, an entity shall disclose;

In the Notes to the FS:

1) IF REQUIRED BY STANDARD

(a) title of standard


(b) change in AP is made in accordance with its transitional provisions
(c) nature of the change in AP
(d) description of the transitional provisions
(e) how transitional provisions have an effect on future periods
(f) the amount of adjustment for the current & each prior period presented for each FS line affected
(g) the amount of the adjustment relating to periods before those presented
(h) Circumstances that led to existence of that condition and description of how and from when the
change in AP has been applied if retrospective application is impracticable

2) Voluntary change

(a) nature of change in AP


(b) reasons why applying the new AP provides reliable and more relevant info
(c) the amount of adjustment for the current & each prior period presented for each FS line affected
(d) amount of the adjustment relating to periods before those presented
(e) Circumstances that led to existence of that condition and description of how and from when the
change in AP has been applied if retrospective application is impracticable

When an entity has not applied a new Standard that has been issued but is not yet effective, the entity
shall disclose:

(a) this fact; and

(b) known or reasonably estimable information relevant to assessing possible impact that application of
new Standard on FS in the period of initial application.

243
ACCA Financial Reporting (FR)
CHAPTER 12: IAS 8 ACCOUNTING POLICIES, CHANGES IN ACCOUNTING ESTIMATES
AND ERRORS

12.4 Changes in Accounting Estimates

Learning Outcome (ACCA Study Guide Area B)


B9d : Account for changes in accounting estimates, changes in accounting policy and correction of
prior period errors.

12.4.1 Definition of Accounting Estimates


An15.1.53
adjustment of the carrying amount of an asset or liability, or related expense or the amount of the
periodic consumption of an asset, resulting from reassessing the present status of expected future
benefits and obligations associated with the asset or liability.

Note:

Changes in Accounting Estimate result from new information or new developments and, accordingly, are
not corrections of errors.

Because of the uncertainties inherent in business activities, many items in Financial Statements cannot be
measured with precision but can only be estimated. Estimation involves judgements based on the latest
available, reliable information. For example, estimates may be required of:

(a) bad debts


(b) inventory obsolescence
(c) the Fair Value of financial assets or financial liabilities
(d) the Useful Lives, or expected consumption pattern of future economic benefits. embodied in
depreciable assets
(e) warranty obligations

Table 12.4.1 Accounting treatment of changes in AE


The change shall be recognised prospectively in profit and loss in the:

Period of change Period of change and future periods


if the change affects that period only if the change affects both

12.4.2 Disclosure for Accounting Estimate

In the notes to Financial statement:

An entity shall disclose the nature and amount of a change in accounting estimate that has an effect in
the current period or is expected to have an effect in future periods except for the disclosure of the effect
on future periods when it is impracticable to estimate that effect.

If the amount of the effect in future periods is not disclosed because estimating it is impracticable, an
entity shall disclose that fact.

244
ACCA Financial Reporting (FR)
CHAPTER 12: IAS 8 ACCOUNTING POLICIES, CHANGES IN ACCOUNTING ESTIMATES
AND ERRORS

12.4.3 How to differentiate between a change in AP & AE


A change in AP has occurred when there has been a change to any one of the following:

i. Recognition criteria
ii. Measurement basis
iii. Presentation
If the above is NOT affected, then it would be treated as a change in AE.

When it is difficult to distinguish a change in an AP from a change in an AE, the change is treated as a
change in an AE.

Exercise 12.4.3 (a)


An entity has previously charged interest incurred in connection with the construction of tangible non-
current assets to the income statement. Following the revision of IAS 23, and in accordance with the
revised requirements of that standard, it now capitalises this interest.

Solution:
Affect - recg - interest cost become part of the assets
- presentation - now in SOFP not in P/L
Therefore, this is a change in _____________
Exercise 12.4.3 (b)
An entity has previously depreciated vehicles using the reducing balance method at 40% pa. It now uses
the straight-line method over a period of five years.

Solution:
Does not affect - recg - still @ cost less Acc depn
Meas - still based on consumption pattern of eco benefits
presentation - still in P/L (exp) & SOFP (NBV)

Therefore, this is a change in ______________


Exercise 12.4.3 (c)
An entity has previously shown certain overheads within cost of sales. It now shows those overheads
within administrative expenses.

Solution:
Affect -
Therefore, this is a change in _______________
Exercise 12.4.3 (d)
An entity has previously measured inventory at weighted average cost. It now measures inventory using
the first in first out (FIFO) method.

Solution:
Affect -
Therefore, this is a change in _______________

245
ACCA Financial Reporting (FR)
CHAPTER 12: IAS 8 ACCOUNTING POLICIES, CHANGES IN ACCOUNTING ESTIMATES
AND ERRORS

12.5 Prior Period Errors

Learning Outcome (ACCA Study Guide Area B)


B9d : Account for changes in accounting estimates, changes in accounting policy and correction of
prior period errors.

12.5.1 Definition

Prior period errors which are omissions from, and misstatements in, an entity’s FS for one or more prior
periods arising from failure to use/or from misuse of reliable information:

a. that was available when the Financial Statement for that period were issued; and
b. could have been reasonably expected to be taken into account in preparation of Financial Statemen t

Material Omissions or misstatements are items which are material if omitting, misstating or obscuring it
could reasonably be expected to influence decisions that the primary users of general purpose financial
statements make on the basis of those financial statements, which provide financial information about a
specific reporting entity.

For instance, computation errors, material manipulation of figures, errors in application of AP, fraud etc.

Current period errors discovered in that period should be corrected before Financial Statement are
authorised for issue.

246
ACCA Financial Reporting (FR)
CHAPTER 12: IAS 8 ACCOUNTING POLICIES, CHANGES IN ACCOUNTING ESTIMATES
AND ERRORS

12.5.2 Accounting treatment

Is it material?

Yes No
Corrected retrospectively Corrected in current Reporting Period

Restating the comparative amount for prior period(s) in which


error occurred

Restating the opening balance of Assets, Liabilities and Equities


for earliest prior period presented if the error occurred before
that date

If it is impracticable to determine the cumulative effect of a prior periods error, an entity can correct the
error prospectively.

12.5.3 Disclosure

In the Notes to Financial Statement:

An entity shall disclose the following:

a. the nature of the prior period error;


b. the amount of the correction at the beginning of the earliest prior period presented; and
c. if retrospective restatement is impracticable for a particular prior period, the circumstances that led
to the existence of that condition and a description of how and from when the error has been
corrected

Applicable to all disclosures – Financial Statement of subsequent periods need not repeat these
disclosures.

247
ACCA Financial Reporting (FR)
CHAPTER 12: IAS 8 ACCOUNTING POLICIES, CHANGES IN ACCOUNTING ESTIMATES
AND ERRORS

12.6 Check Understanding

Question 1
Change Co changed its accounting policy in 2010 with respect to valuation of inventories. Up to 2009,
inventories were valued using AVCO method. In 2010, the method was changed to FIFO as it was
considered to more accurately reflect the usage and flow to inventories in the economic cycle. The
impact on inventory valuation was determined to be:

At 31 Dec 2008 An increase of $10,000


At 31 Dec 2009 An increase of $15,000
At 31 Dec 2010 An increase of $20,000

SOPL prior to adjustment 2010 ($) 2009 ($)


Rev 250,000 200,000
COS (100,000) (80,000)
GP 150,000 120,000
DC (60,000) (50,000)
AE (25,000) (15,000)
NP 65,000 55,000

Present the change in the accounting policy in the SOPL and the SOCIE in accordance with IAS 8.
Solution:
2008 2009 2010
yrs b4 immediate prior prior period current period
period
31.12.2008 31.12.2009 new 31.12.2010
AP

adj op RE in SOCIE restate comparative adj current year SOPL


in previous year SOPL

248
ACCA Financial Reporting (FR)
CHAPTER 12: IAS 8 ACCOUNTING POLICIES, CHANGES IN ACCOUNTING ESTIMATES
AND ERRORS

SOPL for the Y/E 31.12.2010 Comparative


2010 2009 (restated)
$ $
Sales revenue
Cost of sales
Gross profit
Distribution costs
Administrative expenses
Profit for the period

SOCIE for the Y/E 31.12.2010


RE ($)
Bal at 1 Jan 2009 (old AP) x
Changes in AP
Restated balance (new AP)
Changes in equity for 2009
Dividends (x)
TCI for the year (new AP)
Bal at 31 Dec 2009 xx
Changes in equity for 2010
Dividends (x)
TCI for the year (new AP)
Bal at 31 Dec 2010 xxx

249
ACCA Financial Reporting (FR)
CHAPTER 12: IAS 8 ACCOUNTING POLICIES, CHANGES IN ACCOUNTING ESTIMATES
AND ERRORS

Question 2

The directors of Tunshill are disappointed by the draft profit for the year ended 30/9/2010. The Co’s
assistant accountant has suggested two areas where she believes the reported profit may be improved:

i. A major item of plant that cost $20 million to purchase and install on 1/10/2007 is being depreciated
on a SLM over a 5-year period (assuming no residual value). The plant is wearing well and at the beg
of the current year (1 October 2009) the production manager believed that the plant was likely to
last 8 years in total (i.e. from the date of its pr). The assistant accountant has calculated that, based
on an 8-year life (and no residual value) the AD of the plant at 30/9/2010 would be $7·5 million ($20
million/8 years’ x 3). In the FS for the year ended 30/9/2009, the AD was $8 million ($20 million/5
years’ x 2). Therefore, by adopting an eight-year life, Tunshill can avoid a depreciation charge in the
current year and instead credit $0·5 million ($8 million – $7·5 million) to the SOPL in the current year
to improve the reported profit.

Comment on the acceptability of the assistant accountant’s suggestions and quantify how they would
affect the FS if they were implemented under IFRS. Ignore taxation.

Solution Question (i):

The estimate of the useful life of a non-current asset form an important part of the accounting estimate
of the depreciation charge. Like most estimates, an annual review of their appropriateness is required
and it is usual, as in this case, to revise the estimate of the remaining useful life of plant.

This appears to be an acceptable reason for a revision of the plant’s life if it is based on genuine
reassessment by the production manager, whereas it would be unacceptable to increase the estimate
simply to improve the company’s reported profit.

However, the assistant accountant’s calculation of the financial effect of the revised life is incorrect.
Where there is an increase (or decrease) in the estimated remaining life of a non-current asset, its
carrying amount (at the time of the revision) is allocated over the new remaining life (after allowing for
any estimated residual value).

Revised depreciation = NBV / RUL

Extract of SOPL fye 30/9/2010 Extract of SOFP as at 30/9/2010


Expenses: Depreciation NCA: Net Book Value

A depreciation charge for the current year cannot be avoided and there will be no credit to the SOPL as
suggested by the assistant accountant. It should be noted that the incremental effect of the revision to
the estimated life of the plant would be to improve the reported profit by $2 million being the
difference between the depreciation based on the old life ($4 million) and the new life ($2 million).

250
ACCA Financial Reporting (FR)
CHAPTER 12: IAS 8 ACCOUNTING POLICIES, CHANGES IN ACCOUNTING ESTIMATES
AND ERRORS

ii. Most of Tunshill’s competitors value their inventory using the AVCO basis, whereas Tunshill uses the
FIFO basis. The value of Tunshill’s inventory at 30 September 2010 (on the FIFO basis) is $20 million,
however on the AVCO basis it would be valued at $18 million. By adopting the same method (AVCO)
as its competitors, the assistant accountant says the Co would improve its profit for the year ended
30 September 2010 by $2 million. Tunshill’s inventory at 30 September 2009 was reported as $15
million, however on the AVCO basis it would have been reported as $13·4 million.

Comment on the acceptability of the assistant accountant’s suggestions and quantify how they would
affect the FS if they were implemented under IFRS. Ignore taxation.

Solution Question (ii):

Whilst both methods (FIFO and AVCO) are acceptable methods of valuing inventory under IAS 2
Inventories, changing an accounting policy to be consistent with that of competitors is not a convincing
reason.

Generally, changes in accounting policies should be avoided unless a change is required by a new or
revised accounting standard or the new policy provides more reliable and relevant information
regarding the entity’s position.

In any event the assistant accountant’s calculations are again incorrect and would not meet the
intention of improving reported profit. The most obvious error is that changing from FIFO to AVCO will
cause a reduction in the value of the closing inventory at 30 September 2010 effectively reducing, rather
than increasing, both the valuation of inventory and reported profit.

A change in accounting policy must be accounted for as if the new policy had always been in place
(retrospective application).

In 30/9/2010

If both the opening and closing inventories would need to be measured at AVCO, impact to profit:

In 30/9/2009 (restated comparative figure for 2009)

If closing inventories would need to be measured at AVCO, impact to profit:

251
ACCA Financial Reporting (FR)
CHAPTER 12: IAS 8 ACCOUNTING POLICIES, CHANGES IN ACCOUNTING ESTIMATES
AND ERRORS

Question 3 (Extract of Dec’07)

Emerald has had a policy of writing off development expenditure to the SOPL as it was incurred. In
preparing its financial statements for the year ended 30 September 2007 it has become aware that,
under IFRS rules, qualifying development expenditure should be treated as an intangible asset. Below is
the qualifying development expenditure for Emerald:
$’000
Year ended 30 September 2004 300
Year ended 30 September 2005 240
Year ended 30 September 2006 800
Year ended 30 September 2007 400

All capitalised development expenditure is deemed to have a four-year life. Assume amortisation
commences at the beginning of the accounting period following capitalisation. Emerald had no
development expenditure before that for the year ended 30 September 2004.

Treating the above as the correction of an error in applying an accounting policy.

Show the extract of SOPL & SOFP (including comparative figures), and the SOCIE in respect of the
development expenditure for the year ended 30 September 20X7.

Solution:

20x5 20x6 20x7


yrs b4 immediate prior prior period current period
period
old AP 30.9.x5 old AP 30.9.x6 new AP 30.9.x7

Wrong treatment

Correction

Op RE in SOCIE restate comparative adj current year SOPL


in previous year SOPL

252
ACCA Financial Reporting (FR)
CHAPTER 12: IAS 8 ACCOUNTING POLICIES, CHANGES IN ACCOUNTING ESTIMATES
AND ERRORS

Extract of SOPL for the Y/E 30 Sep 20X7 20X6 (comparative)


Expenses:
Amortisation of devl expd

Extract of SOFP as at 30 Sep 20X7 20X6 (comparative)


NCA
Capitalised devl costs
- Acc amortisation
Carrying amount

Extract of SOCIE for the Y/E 31 Sept 20X7 RE ($)


Bal at 1 Oct X5 (as previously stated)
Correction of error
Restated balance (new AP)

253
ACCA Financial Reporting (FR)
CHAPTER 12: IAS 8 ACCOUNTING POLICIES, CHANGES IN ACCOUNTING ESTIMATES AND ERRORS

12.7 Chapter 12 Summary

Diagram 12.7.: Summary of IAS 8 Accounting Policies, changes in Accounting Estimates and Errors

254
ACCA Financial Reporting (FR)
CHAPTER 13: IAS 33 EARNINGS PER SHARE

CHAPTER 13: IAS 33 EARNINGS PER


SHARE

LEARNING OUTCOME
At the end of the chapter, you should be able to:

TLO B9e. Earnings per share (EPS)


(i) Calculate the EPS in accordance with relevant accounting standards (dealing with
bonus issues, full market value issues and rights issues)

(ii) Explain the relevance of the diluted eps and calculate the diluted eps involving
convertible debt and share options

255
ACCA Financial Reporting (FR)
CHAPTER 13: IAS 33 EARNINGS PER SHARE

13.1 Overview

One of the most commonly used performance measures worldwide is basic earnings per share (EPS),
which is calculated as the profit or loss attributable to the ordinary equity holders divided by the
number of shares in issue.

In addition to being an important independent measure, it also is a component in the price earnings
(P/E) ratio which often forms a pivotal role in the valuation of businesses. A meaningful comparison
between entities, or against a benchmark figure, can only be made where entities measure their EPS
figure on a consistent basis. IAS 33 prescribes what that consistent basis should be.

Standard EPS calculations assist in comparisons which are meaningful across entities, but they take
account of all income and expenses that have been reported during the period, whether or not they
are likely to recur in the future. These calculations provide a historical performance measure and do
not purport to provide a measure of future performance. So entities frequently present alternative
forms of EPS, based on income and expenses which have been adjusted to exclude non-recurring
items; entities generally refer to the adjusted profit figure as ‘maintainable earnings’. Industry or
market standard EPS figures are also often reported. Both of these additional performance measures
are claimed to provide a more realistic measure of the entity’s performance in future periods.

Compliance with IAS 33 is mandatory for:

• The separate financial statements of entities whose ordinary shares are publicly traded or are in
the process of being issued in public markets.
• The consolidated financial statements for groups whose parent has shares similarly traded/being
issued.

Other entities need not present EPS (because their shares are not traded, there is no readily available
market price which can be used to calculate the P/E ratio), but if they do voluntarily, they should
comply with IAS 33.

IAS 33 requires the EPS to be presented in the statement of profit or loss

256
ACCA Financial Reporting (FR)
CHAPTER 13: IAS 33 EARNINGS PER SHARE

13.2 Needs for IAS 33

EPS is widely regarded as the most important indicator of a Co performance. It is used to compute the
major stock market indicator of performance (P/E ratio).
It is important for the users of the FS are able to compare the EPS of:

Diff entities the same entity in diff accounting periods

Therefore, IAS 33 achieves comparability by:


i defining earnings
ii. prescribing methods for determining the no of shares to be included in calculation of EPS
iii. requiring standard presentation & disclosures

13.2.1 Definition for EPS


15.1.54
A measure of the amount of profits earned by a company for each ordinary share. Earnings are
profit after tax and preference dividends.

13.3 Basic EPS

Learning Outcome (ACCA Study Guide Area B)


B9e : Earnings per share (EPS)
(i) Calculate the eps in accordance with relevant accounting standards (dealing with bonus
issues, full market value issues and rights issues)

Earnings
Weighted average number of Ordinary shares (WANOS) outstanding during the
period

13.3.1 Computation of earnings


15.1.55

= Net Profit - Irredeemable preference dividend

Redeemable preference dividend will already have been treated as finance cost

257
ACCA Financial Reporting (FR)
CHAPTER 13: IAS 33 EARNINGS PER SHARE

Exercise 13.3:
Flame Co is a company with a called up and paid up capital of 100,000 ordinary shares of $1 each
and 20,000 10% redeemable preference shares of $1 each.

The gross profit was $200,000 and trading expenses were $50,000. Flame Co paid the required
preference share dividend and an ordinary dividend of 42c per share. The tax charge for the year was
estimated at $40,000.

Calculate basic EPS for the year.

Earnings = EPS=

13.3.2 Computation of WANOS outstanding during the period


15.1.56
When there are any changes to the number of Ordinary Shares outstanding (when there is a cash
inflow/outflow)

Example 13.3.2:
Justina Co, a listed company, has the following share transactions during 20X7.

Date Details Shares issued


1 January 20X7 Balance at beginning of year 170000
31 May 20X7 Issue of new shares for cash 80000
31 December 20X7 Balance at year end 250000

The weighted average number of shares will be:


Solution:

258
ACCA Financial Reporting (FR)
CHAPTER 13: IAS 33 EARNINGS PER SHARE

13.4 Situation involving a Change in the number of Ordinary Shares outstanding during
the year

13.4.1 Fresh issue


15.1.57
- Shares are issue at full market price
- Inflow of cash from the date of issue (inflows helps in generating income only for the Outstanding
period) therefore, no of shares (denominator) must be calculate on pro-rata basis
- Not need to restate previous year EPS as there was no fresh issue at that time

Exercise 13.4.1 (fresh issue):

On 30 September 20X2, Boffin Co made an issue at full market price of 1,000,000 ordinary shares. The
Co's accounting year runs from 1 January to 31 December. Relevant information for 20X1 and 20X2 is
as follows.:

20X2 20X1
Shares in issue as at 31 December 9,000,000 8,000,000
Profits after tax and preference dividend 3,300,000 3,280,000

Calculate the EPS for 20X2 and the corresponding figure for 20X1.

Solution:
20X2 20X1
Weighted average number of shares:

Earnings

EPS =

In spite of the increase in total earnings by $20,000 in 20X2, the EPS is not as good as in 20X1, because
there was extra capital employed for the final 3 months of 20X2.

259
ACCA Financial Reporting (FR)
CHAPTER 13: IAS 33 EARNINGS PER SHARE

13.4.2 Bonus issues


15.1.58
- Shares are issue for existing shareholders for free
- No inflow of cash (increase in no of shares with no increase in resources), no contribution to earnings
- Process of capitalization of reserve Dr Reserve
Cr OSC
- Therefore, bonus issue to be taken in full in calculate of the denominator (not need to pro-rata)
- must restate previous year EPS to enable proper comparison

Exercise 13.4.2 (bonus issue):


Greymatter Co had 400,000 shares in issue, until on 30 September 20X2 it made a bonus issue of
100,000 shares. Calculate the EPS for 20X2 and the corresponding figure for 20X1 if total earnings
were $80,000 in 20X2 and EPS for 20X1 was 18.75c. The company's accounting year runs from 1
January to 31 December.

Shares at 1 January
Bonus issue
Weighted average number of shares

EPS =

The number of shares for 20X1 must also be adjusted if the figures for EPS are to remain comparable.
The EPS for 20X1 is therefore restated as:

260
ACCA Financial Reporting (FR)
CHAPTER 13: IAS 33 EARNINGS PER SHARE

13.4.3 Rights issues


15.1.59
Shares are issue for existing shareholders at below market price
- There is cash inflow with a bonus element (combination of fresh issue & bonus issue)
- Therefore, need to calculate the theoretical ex-rights price (TERP) & denominator must pro-rate into
before right issue (BRI) & after right issue (ARI)
- Must restate previous year EPS

Exercise 13.4.3 (TERP):


Egghead Co has 10,000,000 shares in issue. It now proposes to make a 1 for 4 rights issue at a price of
$3 per share. The market value of existing shares on the final day before the issue is made is $3.50.

What is the theoretical ex-rights price per share?

Solution:

$
Before issue 4 shares, value $3.50 each
Rights issue 1 share, value $3
Theoretical value of 5 shares

Theoretical ex-rights price =

261
ACCA Financial Reporting (FR)
CHAPTER 13: IAS 33 EARNINGS PER SHARE

Example 2 (rights issue):


Brains Co had 100,000 shares in issue, but then makes a 1 for 5 rights issue on 1 October 20X2 a t a
price of $1. The market value on the last day of quotation with rights was $1.60.

Calculate the EPS for the year ended 31 December 20X2 and the corresponding figure for 20X1 given
total earnings of $50,000 in 20X2 and $40,000 in 20X1.

Solution:

S1: Cal of TERP $


Before issue 5 shares, value $1.6 each
Rights issue 1 share, value $1
Theoretical value of 6 shares

Theoretical ex-rights price (TERP) =

S2: Cal crn yr EPS

- no of shares BRI = no of shares BRI x fraction of the yr x (old mkt px/TERP)


=
=
- no of shares ARI = no of shares ARI x fraction of the yr
=
=

Weighted average number of shares


=

EPS =

S3: Restate previous yr EPS


= old EPS x TERP/old mkt px
=

262
ACCA Financial Reporting (FR)
CHAPTER 13: IAS 33 EARNINGS PER SHARE

13.4.4 Partly paid shares


15.1.60

If ordinary shares are partly paid they are treated as a fraction of an OS to the extent they are entitled
to dividends relative to fully paid shares.

Example:

1.1.X1 1.10.X1 31.12.X1


Opening no of OSC: both fully
- 100,000 fully paid paid
- 50,000 OSC of $3,
only $2 paid (balance only received on 1.10.X1)

Weighted average number of shares 31.12.X1 =

263
ACCA Financial Reporting (FR)
CHAPTER 13: IAS 33 EARNINGS PER SHARE

13.5 Diluted EPS

Learning Outcome (ACCA Study Guide Area B)


B9e : Earnings per share (EPS)
(iii) Explain the relevance of the diluted eps and calculate the diluted eps involving convertible
debt and share options

A Co may have issue some securities which may give rise to potential OS in issue in the future

Therefore, future no of OS in issue might increase, results in a fall in the EPS (dilution in EPS).

DEPS is calculated to indicate to the investors the possible effects of a future dilution (calculated using
current earning but assuming that the worst possible future dilution)

Example of dilutive factors:

i. Convertible debt/loan notes


ii. Share options or warrants

DEPS = Adjusted Earnings (dilutedearnings)


Diluted weighted average no of OS outstanding

264
ACCA Financial Reporting (FR)
CHAPTER 13: IAS 33 EARNINGS PER SHARE

13.5.1 Convertible debt/loan notes


15.1.61

If converted, the interest would be saved:


i. Earnings would increase
ii. Number of ordinary shares would increase

DEPS = Earnings as BEPS + saving of int (net of tax)


Basic WANOS + weighted average max conversion

Exercise 13.5.1 (a) (convertible loan stock):


In 20X7 Farrah Co had a basic EPS of 105c based on earnings of $105,000 and 100,000 ordinary $1
shares. It also had in issue $40,000 15% convertible loan stock which is convertible in two years' time
at the rate of 4 ordinary shares for every $5 of stock. The rate of tax is 30%.

Calculate the diluted EPS.

Adj earnings $
Earnings as BEPS
+ saving of int
Adj earnings

Adj Weighted ave no of OS outstanding


Basic WANOS
+ weighted ave max conversion
Adj WANOS

EPS = 105c
DEPS =

265
ACCA Financial Reporting (FR)
CHAPTER 13: IAS 33 EARNINGS PER SHARE

Exercise 13.5.1 (b) (convertible loan stock):

On 1 April 20X1, an entity issued $1,250,000 8% convertible unsecured bonds for cash at par. Each
$100 nominal of the loan stock will be convertible in 20X6/20X9 into the no. of ordinary shares set out
below:

On 31 December 20X6 124 shares


On 31 December 20X7 120 shares
On 31 December 20X8 115 shares
On 31 December 20X9 110 shares

Relevant information

Issued share capital:


$500,000 in 10% cumulative preference shares of $1.
$1,000,000 in ordinary shares of 25c = 4,000,000 shares.
Income tax rate is 45%.

Trading results for the year ended 31 December 20X2 ($) 20X1 ($)
Profit before interest and tax 1,100,000 991,818
Interest on 8% convertible unsecured bonds (100,000) (75,000)
Profit before tax 1,000,000 916,818
Income tax (450,000) (412,568)
Profit after tax 550,000 504,250

Calculate the BEPS & DEPS of X1 & X2.

Solution:

BEPS X2 X1 DEPS X2 X1
Profit after tax Basic earnings
- pref divd + savings of interest
Basic Earnings (net of tax)
Adj earnings
Basic weighted average
no. of ordinary shares BWANOS
(BWANOS) + weighted ave max conversion
Adj WANOS

EPS = DEPS =

266
ACCA Financial Reporting (FR)
CHAPTER 13: IAS 33 EARNINGS PER SHARE

13.5.2 Share options or warrants


15.1.62

Gives the holder the right to buy shares in the future at a predetermined price
- There is cash inflow at the time the option is exercise
- Therefore, the no of shares issued on the exercise of the option/warrant is split into:

i. no of shares that would have been issued if the cash received had been used to buy shares at
Fair value
ii. the remainder are treated as bonus issue/ free shares

DEPS = Earnings as BEPS (no adjustment)


BWANOS + weighted average no. of free shares

Example 13.5.2 (options):

Brand Co has the following results for the year ended 31 December 20X7.

Net profit for year $1,200,000


Weighted average number of ordinary shares outstanding during year 500,000 shares
Average fair value of one ordinary share during year $20
Weighted average number of shares under option during year 100,000 shares
Exercise price for shares under option during year $15

Calculate both basic and diluted earnings per share.

Solution:

(a) BEPS =

(b) Adj Weighted ave no of OS outstanding


BWANOS
+ weighted ave no of free shares

i. Cal proceeds received from options =

ii. No of shares issued at FV with the proceeds received =

iii. No of free shares =

DEPS =

267
ACCA Financial Reporting (FR)
CHAPTER 13: IAS 33 EARNINGS PER SHARE

13.5.3 Presentation of EPS & DPS


15.1.63
- Should be presented separately on the face of SOPL&OCI for each class of OS
- Disclosure must still be made where the EPS figure are negative (i.e. Loss per share)

Statement of profit or loss and other comprehensive income 2011 2010


Revenue

Net profit

TCI

EPS xx xx
DEPS xx xx

13.5.4 Disclosure
15.1.64

(a) the amount used as the numerators in calculated BEPS & DEPS, and a reconciliation of those
amount to the net profit or loss for the period
(b) the weighted average no of OS used as the denominator in calculated the BEPS & DEPS, and a
reconciliation of these denominators to each other

268
ACCA Financial Reporting (FR)
CHAPTER 13: IAS 33 EARNINGS PER SHARE

13.5 Check Understanding

Question 1 (Fresh issue):

A plc has four million ordinary shares in issue and ranking for dividend at 01.01.19X1. On 30 Sept,
one million further shares were issued. Earnings per IAS 33 for the year ended 31.12.19X1 were
$500,000.

Solution:

$'m
Weighted average number of shares:

EPS =

Question 2 (Fresh issue):


A company issued 200,000 shares full market price ($3.00) on 1 July 19X8. Relevant information

19X8 19X7
Ordinary profit attributable to the ordinary shareholder for the year ending 31 Dec $550,000 $460,000
Number of ordinary shares in issue at 31 Dec 1,000,000 800,000

Solution:
19X8 19X7
Weighted average number of shares:

EPS for 19X8 =

EPS for 19X7 =

269
ACCA Financial Reporting (FR)
CHAPTER 13: IAS 33 EARNINGS PER SHARE

Question 3 (Bonus issue):


A company makes a bonus issue of one new share for every five existing shares held on 1 July 19X8.
Relevant information:

19X8 19X7
Ordinary profit attributable to the ordinary shareholder for the year ending 31 $550,000 $460,000
Dec
Number of ordinary shares in issue at 31 Dec 1,200,000 1,000,000

Solution:

EPS for 19X7 =

FS for 19X8 19X8 19X7 (restated)

EPS =

Question 4 (Bonus issue):

$
Net profit 20X1 180
Net profit 20X2 600
Ordinary shares outstanding until 200
30.09.20X2
Bonus issue at 01.10.20X2 2 ordinary shares for each ordinary
share outstanding at 30.09.20X2

Solution:

EPS for 20X1 =

FS for 20X2 20X2 20X1 (restated)

EPS =

270
ACCA Financial Reporting (FR)
CHAPTER 13: IAS 33 EARNINGS PER SHARE

Question 5 (Rights issue):


The following information relates to ABC Plc:

Year ended 31.12.


19X6 19X5
Net profit earned for equity ($'000) 60000 50000
No. of ordinary shares at 31 December ('000) 90000 75000

A rights issue was completed on 30 September 19X6 on a basis of 1 for 5 at $2.00. The average of
the closing prices of the last 7 days of quotation cum rights was $3.20.

Required:
(a) Calculate the theoretical ex rights price and determine the bonus element.

(b) Calculate the weighted average no. of shares & compute the EPS for both financial years.

Solution:

S1: Cal of TERP $


Before issue
Rights issue
Theoretical value of

TERP =

S2: Cal crn yr EPS


- no of shares BRI = no of shares BRI x fraction of the yr x (old mkt px/TERP)

- no of shares ARI = no of shares ARI x fraction of the yr

Weighted average number of shares =

EPS =

S3: Restate previous yr EPS


= old EPS x TERP/old mkt px
=

271
ACCA Financial Reporting (FR)
CHAPTER 13: IAS 33 EARNINGS PER SHARE

Question 6 (Rights issue):


A company issued one new share for every two existing shares held by way of rights at $1.50 per
share on 1 July 19X8. Pre-issue market price was $3.00 per share.

Relevant information: 19X8 19X7


Ordinary profit attributable to the ordinary shareholder for the year ending 31 Dec $550,000 $460,000
Number of ordinary shares in issue at 31 Dec 1,200,000 800,000

Solution:

S1: Cal of TERP $


Before issue
Rights issue
Theoretical value of

TERP =

S2: Cal crn yr EPS


- no of shares before right issue= no of shares BRI x fraction of the yr x (old mkt px/TERP)
=

- no of shares after right issue = no of shares ARI x fraction of the yr


=

Weighted average number of shares =

EPS =

S3: Restate previous yr EPS


= old EPS x TERP/old mkt px
=

272
ACCA Financial Reporting (FR)
CHAPTER 13: IAS 33 EARNINGS PER SHARE

Question 7 (EPS & DEPS):


On 1 January the issued share capital of Pillbox was 12 million preference shares of $1 each and 10
million ordinary shares of $1 each. Assume where appropriate that the income tax rate is 30%. The
earnings for the year ended 31 December were $5,950,000.

Calculate the EPS separately in respect of the year ended 31 December for each of the following
circumstances (a)-(f), on the basis that:

(a) there was no change in the issued share capital of the company during the year ended 31
December

Solution:
EPS =

(b) the company made a bonus issue on 1 October of one ordinary share for every four shares in issue
at 30 September

Solution:

Bonus issue
no of free shares =

EPS =

(c) the company issued 1 share for every 10 on 1 August at full market value of $4

Solution:

Fresh issue
Weighted average number of shares:

EPS =

273
ACCA Financial Reporting (FR)
CHAPTER 13: IAS 33 EARNINGS PER SHARE

(d) the company made a rights issue of $1 ordinary shares on 1 October in the proportion of 1 of every
3 shares held, at a price of $3. The middle market price for the shares on the last day of quotation
cum rights was $4 per share

Solution:

Right issue:

S1: TERP
Before issue
Rights issue
Theoretical value of

TERP =

S2: Crn yr EPS


no of shares BRI = no of shares BRI x fraction of the yr x (old mkt px/TERP)
=

no of shares ARI = no of shares ARI x fraction of the yr

Weighted average number of shares =

EPS =

(e) the company made no new issue of shares during the year ended 31 December, but on that date
it had in issue $2,600,000 10% convertible bonds. These bonds will be convertible into ordinary
$1 shares as follows:

20X6 90 $1 shares for $100 nominal value bonds


20X7 85 $1 shares for $100 nominal value bonds
20X8 80 $1 shares for $100 nominal value bonds
20X9 75 $1 shares for $100 nominal value bonds

274
ACCA Financial Reporting (FR)
CHAPTER 13: IAS 33 EARNINGS PER SHARE

Solution:

Convertible bonds – DEPS

Adj earnings $
Earnings as BEPS
+ saving of int (net of tax)
Adj earnings

Adj WANOS outstanding


Basic WANOS
+ weighted ave max conversion
Adj WANOS

DEPS:

(f) the company made no issue of shares during the year ended 31 December, but on that date there
were outstanding options to purchase 74,000 ordinary $1 shares at $2.50 per share. Share price
during the year was $4.

Solution:

Options - DEPS
Adj WANOS outstanding
Basic WANOS
+ weighted ave no of free shares

DEPS:

275
ACCA Financial Reporting (FR)
CHAPTER 13: IAS 33 EARNINGS PER SHARE

Question 8 (Partly paid shares & right issue)


ABC group has traded for many years. The issued share capital on 1 Jan 2002 was 300,000 $1 OS. Of
this amount, there were 50,000 $1 OS, which were only paid up to the amount of 75c. The shares
were issued at par value. The following information relates to the period 1 Jan 2002 to 31 Dec 2003.

a. On 1 April 2002, the outstanding money on the partly paid shares were received.
b. During 2003, ABC decided to raise a further capital by making right issue. The right issue took
place on 31 Aug 2003 on a 1 new share for every 2 existing shares basis @ $1.20 per share. The
actual cum-rights price on the last day of quotation cum rights was $1.80.

The following extract of SOPL for the years ending 31 Dec 2002 ($) 2003 ($)
Net profit for the year 200,000 250,000

Cal EPS for each of the years ending 31 Dec 2002 & 2003, showing comparative figure for 2003.

Answer
Weighted average number of shares:

EPS for 2002 =

Right issue in 2003:


S1: TERP

S2: Current year EPS


- no of shares before right issue

- no of shares after right issue

Total no of shares =

EPS =

S3: Restate previous year EPS

276
ACCA Financial Reporting (FR)
CHAPTER 13: IAS 33 EARNINGS PER SHARE

Question 9 (Bonus issue & right issue)

Extract of SOFP of Radan as at 1 April 2006 $'000


Ordinary shares of 25c each 4,000
8% preference shares 1,000
Reserves 2,840

10% convertible loan notes 2,000

Extract of SOPL for the year to 31 March 2007 $'000


Net profit for the year 910

The following information is relevant:


a. A bonus issue of 1 new share for every 8 shares held was made on 7 Sep 2006.

b. A fully subscribed right issue of 1 new share for every 5 ordinary shares held at a price of $0.50
each was made on 1 Jan 2007. Immediately prior to the issue, the share price of Radan's ordinary
shares was $1.40 each (assuming that bonus issue ranks for right issue).

c. The terms of conversion of the 10% debentures are:

Year Debentures Ordinary shares


2009 to 2013 $100 100
2014 $100 120
Income tax is to be taken at 33%.

d. The EPS was correctly reported in last year's accounts at 8c.

Cal the EPS for Radan for the year ended 31 March 2007
i. on a basic basis (including comparative)
ii. on a diluted basis (ignore the comparative)

277
ACCA Financial Reporting (FR)
CHAPTER 13: IAS 33 EARNINGS PER SHARE

Solution:

(i) Cal of BEPS


S1: TERP

S2: Current year EPS


- no of shares before right issue (including bonus issue)

- no of shares after right issue

Total no of shares =

EPS =

S3: Restate previous year EPS

(ii) Compute of DEPS


Adjusted earnings $
Earnings as BEPS
+ saving of interest
Adjusted earnings

Adjusted Weighted average no of OS outstanding


BWANOS
+ weighted average max conversion
Adjusted WANOS

DEPS =

278
ACCA Financial Reporting (FR)
CHAPTER 13: IAS 33 EARNINGS PER SHARE

Question 10 (extract of June'03)

Extracts of Niagara's SOPL for the year to 31 March 2003 are:


$'000
Net profit for the year 2,585

Niagara paid an interim ordinary dividend of 3c per share in June 2002 and declared a final dividend
on 25 March 2003 of 6c per share.

The issued share capital of Niagara on 1 April 2002 was: $'m


- Ordinary shares of 25c each 3
- 8% irredeemable preference shares 1

The company also had in issue $2m 7% convertible loan stock dated 2005. The loan stock will be
redeemed at par in 2005 or converted to ordinary shares on the basis of 40 new shares for each
$100 of loan stock. Niagara's income tax rate is 30%. There is also an existence share warrants
(issued in 2001) which entitle the directors to receive 75,000 new shares in total in 2005 at no cost
to the directors.

The following shares issue took place during the year to 31 March 2003:

▪ 1 July 2002, a right issue of 1 new share at $1.5 for every 5 shares held. The market price of
Niagara's shares the day before the rights was $2.4.

▪ 1 October 2002, an issue of $1m 6% non-redeemable preferences shares at par.

Both issues were fully subscribed.

Niagara's basic EPS in the year 31 March 2002 was correctly disclosed at 24c.

Calculate the BEPS (including comparative) and DEPS (ignoring comparative) assuming all
instruments are dilutive in nature.

279
ACCA Financial Reporting (FR)
CHAPTER 13: IAS 33 EARNINGS PER SHARE

Question 11:

Fenton had 5,000,000 ordinary shares in issue on 1 January 20X1. On 31 January 20X1, the company
made a rights issue of 1 for 4 at $1.75. The cum rights price was $2 per share. On 30 June 20X1, the
company made an issue at full market price of 125,000 shares. Finally, on 30 November 20X1, the
company made a 1 for 10 bonus issue. Profit for the year was $2,900,000. The reported EPS for year
ended 31 December 20X0 was 46.4c.

(a) What was the earnings per share figure for year ended 31 December 20X1 and the restated EPS
for year ended 31 December 20X0?

Sinbad had the same 10 million ordinary shares in issue on both 1 January 20X1 and 31 December
20X1. On 1 January 20X1 the company issued 1,200,000 $1 units of 5% convertible loan stock.
Each unit of stock is convertible into 4 ordinary shares on 1 January 20X9 at the option of the
holder. The following is an extract from Sinbad's SOPL for the year ended 31 December 20X1:

$'000
Profit before interest and tax 980
Interest payable on 5% convertible loan stock (60)
Profit before tax 920

Income tax expense (at 30%) (276)


Profit for the year 644

(b) What was the basic and diluted earnings per share for the year ended 31 December 20X1?

Talbot has in issue 5,000,000 50c ordinary shares throughout 20X3. During 20X1 the company
had given certain senior executives options over 400,000 shares exercisable at $1.10 at any time
after 31 May 20X4. None were exercised during 20X3. The average market value of one ordinary
share during the period was $1.60. Talbot had made a profit after tax of $540,000 in 20X3.

(c) What is the basic and diluted earnings per share for the year ended 31 December 20X3?

280
ACCA Financial Reporting (FR)
CHAPTER 13: IAS 33 EARNINGS PER SHARE

Exercise 12 (Extract of Dec'02)


The following information obtained from published financial statements of Taylor:
EPS
Year to 30 Sep 2002 2001
Basic EPS 25cents20cents

The EPS is based on attributable earnings of $50m ($30m in 2001) and 200m ordinary shares in issue
throughout the year (150m weighted average no of ordinary shares in 2001).
Extracts of SOFP $'000 $'000
8% convertible loan stock 200 200

The loan stock is convertible to ordinary shares in 2004 on the basis of 70 new shares for each $100
loan stock.

Notes to the FS:


There are directors' share options (in issue since 1999) that allow Taylor's directors to subscribe for a
total of 50m new ordinary shares at a price of $1.50 each.
(Assume the current income tax rate of 25% and the market price of its ordinary shares throughout
the year has been $2.50)

a. Explain why the trend of EPS may be different from the trend of the reported profit, and which
is the more useful measure of performance. (3 marks)

b. Explain the relevance of the diluted EPS measure (4 marks)

Solution:

(a) Trend of EPS vs trend of reported profit


The trend shown by a comparison of company's profits over time is rather a 'raw' measure of
performance and can be misleading without careful interpretation of all the events that the
company has experienced.
In the year to 30 Sep 2002, Taylor's EPS has increased by 25% (from 20c to 25c), whereas its profit
has increased by a massive 67% (from $30m to $50m).

It is not possible to determine exactly what has caused the difference between the % increase in EPS
and the % increase in reported profit of Taylor, but a simple example may illustrate a possible
explanation:

Assume Co A acquired Co B by way of a share exchange. Both companies had the same market value
and the same profits. A comparison of A's post combination profits [incorporate both companies'
profits] with its pre-combination profits [only Co A's profit alone] would be very misleading.

The trend shown by the EPS goes same way to addressing such distortion.
The increase in post combination profits would also be accompanied by the increase in the issued
share capital (due to share exchange) thus the reported EPS of Co A would not be distorted by its
acquisition growth.
It can therefore be argued that the trend of EPS is a more reliable measure of its earnings
performance than the trend shown by reported profits.

281
ACCA Financial Reporting (FR)
CHAPTER 13: IAS 33 EARNINGS PER SHARE

(b) Relevance of DEPS


DEPS highlights the problem of relying too heavily on a company's BEPS when trying to predict
future performance.

Certain circumstances exist which may cause future EPS to be lower than current levels irrespective
of future profit performance. These are said to cause a dilution in EPS.

Common examples are existence of convertible loan stock or share options that may cause an
increase in the future no of shares without being accompanied by a proportionate increase in
earnings.
The BEPS is a measure of past performance while DEPS is more looking forward and is intended to
act as a warning to existing and prospective shareholders that, based on current level of earnings,
the BEPS would be lower of the diluting circumstances had crystalised.

DEPS is still based on past performance; it does give effect to potential ordinary shares outstanding
during the period. It is not a prediction of the future EPS, as these will be based on future profits and
the no of shares in issue in the future.

282
ACCA Financial Reporting (FR)
CHAPTER 13: IAS 33 EARNINGS PER SHARE

13.6 Chapter 13 Summary

Diagram 13.7.: Summary of IAS 33 EPS

283
ACCA Financial Reporting (FR)
CHAPTER 14: IFRS 3 BUSINESS COMBINATION

CHAPTER 14: IFRS 3 BUSINESS


COMBINATION

LEARNING OUTCOME
At the end of the chapter, you should be able to:
TLO A4a. Describe the concept of a group as a single economic unit.
TLO A4b. Explain and apply the definition of a subsidiary within relevant accounting standards.
TLO A4c. Using accounting standards and other regulation, identify and outline the
circumstances in required to prepare consolidated which a group is financial
statements
TLO A4d. Describe the circumstances when a group may claim exemption from the
preparation of consolidated financial statements.
TLO A4e. Explain why directors may not wish to consolidate a subsidiary and when this is
permitted by accounting standards and other applicable regulation.
TLO A4f. Explain the need for using coterminous year ends and uniform accounting polices
when preparing consolidated financial statements.
TLO A4g. Explain why it is necessary to eliminate intra group transactions.
TLO A4h. Explain the objective of consolidated financial statements.
TLO A4i. Explain why it is necessary to use fair values for the consideration for an investment
in a subsidiary together with the fair values of a subsidiary’s identifiable assets and
liabilities when preparing consolidated financial statements.
TLO A4j. Define an associate and explain the principles and reasoning for the use of equity
accounting.

284
ACCA Financial Reporting (FR)
CHAPTER 14: IFRS 3 BUSINESS COMBINATION

14.1 Concept of a group

Learning Outcome (ACCA Study Guide Area A)


A4a : Describe the concept of a group as a single economic unit.

When one company owns enough shares in another company to have a majority of votes at that
company's annual general meeting (AGM), the first company may appoint all the directors of, and
decide what dividends should be paid by, the second company .

This degree of control enables the first company to manage the trading activities and future plans of
the second company as if it were merely a department of the first company.

The key principle underlying group accounts is the need to reflect the economic substance of the
relationship.

The 1st Co who has control  Is the Parent / Holding Co

The 2nd Co who follows the policies & rules of the 1st Co  Is the Subsidiary Co

 = control  The parent & its sub together constitute a GROUP!

Legal point of view P is an individual legal entity

S is an individual legal entity

Based on substance over form  P controls S  therefore form a single economic entity

14.2 Accounting principles underlying group accounts

Learning Outcome (ACCA Study Guide Area A)


A4b : Explain and apply the definition of a subsidiary within relevant accounting standards.

The key issue underlying group accounts is the need to reflect the economic substance of the
relationship between companies where one (a parent) has control over another (a subsidiary), which
together comprise a group.

Consolidated financial statements are the financial statements of a group presented as those of a
single economic entity.
is a parent and all its subsidiaries

controls one or more subsidiaries being controlled by another entity

285
ACCA Financial Reporting (FR)
CHAPTER 14: IFRS 3 BUSINESS COMBINATION

14.3 Control and power

Learning Outcome (ACCA Study Guide Area A)


A4c : Using accounting standards and other regulation, identify and outline the circumstances in
required to prepare consolidated which a group is financial statements

14.3.1 Control
15.1.65
An investor controls an investee if it has all of the following:
▪ has power over the investee
▪ has exposed, or has rights to variable returns from its involvement with the investee
▪ has the ability use its power to affect the investor's return

14.3.2 Power
15.1.66

An investor has power over an investee when the investor has:

 Existing rights that give it the current ability to direct the relevant activities

Sometimes assessing power is straightforward, such as activities that


when power over an investee is obtained directly and significantly affect
solely from the voting rights granted by equity the investee’s
instruments such as shares, and can be assessed by returns.
considering the voting rights from those shareholdings.
E.g. Operating and
In other cases, the assessment will be more complex financing activities
and require more than one factor to be considered
[E.g. when power results from one or more contractual
arrangements]

Example 14.3.2 (a) (majority of voting rights)

An investor owns 40% equity shares of an investee. It enters into an arrangement with another
shareholders of the investee to have the power to exercise the other shareholder's 11% voting
rights. In this case, the investor's own shareholdings and the arrangement with the other
shareholder give it the current ability to exercise more than a majority of the voting rights in the
investee.

Example 14.3.2 (b) (de facto control)

An investor might have control over an investee even when it has less than a majority of the voting
rights of that investee (a concept known as ‘de facto control’).

A holds 48% of the voting rights of B; the remaining 52% of B is widely held by thousands of
shareholders (none of whom holds more than 1% of the voting rights). A has power over B, because
A has a dominant voting interest (based on the absolute size of its holding, and relative to other
shareholders), and a large number of shareholders would have to agree to outvote A.

286
ACCA Financial Reporting (FR)
CHAPTER 14: IFRS 3 BUSINESS COMBINATION

Example 14.3.2 (c) (Potential voting rights)

When assessing whether it has power over an investee, an investor also considers the potential voting
rights that it holds, as well as potential voting rights held by others. Common examples of potential
voting rights include options, forward contracts, and conversion features of a convertible instrument.

A holds 40% of the voting rights of B, and holds a currently exercisable in-the-money option to acquire
a further 20% of the voting rights of B.

Assuming that voting rights give power over B, the option is substantive and no other facts and
circumstances are relevant to this assessment, a would likely have power over B, because A can
currently exercise its right to obtain a majority of B’s voting shares at any time.

Example 14.3.2 (d) (Protective rights)

Only substantive voting rights are considered in assessing power.

Protective rights are designed to protect the interests of their holder without giving that party power
over the investee to which those rights relate. Therefore, an investor that holds only protective rights
cannot have power over an investee and so cannot control an investee.

For example, a lender’s right to restrict a borrower from undertaking activities that could significantly
change the credit risk of the borrower to the detriment of the lender.

287
ACCA Financial Reporting (FR)
CHAPTER 14: IFRS 3 BUSINESS COMBINATION

14.4 Types of group structures

14.5 Composition of group accounts

14.5.1 Content of group accounts


15.1.67
Group accounts comprise of:

• Consolidated statement of financial position (COSFP)


• Consolidated statement of profit or loss and other comprehensive income (CSOPL& OCI)
• Consolidated statement of changes in equity (CSOCE)
• Consolidated statement of cash flows (CSOCF)
• Notes to the accounts and comparative figures

288
ACCA Financial Reporting (FR)
CHAPTER 14: IFRS 3 BUSINESS COMBINATION

Points to note:

1. The consolidated statement of financial position is presented in addition to the parent's own
individual statement of financial position.
2. The consolidated statement of profit or loss is usually presented instead of the parent's own
individual statement of profit or loss.
3. The parent's own statement of financial position shows its investment in subsidiaries in
noncurrent asset investments (usually at cost).
4. The parent's own individual statement of profit or loss shows the dividend income received
and receivable from subsidiaries.

Standards relevant to group accounts are:


▪ IFRS 3 Business Combinations (revised Jan 2008)
▪ IAS 27 Separate Financial Statements (revised May 2011)
▪ IFRS 10 Consolidated Financial Statements (issued May 2011)
▪ IAS 28 Investments in Associates (revised May 2011)

IFRS 10 requires a parent to present consolidated financial statements. It should consolidate all
subsidiaries, both foreign and domestic.

14.5.2 Exemption from preparing group accounts


15.1.68
Learning Outcome (ACCA Study Guide Area A)
A4d : Describe the circumstances when a group may claim exemption from the preparation of
consolidated financial statements.

A parent need not present consolidated financial statement if and only if all of the following hold:

a. The parent is itself a wholly-owned subsidiary or it is a partially owned subsidiary of another


entity and its other owners, including those not otherwise entitled to vote, have been informed
about, and do not object to, the parent not presenting consolidated financial statement

b. its debt or equity instruments are not traded in a public market (a domestic or foreign stock
exchange or an over-the-counter market, including local and regional markets)

c. it did not file, nor is it in the process of filing, its financial statements with a securities commission
or other regulatory organizations for the purpose of issuing any class of instruments in a public
market, and

d. its ultimate or any intermediate parent of the parent produces consolidated financial statement
available for public use that comply with International Financial Reporting Standards.

289
ACCA Financial Reporting (FR)
CHAPTER 14: IFRS 3 BUSINESS COMBINATION

14.5.3 IAS 27 Separate Financial Statements


15.1.69

A parent that does not present consolidated FS must comply with the IAS 27 rules on separate FS

IAS 27 Separate Financial Statements prescribes the accounting and disclosure requirement for
investments in subsidiaries, joint ventures and associates when an entity prepares separate FS.

▪ at cost, or ▪ The fact that the FS are separate FS


▪ in accordance with IFRS 9 ▪ List of significant Invst in Subs/ JV/
Associate
▪ Description of method used to account
for the Invst

14.5.4 Exclusion of a Subsidiary from Consolidation


15.1.70

Learning Outcome (ACCA Study Guide Area A)


A4e : Explain why directors may not wish to consolidate a subsidiary and when this is permitted by
accounting standards and other applicable regulation.

The rules on exclusion of subsidiaries from consolidation are necessarily strict, because this is a
common method used by entities to manipulate their results. For example, if a subsidiary which carries
a large amount of debt can be excluded, then the gearing of the group as a whole will be improved
(e.g. off-balance sheet financing).

Past practice under IAS 27 Revised IAS27 /IFRS 10


Subsidiary can be excluded from
consolidation if:

Control is intended to be temporary  If meet the conditions in IFRS 5 to be classified


(E.g. Subs is purchased w intention to dispose as held for sale, it is not accounted for under
within 12 months) IFRS 10 but IFRS 5

Dissimilar activities  Must consolidate results and then giving


additional info about the different business
activity of the subs (follow IFRS 8 Segment
reporting)

Operates under severe long-term restrictions  Must consolidate until control is actually lost
and impair its ability to transfer funds to the
parent

290
ACCA Financial Reporting (FR)
CHAPTER 14: IFRS 3 BUSINESS COMBINATION

14.5.5 Different Reporting Dates


15.1.71

Learning Outcome (ACCA Study Guide Area A)


A4f : Explain the need for using coterminous year ends and uniform accounting polices when
preparing consolidated financial statements.

In most cases, all group companies will prepare accounts to the same reporting date. One or more
subsidiaries may, however, prepare accounts to a different reporting date from the parent.

The subsidiary may prepare additional statements to the reporting date for consolidation purposes
(i.e.: P Co Y/E = 31 Dec, S Co Y/E = 30 Sep ---> S Co must prepare additional financial statement for 3
months till 31 Dec)

If it is not possible for S to prepare additional statements, the S's accounts may still be used for the
consolidation, provided that the gap between the reporting dates is three months or less.

[i.e.: P Co 12 months (1 Jan - 31 Dec) + S Co 9 months (1 Jan - 30 Sep)

Where the S's accounts are drawn up to a different accounting date, adjustments should be made for
the effects of significant transactions or other events that occur between that date and the parent's
reporting date.

14.5.6 Uniform Accounting Policies


15.1.72
Learning Outcome (ACCA Study Guide Area A)
A4f : Explain the need for using coterminous year ends and uniform accounting polices when
preparing consolidated financial statements.

Consolidated financial statements should be prepared using uniform accounting policies for like
transactions and other events in similar circumstances. If use different accounting policies, adjustment
must be made for consolidation purposes .

14.5.7 Date of Inclusion/Exclusion


15.1.73

The results of subsidiaries to be included in the consolidated financial statements from:

a. the date of 'acquisition'  the date which investor obtains control of investee, to
b. the date of 'disposal'  the date which investor loses control of the investee

Once an investment is no longer a subsidiary, it should be treated as an associate under IAS 28 (if
applicable) or as an investment under IFRS 9.

291
ACCA Financial Reporting (FR)
CHAPTER 14: IFRS 3 BUSINESS COMBINATION

14.6 Non-controlling interest

Non-controlling interest is the equity in a subsidiary not attributable, directly or indirectly, to a parent
(investors from outside the group). In some situations, a parent may not own all of the shares in the
subs (i.e.: own 80%), there is a non-controlling interest of 20%.

In consolidated financial statements, non-controlling interest is presented within equity, separately


from the owner's equity of the parent.

100% 51% 95%

NCI = NCI = NCI =

14.7 Issue of related parties

The relationship between a parent and a subsidiary is the most obvious example of a related party
(IAS 24) relationship and it offers a number of opportunities for manipulating results. For example,
any of the following could take place to improve the parent’s individual FS:

▪ S sells goods to H at an artificially low price. This increases H’s profit while reducing profit in
the S, thus increasing profit available for distribution to H’s shareholders at the expense of the
non-controlling interest.
▪ H sells goods to S at an artificially high price. This has the same result as above.
▪ S makes a loan to H at an artificially low rate of interest or H makes a loan to S at an artificially
high rate of interest. The loans will be cancelled on consolidation but the interest payments
will transfer profits from the S to H.
▪ H can sell an asset to S at an amount in excess of its carrying amount. This again serves to
transfer profit (and cash) to H.
▪ Many large businesses consist of several companies controlled by one central or
administrative company. S may be charged by a large share of group operating expenses to
be favourable to H.

IAS 24 draws attention to the significance of related party relationships and transactions – that
transactions between the parties may not be at arm’s length’ and that users of the accounts must be
made aware of this, as it may affect their view of the financial statements.

292
ACCA Financial Reporting (FR)
CHAPTER 14: IFRS 3 BUSINESS COMBINATION

14.8 Goodwill

14.8.1 Definition
15.1.74
Goodwill is created by good relationship between a business and its customers:

▪ by building up a reputation for high quality products / services


▪ by responding promptly and helpfully to queries and complaints from customers
▪ good management and staff

Definition by IFRS 3

Future eco benefits arising from assets that are not capable of being individually identified and
separately recognised.

14.8.2 Purchased goodwill


15.1.75
The excess of the cost of a business combination over the acquirer's interest in the net fair value of
the assets, liabilities and contingent liabilities of the business acquired.

14.8.3 Accounting treatment for Goodwill


15.1.76

Goodwill

Purchased Internally Generated

Positive Negative
Capitalise as intangible non-current Reassess whether there is error in calculation of cost of
assets in the consolidated statement of business combination, or fair value of net asset
financial position (Cost – Annual acquired, or due to bargain purchase
Impairment Loss)
No amortisation Credit any remainder to the consolidated statement of
profit and loss on date of acquisition
Test annually for impairment (IAS 36) The gain shall be attributed to the acquirer

293
ACCA Financial Reporting (FR)
CHAPTER 14: IFRS 3 BUSINESS COMBINATION

Reasons for bargain purchase

▪ The acquiree is in a poor financial position and needs to realise assets quickly
▪ The acquiree's share price may be depressed due to poor profit record or in expectation
of future losses and the offer price seems reasonable in comparison to the stock mkt
price
▪ The acquiree's assets may be worth more than their carrying amount if put to a
different use or simply that the co does not realise how much the assets are really worth

▪ The acquirer may have good negotiating skills

14.8.4 Computation of Goodwill


15.1.77

FULL goodwill/ Fair Value Method Partial goodwill/ Proportion of net assets
method
Fair value of Consideration Paid xx FV of Consideration Paid xx
+ Fair value of non-controlling xx + Fair value of non-controlling xx
interest interest
(Non-controlling interest’s shares X (Non-controlling interest % X Fair
Subsidiary’s share price) value of net assets acquired)
- Fair value of net assets acquired (xx) - Fair value of net assets acquired (xx)
Goodwill at acquisition date xx Goodwill at acquisition date xx

Initial measurement at the date of acquisition

Example 14.8.4:
Daddy Co acquired 80% of 10,000 $1 ordinary shares of Baby Co on 31 Dec 2015 for $78,000. At this
date, the net assets of Baby Co were $85,000.

What goodwill arises on the acquisition if:


i) Non-controlling interest is valued using fair value method. Baby Co’s shares were trading at $9.50
just prior to acquisition by Daddy Co.
ii) Non-controlling interest is valued using proportion of net assets method?

FULL GW/ FV Method Partial GW/ Proportion of NA method


FV of Consideration Paid FV of Consideration Paid
+ FV of NCI + FV of NCI
(NCI’s shares x Subs’ sh px) (NCI % x FV of NA acq)
- FV of NA acquired - FV of NA acquired
GW at acquisition date GW at acquisition date

294
ACCA Financial Reporting (FR)
CHAPTER 14: IFRS 3 BUSINESS COMBINATION

Subsequent measurement of goodwill at year end

Tested for annual impairment and whenever there is an indication of impairment as per IAS 36

Double entries for impairment loss:

Full GW / FV method Partial GW / Proportion of NA method


Dr CRE (H's %) Dr Post RE of S Dr CRE
Dr NCI (NCI's %) Cr GW
Cr GW

14.9 Cost of Investment

The consideration paid by the parent (acquirer) for the shares in subsidiary (acquiree) can take
different forms

▪ Cash paid
▪ Fair value of other considerations
✓ Deferred consideration
✓ Contingent consideration
✓ Shares exchange/ issued
✓ Loan notes issued

Acquisition/ incidental costs

The direct acquisition or incidental cost such as professional fees (i.e.: legal, acco unting, valuation)
attributable to the combination should be expensed as incurred.

Dr

Cr

If the acquisition cost has been wrongly included in the cost of investment, then a correction entry
must be done in order not to overstate goodwill.

Dr

Cr

Issue cost/ transaction cost

The issue cost of debt / equity associated with the acquisition should reduce the proceeds from the
debt / equity issue in accordance with IFRS 9/IAS 32.

295
ACCA Financial Reporting (FR)
CHAPTER 14: IFRS 3 BUSINESS COMBINATION

14.9.1 Deferred Consideration


15.1.78

An agreement to settle part of the consideration at a future date

a) If in the form of cash

Fair value of deferred consideration to be recognised as part of the consideration by discounting the
amount payable to present value (PV) at acquisition using the cost of capital

1
r: interest rate
(1+𝑟)𝑛
n: no of years to settlement

Dr Investment in Subsidiary
Cr Deferred consideration (NCL / CL)

Example 14.9.1 (a)

The parent acquired 75% of the Subsidiary's $80m $1 ordinary share capital on 1 Jan 2009. It paid
$3.50 per shares and agreed to pay a further $108m on 1 Jan 2010. The parent cost of capital is 8%.

Computation of cost of combination @ 1 Jan 2009 $'m


Cash paid
Deferred consideration
Total consideration

At 31 Dec 2009, $8m ($100 x 8%) will be charged to finance costs, being the unwinding of the discount
on the deferred consideration. The deferred consideration was discounted by $8m to allow for the
time value of money. At 1 Jan 2010, the full amount becomes payable.

Example 14.9.1 (b)

On 1 October 2010, Paladin purchased 8m shares in Saracen on the following terms:


▪ an immediate payment of $4 per share on 1 October 2010; and
▪ a further amount deferred until 1 October 2011 of $5·4 million.
Paladin’s cost of capital is 8% per annum.

Compute the Cost of Investment in Saracen

296
ACCA Financial Reporting (FR)
CHAPTER 14: IFRS 3 BUSINESS COMBINATION

If is the form of equity shares:

▪ Fair value of deferred consideration to be recognised as part of the consideration by using the
market price of shares at date of acquisition and
▪ The deferred amount is recognised as part of the equity under the separate heading such as
"shares to be issued"
▪ No adjustments will be made for any subsequent changes in mkt price of shares

Dr Investment in Subsidiary
Cr Shares to be issued

14.9.2 Contingent Consideration


15.1.79

▪ An agreement to settle in the future provided certain conditions attached to the agreement
are met.
▪ Fair value of contingent consideration to be recognised as part of the consideration as long as
it can be measured reliably even if payment is not probable at date of acquisition
▪ Can be in the form of shares or cash (treatment same as deferred consideration)

Example 14.9.2

The parent acquired 60% of the Subs's $100m OSC on 1 Jan 2009 for a cash payment of $150m and a
further payment of $50m on 31 Dec 2010 if the subsidiary’s post acq profit have exceeded an agreed
figure by that date. The parent cost of capital is 10%.

Initial measurement @ 1 Jan 2009 $'m


Cash
Contingent consideration

Subsequent measurement @ 31 Dec 2009

297
ACCA Financial Reporting (FR)
CHAPTER 14: IFRS 3 BUSINESS COMBINATION

14.9.3 Subsequent Measurement of Contingent Consideration


15.1.80

If the cost of acquisition changes subsequently due to events taken place after the acquisition date
(i.e. meeting an earning target which trigger higher payment than was provided for at the acquisition
date), they are treated as changes in accounting estimates and accounted for prospectively with any
changes recognised in profit or loss. Goodwill is therefore not reinstated.

Increase of contingent consideration Decrease of contingent consideration

Dr Consolidated Retained Earnings Dr Contingent consideration


Cr Contingent consideration Cr Consolidated Retained Earnings

Example 14.9.3

On 1 April 20X0 Picant acquired 75% of Sander's 8,000,000 equity shares in a share exchange of three
shares in Picant for every two shares in Sander. The market prices of Picant's and Sander's shares at
the date of acquisition were $3.20 and $4.50 respectively.

In addition to this Picant agreed to pay a further amount on 1 April 20X1 that was contingent upon
the post-acquisition performance of Sander. At the date of acquisition Picant assessed the fair value
of this contingent consideration at $4.2 million, but by 31 March 20X1 it was clear that the actual
amount to be paid would be only $2.7 million (ignore discounting).

Compute the Cost of Investment in Sander at the date of acquisition

14.9.4 Shares Exchange


15.1.81

The parent Co will issue shares in its own Co in return for the shares acquired in subs.

The share price (market price of shares) at date of acquisition should be used to record the cost of
the shares at FV.

Example
The parent acquired 75% of the Subs's $20,000 $1 OSC on 1 Jan 2009 by issuing 5 of its own $1 shares
for every 4 shares in the subs. The mkt value of the parent Co's shares is $6.

Computation of cost of combination @ 1 Jan 2009

Double entry
Dr Investment in subs
Cr Share cap
Cr Share premium

298
ACCA Financial Reporting (FR)
CHAPTER 14: IFRS 3 BUSINESS COMBINATION

14.10 Calculation of Fair Value of Net Asset Acquired


The acquirer should recognise the acquiree's identifiable assets, liabilities and contingent liabilities
at DOA at their FV.

Recognize only if: Recognize only if: Recognize only if its FV can
Probable outflow of be measured reliably
Probable inflow of future future economic benefit (departure from normal
economic benefit to the will be required to settle rules in IAS 37)
acquirer and its fair value the obligation and its FV
can be measured reliably. can be measured reliably.

14.10.1 Restructuring and Future Losses


15.1.82
An acquirer should not recognise liability for future losses or other costs expected to be incurred as a
result of the biz comb.

IFRS 3 explains that a plan to restructure a subsidiary after acquisition is not a present obligation of the
acquire at DOA and it does not meet the definition of a contingent liability.

Therefore, an acquirer should not recognise liability for a restructuring plan as part of allocating the cost
of the combination unless the subsidiary was already committed to the plan before the acquisition.

14.10.2 Intangible Assets


15.1.83

The acquiree may have intangible assets, such as development expenditure. These can be recognised
separately from goodwill only if they are identifiable (refer to IAS 38).

Identifiable assets & liabilities might include assets & liabilities not previously recognised in the
acquiree's FS.

For example, the acquiree may also have internally generated assets such as brand names which have
not been recognised as intangible assets (IAS 38), as the acquiring company is giving valuable
consideration for these assets, they are now recognised as assets in the consolidated FS.

Example 14.10.2
If asset/liability is recognised by acquirer (holding) at DOA where acquiree (subs) did not recognised
previously:

For asset recognized (i.e.: intangible asset) For liability recognized (i.e.: contingent
Dr Intangible asset (NCA in CSOFP) liability)
Cr Pre net assets of S  Goodwill decrease Dr Pre net assets of S  Goodwill increase
 Cr Contingent liability (CL in CSOFP)
Subsequent depreciation/ amortization
charged
Dr Post RE of S
Cr Intangible asset

299
ACCA Financial Reporting (FR)
CHAPTER 14: IFRS 3 BUSINESS COMBINATION

14.11 Check Understanding

Question 1

On 1 Jan 2010, KB acquired 80% of KC. The following details are relevant:
Consideration was composed of:

▪ $170,000 cash
▪ 18,000 ordinary shares at par value of $1
▪ 12,500 shares to be issued in 2 years, dependent upon a 5% year on year rise in profits

Share issues costs amount to $3,200


Other cost of acquisition amount to $16,000
The profits of KC have consistently increased by 3% annually over the last 5 years.
The market price of KB's ordinary shares at 1 Jan 2010 was $1.60.

Compute the Cost of Investment in KC at the date of acquisition

Solution:

Fair Value of CP $
Cash
Shares issued
Shares to be issues

Question 2

Parentis, a public listed company, acquired 600 million equity shares in Offspring on 1 April 20X6.

The purchase consideration was made up of:


▪ A share exchange of one share in Parentis for two shares in Offspring
▪ The issue of $100 10% loan note for every 500 shares acquired; and
▪ A deferred cash payment of 11 cents per share acquired payable on 1 April 20X7.

The value of each Parentis share at the date of acquisition was 75 cents and Parentis has a cost of
capital of 10% per annum.

Compute the Cost of Investment in Offspring

Solution:

Fair Value of CP $'000


Shares issued
Loan note issued
Deferred consideration

300
ACCA Financial Reporting (FR)
CHAPTER 14: IFRS 3 BUSINESS COMBINATION

Question 3

H acquired 75% of the OSC of S at 1 April 20X4. S had established a line of products under the brand
name of Titanware. Acting on behalf of H, a firm of specialists, had valued the brand name at a value
of $40 million with an estimated life of 10 years as at 1 April 20X4. The brand is not included in S's
statement of financial position.

Console adjustment to be made for CSOFP as at 31 March 20X5

Solution:

Question 4

H acquired 75% of the OSC of S at 1 April 20X4. S's development project was completed on 30
September 20X4 at a cost of $50 million. $10 million of this had been amortised by 31 March 20X5.

Development costs capitalised by S at the date of acquisition were $18 million. H's directors are of
the opinion that Samson's development costs do not meet the criteria in IAS 38 Intangible Assets for
recognition as an asset.

Console adjustment to be made for CSOFP as at 31 March 20X5

Solution:

301
ACCA Financial Reporting (FR)
CHAPTER 14: IFRS 3 BUSINESS COMBINATION

Question 5

Highmoor, a public listed company, acquired 80% of Slowmoor's ordinary shares on 1 October 20X2.
Highmoor paid an immediate $1.50 per share in cash and agreed to pay a further $0.60 per share in
two years' time if Slowmoor made a profit within two years of its acquisition. Highmoor has not yet
recorded the contingent consideration. The fair value of contingent consideration is to be measured
as the present value of the future cash flow. Highmoor's cost of capital is 10% per annum.

The statements of financial position of the two companies at 30 September 20X3 are shown below:

Highmoor Slowmoor
Non-current assets $ million $ million
Property, plant and equipment 585 172
Investments (note (i)) 225 13
Software (note (ii)) nil 40
810 225
Current assets
Inventory 85 42
Trade receivables 95 36
Tax asset nil 80
Bank 20 nil
200 158
Total assets 1,010 383

Equity and liabilities


Equity:
Ordinary shares of $1 each 400 100
Retained earnings - 1 October 20X2 230 150
- profit/loss for year 100 (35)
730 215
Non-current liabilities
12% loan note nil 35
16% Intercompany loan (note (i)) nil 45
- 80
Current liabilities
Trade payables 210 71
Taxation 70 nil
Overdraft nil 17
280 88
Total equity and liabilities 1,010 383

302
ACCA Financial Reporting (FR)
CHAPTER 14: IFRS 3 BUSINESS COMBINATION

The following information is relevant:

(i) Included in Highmoor's investments is a loan of $50 million made to Slowmoor on 1 April 20X3.
On 28 September 20X3, Slowmoor paid $9 million to Highmoor. This represented interest of $4
million for the year and the balance was a capital repayment. Highmoor had not received nor
accounted for the payment, but it had accrued for the loan interest receivable as part of its
accounts receivable figure. There are no other intra group balances.

(ii) The software was developed by Highmoor during 20X2 at a total cost of $30 million. It was sold
to Slowmoor for $50 million immediately after its acquisition. The software had an estimated
life of five years and is being amortized by Slowmoor on a straight-line basis.

(iii) Highmoor's policy is to value the non-controlling interest using the proportionate share of the
subsidiary's identifiable net assets.

(iv) After the acquisition, Slowmoor sold goods to Highmoor for $15 million on which Slowmoor
made a gross profit of 20%. Highmoor had one third of these goods still in its inventory at 30
September 20X3.

Required:

(a) Prepare the consolidated statement of financial position of Highmoor as at 30 September 20X3,
explaining your treatment of the contingent consideration.

303
ACCA Financial Reporting (FR)
CHAPTER 14: IFRS 3 BUSINESS COMBINATION

(b) Describe the circumstances in which the consideration for an acquisition may be less than the
share of the assets acquired. Your answer should refer to the particular issues of the above
acquisition.

Solution:

The consideration given for a business may be less than the fair value of the net assets acquired.
Intuitively it does not make sense for a vendor to sell net assets for less than they are worth. This
view is reflected by the IASB which is rather sceptical about the existence of what is often described
as negative goodwill (bargain purchase). They say where an acquisition appears to create negative
goodwill, a careful check of the value of the assets acquired and whether any liabilities have been
omitted is required.

The consideration may be less than the net assets acquired for several reasons:

- the most obvious is that there has been a bargain purchase through:
- the vendor being in a poor financial position and needing to realize assets quickly, or
- may be due to good negotiating skills on the part of the acquirer, or
- the vendor may not realize how much the assets are really worth.

In relation to the acquisition of Slowmoor the following are questionable issues:

- Highmoor may be trying to deliberately create losses at Slowmoor to avoid paying the further
consideration. An example of this may be the transfer price of the software.

- The tax asset of Slowmoor may be questionable. Accounting standards (IAS 12) are quite
restrictive over the recognition of tax assets.

304
ACCA Financial Reporting (FR)
CHAPTER 14: IFRS 3 BUSINESS COMBINATION

14.12 Chapter 14 Summary


Diagram 14.12.: Summary of IFRS 3 Business Combination

305
ACCA Financial Reporting (FR)
CHAPTER 15: CONSOLIDATED FINANCIAL STATEMENTS

CHAPTER 15: CONSOLIDATED


FINANCIAL STATEMENTS
LEARNING OUTCOME
At the end of the chapter, you should be able to:
TLO D2a. Prepare a consolidated statement of financial position for a simple group (parent and
one subsidiary and associate) dealing with pre and post-acquisition profits, non-
controlling interests and consolidated goodwill.

TLO D2b. Prepare consolidated statement of profit or loss and other comprehensive income for a
simple group dealing with an acquisition in the period and non-controlling interest.
TLO D2c. Explain and account for other reserves (e.g. share premium and revaluation surplus).

TLO D2d. Account for the effects in the financial statements of intra-group trading.

TLO D2e. Account for the effects of fair value adjustments (including their effect on consolidated
goodwill) to:
i. depreciating and non-depreciating non-current assets
ii. inventory
iii. monetary liabilities
iv. assets and liabilities not included in the subsidiary’s own statement of financial
position, including contingent assets and liabilities

TLO D2f. Account for goodwill impairment


.
TLO D2g. Describe and apply the required accounting treatment of consolidated goodwill.

TLO D2h. Explain and illustrate the effect of the disposal of a parent’s investment in a subsidiary
in the parent’s individual financial statements and/or those of the group (restricted to
disposals of the parent’s entire investment in the subsidiary).

306
ACCA Financial Reporting (FR)
CHAPTER 15: CONSOLIDATED FINANCIAL STATEMENTS

15.1 Consolidated Statement of Financial Position

Learning Outcome (ACCA Study Guide Area D)


D2a: Prepare a consolidated statement of financial position for a simple group (parent and one
subsidiary and associate) dealing with pre and post-acquisition profits, non-controlling
interests and consolidated goodwill.
D2c: Explain and account for other reserves (e.g. share premium and revaluation surplus)
D2d: Account for the effects in the financial statements of intra-group trading
D2e: Account for the effects of fair value adjustments (including their effect on consolidated
goodwill) to:
i. depreciating and non-depreciating non-current assets
ii. inventory
iii. monetary liabilities
iv. assets and liabilities not included in the subsidiary’s own statement of financial
position, including contingent assets and liabilities
D2g: Describe and apply the required accounting treatment of consolidated goodwill

When a company buys ordinary share capital in a subsidiary company, the investment is recorded
by:

Dr Investment in Subsidiary (IFRS 3 covers computation of Cost of Investment)


Cr Cash/ Bank/ Ordinary Share Capital

In parent’s books (at the date of acquisition)


Extract of Statement of Financial Position $
Non-current asset: Investment in Subsidiary xx (at cost or Fair value in accordance to IFRS 9)

The amount paid is to acquire the net assets of the subsidiary at fair value
Net assets = Asset – Liabilities or Ordinary share capital + pre-reserves at date of acquisition

In preparing the Consolidated Statement of Financial Position, the Investment in Subsidiary (i.e.
$150) must be eliminated against the:

Ordinary share capital + pre-reserves of the subsidiary (i.e. $100).


The difference of $50 is GOODWILL (IFRS 3)

307
ACCA Financial Reporting (FR)
CHAPTER 15: CONSOLIDATED FINANCIAL STATEMENTS

Pre-acquisition reserves – reserves exist in a subsidiary at acquisition date

Post-acquisition reserves – reserves made after acquisition (included in Consolidated Reserves)

Basic Consolidation:
Step 1: Eliminate cost of investment against Subsidiary’s ordinary share capital + Pre-
Reserve (to calculate goodwill)
Step 2: Adding ALL assets and liabilities line by line (full consolidation basis)

Illustration for pre and post reserves:

DOA SOFP

(Date of acquisition) (reporting date)

31 Dec 2016 31 Dec 2017 pre = $100


Reserve $100 Reserve $150 post = $50

308
ACCA Financial Reporting (FR)
CHAPTER 15: CONSOLIDATED FINANCIAL STATEMENTS

15.1.1 Steps in preparing Consolidated Statement of Financial Position

i. Determine group structure


• Simple group (exam focus)
• Horizontal group
• Vertical group

ii. Determine pre and post-acquisition reserve of subsidiary (Net asset table)
Net Assets of Date of Date of Acquisition
Post Reserve
Subsidiary Reporting (DOR) (DOA)

Ordinary share capital

Retained earnings

iii. Elimination of Cost of Investment in Subsidiary to Calculate Goodwill


FULL goodwill/ Fair Value Method $’000
Fair Value of Consideration Paid
+ Fair Value of Non-controlling interest
- Fair Value of Net Assets acquired
Goodwill at acquisition date

iv. Calculate Consolidated Retained Earnings


Holding's Retained Earnings x
+ Subsidiary's adjusted post retained earnings x Holding % x

v. Non-controlling interest (NCI)


Fair value of non-controlling interest x
+ Subsidiary's adjusted post reserves x Non-controlling interest % x

vi. Adding all assets and liabilities on line by line basis (full consolidation basis)
Include a line for goodwill and non-controlling interest (refer to scenario 1 to 4)

309
ACCA Financial Reporting (FR)
CHAPTER 15: CONSOLIDATED FINANCIAL STATEMENTS

15.1.2 Fair Value Adjustment


15.1.84

IFRS 3 requires the Subsidiary’s assets and liabilities to be recorded at Fair Value for the purpose of
goodwill computation

Situation 1 Situation 2
Both parent and Subsidiary have already
Subsidiary has yet to revalue its non-current asset
revalued its non-current assets in individual
in individual books
books
 
There will be a revaluation surplus in individual
Carried at depreciated Historical Cost
Statement of Financial Position
at Date of acquisition (pre period)
(refer scenario 5)

IFRS 3 requires that the Subsidiary's assets and
liabilities are recorded at fair value for the purposes
of the goodwill computation.

Revalue non-current assets of Subsidiary on date of
acquisition

Loss of Revaluation Gain on Revaluation

Dr Pre Retained Earnings of Dr Net Book Value


Subsidiary / Net assets of Cr Pre Revaluation Surplus of
Subsidiary (affects goodwill Subsidiary / Net assets of
computation) Subsidiary (affects goodwill
Cr Net Book Value computation)

Subsequent changes in Fair value of non-current assets (post period)


*Check if the group has a policy of carrying non-current assets at fair value (Revaluation model)

if YES
make fair value adjustment for consolidation purposes

Loss on revaluation Gain on revaluation


Dr Post Revaluation Surplus/Retained Dr Net book value
Earnings of Subsidiary Cr Post Revaluation Surplus of
Cr Net book value Subsidiary

Additional depreciation for the increased in value if non-current asset in both pre & post
periods
= Fair value profit x depreciation % x no of years since revaluation


only affect post period Dr Post Retained earnings of Subsidiary
Cr Net book value

310
ACCA Financial Reporting (FR)
CHAPTER 15: CONSOLIDATED FINANCIAL STATEMENTS

15.1.3 Intercompany Transactions


15.1.85

When preparing consolidated financial statements, the group (Parent + Subsidiary) is treated as a
single economic entity

From a group point of view, NO transactions occur when transactions are traded between group of
company

This implies that all transaction between parent & Subsidiary must be eliminated to prepare
consolidated financial statements

i. Inter-company loan
Illustration:

Statement of Financial parent Subsidiary Consolidated Statement of $


Position as at 31 Dec 2017 Financial Statement as at 31
Dec 2017

CA $ $ CA

Receivables 150 100 Receivables

CL CL

Payables 600 250 Payables

Additional Information: Loan payable of Subsidiary includes $100 due to Parent.

ii. Inter-company payables and receivables or current accounts


Illustration 1
Statement of Financial Parent Subsidiary Consolidated Statement of $
Position as at 31 Dec 2017 Financial Position as at 31
Dec 2017

CA $ $ CA
Receivables 200 150 Receivables
CL CL
Payables 350 250 Payables

Additional Information: Payables of parent includes $150 due to Subsidiary & Payables of Subsidiary
includes $100 due to parent.

311
ACCA Financial Reporting (FR)
CHAPTER 15: CONSOLIDATED FINANCIAL STATEMENTS

Sometimes current accounts may not agree at the year end, due to:

GOODS IN-TRANSIT CASH IN-TRANSIT


Adjustment to be made to the book of recipient:
Dr Inventory Dr Cash
Cr Payables Cr Receivables

Once the current account's balance agreed, it will cancel out each other

Illustration 2

Statement of Financial Position as at 31 Dec 2017

Current Asset Parent ($) Subsidiary ($)


Inventory 200 80
Subsidiary's current account 450 -
Cash 80 50

Current Liabilities
Parent's current account - 250

Consolidated statement of Financial Position as at 31 Dec 2017

Current Asset ($)


Inventory
Cash

Current Liabilities

Additional info: The difference is due to cash in transit of $100 and goods in transit of $100.

Dr Cash Dr Inventory
Cr Receivables Cr Payables

312
ACCA Financial Reporting (FR)
CHAPTER 15: CONSOLIDATED FINANCIAL STATEMENTS

Example 15.1.3 (a):

S's trade payable account (in records of H) of $7 million does not agree with H's trade receivable
account (in records of S) due to cash in transit of $4 million paid by H.

Consol adjustment:

Example 15.1.3 (b):

S's trade receivables at 30 September 20X8 include $600,000 due from H which did not agree with
H's corresponding trade payable. This was due to cash in transit of $200,000 from H to S.

Consol adjustment:

iii. Inter-company interest


Illustration:
Statement of Financial Parent Subsidiary Consolidated Statement of $
Position as at 31 Dec 2015 Financial Position as at 31 Dec
2017
Non-current asset $ $ Non-current asset
10% Debentures 300 10% Debentures

Current Asset Current Asset


Debenture interest 30 Debenture interest
receivables receivables

Non-current Liability Non-current Liability


10% Debentures 1,000 10% Debentures

Current Liability Current Liability


Debenture interest payables 100 Debenture interest payables

Additional Information: Subsidiary has issued 10% debentures of $1,000 out of which Holding has
purchased debentures with a nominal value of $300

313
ACCA Financial Reporting (FR)
CHAPTER 15: CONSOLIDATED FINANCIAL STATEMENTS

iv. Inter-company dividend (pre and post dividend)

(I) DIVIDENDS PAID FROM POST PROFITS

Situation 1 Situation 2
Both parent & Subsidiary have Subsidiary has already recorded the dividend declared but
posted the related dividend parent has not yet recorded the dividend receivable
declared in their individual books 
 Consolidated adjustments to be made
Just eliminate the inter-company Dr Dividend receivable(Parent's share of dividend)
dividend receivable & payable Cr Consolidated Retained Earnings

Eliminate the inter-company dividend receivable & payable

Illustration
Statement of Financial P S Consolidated Statement of Financial $
Position as at 31 Dec 2017 Position as at 31 Dec 2017
Current Asset Current Asset
Dividend receivable 800 Dividend receivable

CL CL
Dividend payable 1000 Dividend payable/
Declared dividend due to non-
controlling interest

Additional Information: parent bought 80% of ordinary shares in Subsidiary on 31 Dec 2009.

314
ACCA Financial Reporting (FR)
CHAPTER 15: CONSOLIDATED FINANCIAL STATEMENTS

(II) DIVIDEND PAID FROM PRE PROFITS

Dividends of Subsidiary may have been paid by the subsidiary out of pre-acquisition profit


Represents a partial return of the investment paid to acquire the Subsidiary
To be deducted from cost of investment but NOT credited to Retained Earnings of parent.

Situation 1 Situation 2
Parent correctly deducted the pre- Parent wrongly credited to its Retained Earnings
acquisition dividend against its cost of (i.e.: Investment in subs still stated at original
investment amount)
(i.e.: Investment in subsidiary already 
stated at reduced amount) Adjust the Pre-Retained Earnings
 Consolidated adjustments to be made
Only adjust the Pre-Retained Earnings Dr Consolidated Retained Earnings
No need to make any adjustment to Cost (only Parent’s % of dividend received)
of Investment Cr Investment in Subsidiary
* Refer to scenario 10

(v) Inter-company sale of inventories (Parent to Subsidiary and Subsidiary to parent)

If parent sells goods to Subsidiary / Subsidiary sells goods to parent at cost price; NO consolidation
adjustment needed in the Consolidated Statement of Financial Position

If transaction takes place at cost + profit  MUST ELIMINATE unrealized profit (URP) on closing
inventory or goods in transit against seller’s book because the profit of this inventory is unrealized
until it is sold to third party.

Always check who sells parent or subsidiary:

Parent to Subsidiary
Subsidiary to parent
Dr Consolidated retained earnings/ retained
Dr Post retained earnings of Subsidiary
earnings of parent
Cr Closing inventory.
Cr Closing inventory

315
ACCA Financial Reporting (FR)
CHAPTER 15: CONSOLIDATED FINANCIAL STATEMENTS

v. Inter-company sale of Non-current assets (parent to Subsidiary and Subsidiary to parent)

If sale takes place at market price > Net book value; there will be profit on disposal for selling company

 Must eliminate unrealized profit net of additional depreciation from sale of non-current assets
against seller’s book

Always check who sells parent or subsidiary:

Parent to Subsidiary
Subsidiary to Parent
Dr Consolidated retained earnings/
Dr Post RE of subs
retained earnings of parent
Cr Non-current asset
Cr Non-current asset

316
ACCA Financial Reporting (FR)
CHAPTER 15: CONSOLIDATED FINANCIAL STATEMENTS

15.1.4 Consolidation scenarios


15.1.86
Scenario 1
Wholly owned subsidiary (100%)
Date of acquisition = date of preparing consolidated SOFP

Statement of Financial Parent Subsidiary Consolidated Statement


Position as at 31 Dec of Financial Position as
2017 at 31 Dec 2017
Non-Current Asset $ $ Non-Current Asset $
Land and building 300 150 Land and building
Investment in 200 - Goodwill
subsidiary

Current Asset Current Asset


Inventory 5 6 Inventory
Receivable 4 2 Receivable
Cash 3 2 Cash
Total Assets 512 160 Total Assets

Equity & Liabilities Equity & Liabilities


Ordinary share capital 300 100 Ordinary share capital
Retained earnings 170 30 Retained earnings

Non-Current Liability 10 5 Non-Current Liability


Current Liability 32 25 Current Liability
Total Equity & Liability 512 160 Total Equity & Liability

Additional Info: Parent bought 100 ordinary shares in Subsidiary on 31 Dec 2017 (date of acquisition)

[Link] group structure S3. Goodwill computation


Fair value of consideration paid
Fair value of non-controlling interest
Fair value of subsidiary's net asset at DOA
Goodwill

S2. Determine pre & post acquisition reserve


S4. Consolidated retained earnings
Post
Net asset of DOR DOA Holding's retained earnings
Retained
subsidiary = + Subsidiary's post retained earnings x
Earnings
Ordinary share Holding's %
capital
Retained earnings

317
ACCA Financial Reporting (FR)
CHAPTER 15: CONSOLIDATED FINANCIAL STATEMENTS

Scenario 2
Wholly owned subsidiary (100%)
Date of acquisition ≠ date of preparing consolidated statement of financial position
H= parent s=subsidiary

Statement of Financial H S Consolidated Statement


Position as at 31 Dec of Financial Position as
2017 at 31 Dec 2017
Non-Current Asset $ $ Non-Current Asset $
Land and building 300 150 Land and building
Investment in 200 - Goodwill
subsidiary

Current Asset Current Asset


Inventory 5 6 Inventory
Receivable 4 2 Receivable
Cash 3 2 Cash
Total Assets 512 160 Total Assets

Equity & Liabilities Equity & Liabilities


Ordinary share capital 300 100 Ordinary share capital
Retained earnings 170 30 Retained earnings

Non-Current Liability 10 5 Non-Current Liability


Current Liability 32 25 Current Liability
Total Equity & Liability 512 160 Total Equity & Liability

Additional Info: H bought 100 OS in S on 31 Dec 2016 when RE stood at 20

[Link] group structure

S2. Determine pre & post acquisition reserve


Post
Net asset of DOR
DOA + Retained
subsidiary =
Earnings
Ordinary
share capital S4. Consolidated retained earnings
Retained Holding's retained earnings
earnings Subsidiary's post retained earnings x
Holding's %

S3. Goodwill computation


Fair value of consideration paid
Fair value of non-controlling
interest
Fair value of subsidiary's net asset
at DOA

318
ACCA Financial Reporting (FR)
CHAPTER 15: CONSOLIDATED FINANCIAL STATEMENTS

Scenario 3
Partially owned subsidiary
Date of acquisition = date of preparing consolidated statement of financial position

Statement of Financial H S Consolidated Statement


Position as at 31 Dec of Financial Position as
2017 at 31 Dec 2017
Non-Current Asset $ $ Non-Current Asset $
Land and building 300 150 Land and building
Investment in 200 - Goodwill
subsidiary

Current Asset Current Asset


Inventory 5 6 Inventory
Receivable 4 2 Receivable
Cash 3 2 Cash
Total Assets 512 160 Total Assets

Equity & Liabilities Equity & Liabilities


Ordinary share capital 300 100 Ordinary share capital
Retained earnings 170 30 Retained earnings
NCI
Non-Current Liability 10 5 Non-Current Liability
Current Liability 32 25 Current Liability
Total Equity & Liability 512 160 Total Equity & Liability

Additional Info: H bought 80 OS in S on 31 Dec 2017. FV of NCI at acquisition date was $50.

S1. Determine group structure S4. Consolidated retained earnings


Holding's retained earnings
Subsidiary's post retained earnings x Holding's %

S2. Determine pre & post acquisition reserve


Post
Net asset of DOR
DOA + Retained S5. Non-controlling interest
subsidiary =
Earnings Fair value at DOA
Ordinary Subsidiary's post retained earnings
share capital
Retained
earnings

S3. Goodwill computation


Fair value of consideration paid
Fair value of non-controlling
interest
Fair value of subsidiary's net asset
at DOA

319
ACCA Financial Reporting (FR)
CHAPTER 15: CONSOLIDATED FINANCIAL STATEMENTS

Scenario 4
Partially owned subsidiary
Date of acquisition ≠ date of preparing consolidated statement of financial position

Statement of Financial H S Consolidated Statement


Position as at 31 Dec of Financial Position as
2017 at 31 Dec 2017
Non-Current Asset $ $ Non-Current Asset $
Land and building 300 150 Land and building
Investment in subsidiary 200 - Goodwill

Current Asset Current Asset


Inventory 5 6 Inventory
Receivable 4 2 Receivable
Cash 3 2 Cash
Total Assets 512 160 Total Assets

Equity & Liabilities Equity & Liabilities


Ordinary share capital 300 100 Ordinary share capital
Retained earnings 170 30 Retained earnings
NCI
Non-Current Liability 10 5 Non-Current Liability
Current Liability 32 25 Current Liability
Total Equity & Liability 512 160 Total Equity & Liability

Additional Info: Holding bought 80 ordinary shares in Subsidiary on 31 Dec 2016 when retained
earnings is 20. Fair value of non-controlling interest at acquisition date was $50.

[Link] group structure S4. Consolidated retained earnings


Holding's retained earnings
Subsidiary's post retained earnings x
Holding's %
S2. Determine pre & post acquisition reserve
Post
Net asset of DOR DOA
Retained
subsidiary = + S5. Non-controlling interest
Earnings
Fair value at DOA
Ordinary share
capital Subsidiary's post retained
earnings
Retained earnings

S3. Goodwill computation


Fair value of consideration paid
Fair value of non-controlling interest
Fair value of subsidiary's net asset at
DOA
Goodwill

320
ACCA Financial Reporting (FR)
CHAPTER 15: CONSOLIDATED FINANCIAL STATEMENTS

Scenario 5

Partially owned subsidiary


Date of acquisition = date of preparing consolidated statement of financial position
Both parent & Subsidiary have already revalued non-current in their individual books (both follow
revaluation method)

Extract of Statement of Financial Position as at Extract of Consolidated Statement of


31 Dec 2017 Financial Position as at 31 Dec 2017
parent Subsidiary
Non-Current Asset ($'000) ($'000) Non-Current Asset $'000
Land and building 400 200 Land and building
Investment in Subsidiary 200 - Goodwill
Current asset 50 40 Current assets
650 24
0
Equity & liability Equity & liability
Ordinary share capital
($1) 300 100 Ordinary share capital
Revaluation surplus 100 50 Revaluation surplus
Retained earning 170 30 Retained earning
Non-controlling interest
Current liabilities 80 60 Current liabilities
650 240

Holding bought 80,000 ordinary shares in Subsidiary on 31 Dec 2016 when retained earning stood
at $20,000 & RS stood at $30,000.
Holding's policy is to value the non-controlling interest at fair value at the DOA and was valued at
$50,000.

321
ACCA Financial Reporting (FR)
CHAPTER 15: CONSOLIDATED FINANCIAL STATEMENTS

Scenario 6 [FV adjustment]

Extract of Statement of Financial Position as Extract of Consolidated Statement of


at 31 Dec 2017 Financial Position as at 31 Dec 2017
Non-Current Parent Subsidiary
Asset ($'000) ($'000) Non-current liability $'000
Land and
building 400 150 Land and building
Investment in
Subsidiary 200 - Goodwill
Current asset 50 40 Current asset
650 190

Equity & liability Equity and liability


Ordinary share
capital ($1) 300 100 Ordinary share capital
Revaluation
surplus 100 - Revaluation surplus
Retained earning 170 30 Retained earning
Non-controlling interest

Current liabilities 80 60
650 190

Parent bought 80,000 ordinary shares in Subsidiary on 31 Dec 2015 when retained earning stood at
$20,000. Parent's policy is to value the non-controlling interest at fair value at the DOA and was valued
at $50,000.

Subsidiary's land and building at the acquisition date had a fair value of $50,000 in excess of its carrying
amount. The depreciation rate for the non-current asset is 20%.

322
ACCA Financial Reporting (FR)
CHAPTER 15: CONSOLIDATED FINANCIAL STATEMENTS

Scenario 7
Consolidated Statement of
Statement of Financial Position as at Financial Position as at 31
31 Dec 2017 Dec 2017
Parent Subsidiary
Non-Current Asset ($) ($) Non-Current Asset $
Land and building 780 600 Land and building
Investment in Subsidiary:
600 ordinary shares 700 - Goodwill
8% Debentures in 8% Debentures in
Subsidiary 100 - Subsidiary

Current Asset Current Asset


Inventory 200 400 Inventory
Debtors 330 300 Debtors
Current account – Current account –
Subsidiary 60 - Subsidiary
Debenture interest Debenture interest
receivable 8 - receivable
Loan advances 200 - Loan advances
Cash 172 144 Cash
Total assets 2,550 1,444 Total assets

Equity and liability Equity and liability


Ordinary share capital @
$1 1,500 800 Ordinary share capital @ $1
General reserve 200 100 General reserve
Retained earnings 300 (100) Retained earnings
Non-controlling interest

Non-Current Liability Non-Current Liability


8% Debentures - 300 8% Debentures

Current Liability Current Liability


Creditors 550 270 Creditors
Loan payable to parent - 20 Loan payable from Holding
Debenture interest payable - 24 Debenture interest payable
Current account - parent - 30 Current account - Holding
Total Equity and Liability 2,550 1,444 Total Equity and Liability

1. There was no balance in the general reserve at the date of acquisition. While the retained
earnings had a credit balance of $200.

2. Fair value of the non-controlling interest at the acquisition date was $150.

3. The difference btw current account is explained by:


▪ Goods in transit $20
▪ Cash in transit $10

4. The trade debtors in parent includes $10 from Subsidiary and trade creditors in parent
includes $5 from Subsidiary

323
ACCA Financial Reporting (FR)
CHAPTER 15: CONSOLIDATED FINANCIAL STATEMENTS

Working for solutions:

324
ACCA Financial Reporting (FR)
CHAPTER 15: CONSOLIDATED FINANCIAL STATEMENTS

Scenario 8 (post-acquisition dividend)

When parent acquired the shares in Subsidiary Ltd, the retained earnings of Subsidiary $2,000 DR.
There was no balance in the general reserve account. Parent has not yet taken credit for the dividend
declared by Subsidiary. The fair value of non-controlling interest at the date of acquisition is $3,000.

Extract of Statement of Financial Extract of Consolidated Statement of


Position as at 31 Dec 2017 Financial Position as at 31 Dec 2017
parent Subsidiary
Non-Current Asset ($) ($) Non-Current Asset
Investment in Subsidiary
- 7500 ordinary share
capital 6,000 - Investment subsidiary
Goodwill
Equity and liability Equity and liability
Ordinary share capital @
$1 15,000 10,000 Ordinary share capital @ $1
General reserve 8,000 6,000 Gen reserve
Retained earning 6,000 3,000 Retained earning
Non-controlling interest

Current Liability Current Liability


Dividend payables 1,500 500 Dividend payables

325
ACCA Financial Reporting (FR)
CHAPTER 15: CONSOLIDATED FINANCIAL STATEMENTS

Scenario 9

Extract of Statement of Financial Position as Consolidated Statement of Financial


at 31 Dec 2017 Position as at 31 Dec 2017
Non-Current
Asset H ($'000) S ($'000) Non-Current Asset $'000
Investment in
Subsidiary 200 - Goodwill

Current Asset Current Asset


Inventory 20 15 Inventory

Equity and liability Equity and liability


Ordinary share
capital 300 100 Ordinary share capital
Retained earning 170 30 Retained earning
Non-controlling interest

1. Holding bought 80,000 ordinary shares in Subsidiary on 31 Dec 2016 when retained earning stood
at $20,000.
2. Holding's policy is to value the non-controlling interest at fair value at the DOA and was valued at
$50,000.
3. During the year, Holding sold goods to S for $30,000. Holding made a mark-up on cost of 25% on
all these sales. At year end, 50% of these goods remained in Subsidiary’s inventory.

326
ACCA Financial Reporting (FR)
CHAPTER 15: CONSOLIDATED FINANCIAL STATEMENTS

Scenario 10

Extract of Statement of Financial Position as Consolidated Statement of Financial


at 31 Dec 2017 Position as at 31 Dec 2017
Non-Current Parent Subsidiary
Asset ($'000) ($'000) NCA $'000
Investment in
Subsidiary 200 - Goodwill

Current Asset Current Asset


Inventory 20 15 Inventory

Equity and
liability Equity and liability
Ordinary share
capital 300 100 Ordinary share capital
Retained
earning 170 30 Retained earning
Non-controlling interest

1. Holding bought 80,000 ordinary shares in Subsidiary on 31 Dec 2016 when retained earnings
stood at $20,000.
2. Holding's policy is to value the non-controlling interest at fair value at the DOA and was valued
at $50,000.
3. During the year, Subsidiary sold goods to Holding for $40,000. Subsidiary made a margin of
25% on all these sales. At year end, 40% of these goods remained in Holding’s inventory.

327
ACCA Financial Reporting (FR)
CHAPTER 15: CONSOLIDATED FINANCIAL STATEMENTS

Scenario 11

Extract of Statement of Financial Consolidated Statement of Financial


Position as at 31 Dec 2017 Position as at 31 Dec 2017
Non-current Holding Subsidiary
Asset ($'000) ($'000) NCA $'000
Motor vehicle 300 400 Motor vehicle
Investment in
Subs 200 - Goodwill

Equity and
liability Equity and liability
Ordinary
share capital 300 100 Ordinary share capital
Retained
earning 170 30 Retained earning
Non-controlling interest

1. Holding bought 80,000 ordinary shares in Subsidiary on 31 Dec 2016 when retained earning
stood at $20,000.
2. Holding's policy is to value the non-controlling interest at fair value at the DOA and was valued
at $50,000.
3. During the year, Holding sold motor vehicle with a book value of $200,000 to Subsidiary for
$300,000. Depreciation is charged 20% p.a.

328
ACCA Financial Reporting (FR)
CHAPTER 15: CONSOLIDATED FINANCIAL STATEMENTS

Scenario 12

Extract of SOFP as at 31 Dec 2017 Extract of CSOFP as at 31 Dec 2017


Non-current
Asset H ($'000) S ($'000) Non-current Asset $'000
Motor vehicle 300 400 Motor vehicle
Investment in
Subs 200 - Goodwill

Equity and
liability Equity and liability
Ordinary
share capital 300 100 Ordinary share capital
Retained
earning 170 120 Retained earning
Non-controlling interest

1. Holding bought 80,000 ordinary shares in Subsidiary on 31 Dec 2016 when retained earning
stood at $20,000.
2. Holding's policy is to value the non-controlling interest at fair value at the DOA and was valued
at $50,000.
3. During the year, Subsidiary sold motor vehicle with a book value of $200,000 to Holding for
$300,000. Depreciation is charged 10% p.a.

329
ACCA Financial Reporting (FR)
CHAPTER 15: CONSOLIDATED FINANCIAL STATEMENTS

15.2 Consolidated Statement of Profit and Loss

Learning Outcome (ACCA Study Guide Area D)


D2b: Prepare consolidated statement of profit or loss and other comprehensive income for a simple
group dealing with an acquisition in the period and non-controlling interest.

D2f: Account for goodwill impairment

15.2.1 Determination of Date of Acquisition


15.1.87

▪ Only records current year’s item [both parent’s and POST acquisition result of Subsidiary only]
▪ Need to determine date of acquisition
E.g. Current year is 1/1/18 to 31/12/18

i. If parent acquire Subsidiary on 1/1/16 (acquire in previous years)


Consolidated Statement of Profit and Loss = 12 months of parent + 12 months of Subsidiary

ii. If parent acquire Subsidiary on 1/1/18 (acquire in beginning of current year)


Consolidated Statement of Profit and Loss = 12 months of parent + 12 months of Subsidiary

iii. If parent acquire Subsidiary on 1/4/18 (acquire during the year)


Consolidated Statement of Profit and Loss = 12 months of parent + 9/12 months of
Subsidiary

iv. If parent acquire Subsidiary on 31/12/18 (acquire at the end of current year)
No Consolidated Statement of Profit and Loss = Only 12 months of parent

15.2.2 Intercompany Transactions


15.1.88
i. Inter-company sales of inventory

Parent to Subsidiary Subsidiary to Parent

Deduct from Deduct from S’s Deduct from Deduct from P’s Cost
P’s revenue Cost of sales S’s revenue of sales

Dr Consolidated Revenue
Cr Consolidated cost of sales

330
ACCA Financial Reporting (FR)
CHAPTER 15: CONSOLIDATED FINANCIAL STATEMENTS

Illustration

Statement of Profit or Parent Subsidiary Consolidated


Loss for the year Statement of Profit or
ended 31 Dec 2010 Loss for the year
ended 31 Dec 2010
$ $ $
Revenue 1,000 800 Revenue
Cost of sales (600) (500) Cost of sales
Gross profit 400 300 Gross profit

Additional info: During the year, sales from parent to Subsidiary were $300.

i. Unrealised profit from sales of inventories remained in closing inventory

Parent to Subsidiary Subsidiary to parent


CSOFP Dr Consolidated Retained earnings Dr POST Retained earnings of S
Cr Closing Stock Cr Closing stock
CSOPL To decrease the profit To decrease the profit
 add the unrealized profit to cost  add the unrealized profit to cost of
of sales sales

Will affect the calculation of profit


attributable to non-controlling interest

Calculation of profit attributable to


NCI
Subs’ POST Profit After Tax x NCI %
Less: URP on closing stock
Less: URP net of additional
depreciation on NCA

331
ACCA Financial Reporting (FR)
CHAPTER 15: CONSOLIDATED FINANCIAL STATEMENTS

Illustration

Statement of Profit H S Consolidated Statement


or Loss for the year of Profit or Loss for the
ended 31 Dec 2017 year ended 31 Dec 2017
$ $ $
Revenue 1,000 800 Revenue
Cost of sales (600) (500) Cost of sales
Gross profit 400 300 Gross profit
(-) Expenses (200) (150) (-) Expenses
Net profit 200 150 Net profit

Attributable to:
Parent
NCI

Additional Info:

Holding owns 80% of ordinary share capital in Subsidiary. During the year, sales from
Subsidiary to Holding were $300.

Subsidiary usually marks up all sales by 20%. At year end, 40% of inventory remained unsold.

332
ACCA Financial Reporting (FR)
CHAPTER 15: CONSOLIDATED FINANCIAL STATEMENTS

iii Inter-company interest


ONLY eliminates up to Parent’s portion of interest receivables

Dr Interest Income
Cr Finance Cost

E.g. Subsidiary issued 10% debentures of $5000, of which Parent purchased debentures with a
nominal value of $1000.

Statement of Profit or H ($) S ($) Consolidated $


Loss for the year ended Statement of Profit or
31 Dec 2010 Loss for the year
ended 31 Dec 2010
Interest income 100 - Interest income
Finance Cost - 500 Finance Cost

iv. Goodwill impairment


Partial Goodwill / Proportion of net asset
Full Goodwill / Fair value method
method
CSOFP

Dr Post retained earnings of subsidiary Dr Consolidated retained earning


Cr Goodwill Cr Goodwill

CSOPL to decrease the profit to decrease the profit

 Add current year imp to admin expense  Add current year imp to admin expense

will affect Calculation of profit attributable to non-controlling interest

Subsidiary's post Net Profit

Less: URP on closing inventory

URP net of additional depreciation on non-current asset

Goodwill impaired

333
ACCA Financial Reporting (FR)
CHAPTER 15: CONSOLIDATED FINANCIAL STATEMENTS

v. Additional deprecation charged for fair value adjustments for subsidiary

Additional depreciation for current year = fair value gain x depreciation rate

CSOFP

Dr Post retained earnings of Subsidiary

Cr Net book value

CSOPL to decrease the profit


--> add current year additional depreciation to Cost of Sales / Admin expenses

will affect Calculation of profit attributable to non-controlling interest


Subsidiary's post Net Profit
Less: URP on closing stock
URP net of additional depreciation on non-current asset
Goodwill impaired
Additional depreciation on non-current asset for fair value adjustment

vi. Inter-company dividend


Subsidiary’s dividend paid/ declared will become Parent’s dividend received/receivable (Parent’s%
only)

 
Do not show in Subsidiary’s statement of Appear as dividend income in Parent’s SOPL
profit or loss

Only in Subsidiary’s statement of Need to eliminate in preparing consolidated


changes in equity, no adjustment in statement of profit or loss
consolidated statement of profit or loss

334
ACCA Financial Reporting (FR)
CHAPTER 15: CONSOLIDATED FINANCIAL STATEMENTS

15.2.3 Full format of Consolidated Statement of Profit & Loss and Other Comprehensive Income
15.1.89

ABC Group
Consolidated Statement of Profit or Loss for the year ended
$
_________

Revenue (Holding + Subsidiary - inter-companies sales [i])


x
Cost of sales (Holding + Subsidiary - inter-co sales [i] + URP on
closing stock [ii] + URP net of additional depreciation on non- (x)
current asset [ii])
Gross profit xx
Other income (Holding + Subsidiary - inter-co interest income
x Full
[iii]) consolidation
Distribution costs (Holding + Subsidiary - inter-companies 100%
(x)
expenses) Holding +
100%
Administrative expenses (Holding + Subsidiary - inter-co Subsidiary -
expenses + Goodwill impaired [iv] + additional depreciation on (x) inter-co
non-current asset fair value adjusted [v]) transaction
Finance cost (Holding + Subsidiary - inter-companies interest
(x)
[iii])
Profit before tax xx
Income tax expense (Holding + Subsidiary only) (x)
Profit for the year A

Other comprehensive income:


Gains on property revaluation (Holding + Subsidiary) x

Income tax on OCI for the year net of tax (Holding + Subsidiary) (x)

xx
Total comprehensive income for the year B

Profit attributable to:


balancing
Owners of the parent
figure
Non-controlling interest (W1) C
A
Total comprehensive income attributable to:
balancing
Owners of the parent
figure
Non-controlling interest (C + (Subsidiary's post other
D
comprehensive income x non-controlling interest's %))
B

335
ACCA Financial Reporting (FR)
CHAPTER 15: CONSOLIDATED FINANCIAL STATEMENTS

15.3 Disposal of Subsidiary

Learning Outcome (ACCA Study Guide Area D)


D2h: Explain and illustrate the effect of the disposal of a parent’s investment in a subsidiary in the
parent’s individual financial statements and/or those of the group (restricted to disposals of
the parent’s entire investment in the subsidiary)

Where a company disposes of a subsidiary in the year, it will have the following impact on its
Consolidated Financial Statement:

I. CONSOLIDATED STATEMENT OF FINANCIAL POSITION


- No assets or liabilities of the subsidiary will be included in the consolidated statement of
financial position at the year end, as the group will no longer have control over these.
Similarly, there will be no non-controlling interest relating to the subsidiary that has been
disposed.

II. CONSOLIDATED STATEMENT OF PROFIT OR LOSS


- The results of the subsidiary must be consolidated in the statement of profit or loss up to the
date of disposal
- Gain/(Loss) on disposal of the subsidiary must be included in the consolidated in the
statement of profit or loss

$ $
Fair value of consideration received (Sale proceeds) X
Less: Carrying amount of subsidiary at date of disposal (DOD)
Fair value of net assets at date of disposal X
Carrying amount of goodwill at date of disposal X
(Goodwill not impaired)
Less: Non-controlling interest at date of disposal (X) XX
(Fair value basis / Net assets basis)
Gain/ (Loss) on disposal XX

The parent individual Financial Statements would also reflect a Gain/ (Loss) on disposal:

Sale proceeds X
Less: Cost of investment (X)
Gain/ (Loss) on disposal XX
Less: Tax (X)
Net G/ (L) on disposal XX

336
ACCA Financial Reporting (FR)
CHAPTER 15: CONSOLIDATED FINANCIAL STATEMENTS

15.4 Check Understanding

Question 1

Wilmslow acquired 80% of the equity shares of Zeta on 1 April 2016 when Zeta’s retained earnings
were $200,000. During the year ended 31 March 2017, Zeta purchased goods from Wilmslow
totalling $320,000. At 31 March 2017, one quarter of these goods were still in the inventory
of Zeta. Wilmslow applies a mark-up on cost of 25% to all of its sales.

At 31 March 2017, the RE of Wilmslow and Zeta were $450,000 and $340,000 respectively.

What would be the amount of RE in Wilmslow’s consolidated SOFP as at 31 March 2017?

A $706,000
B $542,000
C $498,000
D $546,000

Question 2

Brenda Martin
ASSETS $'000 $'000 $'000 $'000
Non-current assets
Property, plant and equipment 75,000 31,901
Investments:
$1 ordinary shares in Martin at cost 34,000 -
Martin loan notes 150 -
109,150 31,901
Current assets
Inventory, at cost 9,750 4,162
Receivables 17,125 11,325
Cash and cash equivalents 3,150 30,025 1,255 16,742
Total assets 139,175 48,643

EQUITY AND LIABILITES


Capital and Reserves
$1 Ordinary shares 77,000 23,150
Retained earnings 35,362 9,538
Total equity 112,362 32,688

Non-current liabilities
10% Loan note - 200
Current liabilities
Payables 16,613 9,500
Tax 10,200 26,813 6,255 15,755
Total liabilities 26,813 15,955
Total equity and liabilities 139,175 48,643

337
ACCA Financial Reporting (FR)
CHAPTER 15: CONSOLIDATED FINANCIAL STATEMENTS

You are presented with the following information for Brenda, a limited liability company, and its
subsidiary, Martin: The following information is also available:

(i) Brenda purchased 18,520,000 $1 ordinary shares in Martin on 1 November 2008. At that date
Martin’s retained earnings were $5,338,000.

(ii) It is group policy to value the non-controlling interest at full fair value. The fair value of the non-
controlling interest at the acquisition date was $7,408,000.

(iii) Brenda owns $150,000 of Martin’s loan notes. The annual interest of $15,000 due to Brenda has
not been paid and is included in Martin’s payables and Brenda’s receivables.

(iv) During the year ended 31 October 2009 Brenda sold goods to Martin for $15,000,000. Brenda
made a profit on these goods of $2,500,000. Martin still has all of these goods in inventory at 31
October 2009.

(v) At 31 October 2009 Martin owed Brenda $3,000,000 for some of the goods that Brenda supplied
during the year.

(vi) All Martin’s dividends of $6,500,000 were paid in the financial year ended 31 October 2009.

Prepare the consolidated statement of financial position as at 31 October 2009.

338
ACCA Financial Reporting (FR)
CHAPTER 15: CONSOLIDATED FINANCIAL STATEMENTS

Question 3

Reprise purchased 75% of Encore for $2,000,000 10 years ago when the balance on its retained
earnings was $1,044,000. The statements of financial position of the two companies as at 31 March
20X4 are as follows:

Reprise Encore
$'000 $'000
Non-current assets
Investment in Encore 2,000 -
Land and buildings 3,350 -
Plant and equipment 1,010 2,210
Motor vehicles 510 345
6,870 2,555
Current assets
Inventories 890 352
Trade receivables 1,372 514
Cash and cash equivalents 89 51
2,351 917
9,221 3,472
Equity
Share capital - $1 ordinary shares 1,000 500
Retained earnings 4,225 2,610
Revaluation surplus 2,500 -
7,725 3,110
Non-current liabilities
10% debentures 500 -
Current liabilities
Trade payables 996 362
9,221 3,472

The following additional information is available:

1. Included in trade receivables of Reprise are amounts owed by Encore of $75,000. The current
accounts do not at present balance due to a payment for $39,000 being in transit at the year end
from Encore.

2. Included in the inventories of Encore are items purchased from Reprise during the year for $31,200.
Reprise marks up its goods by 30% to achieve its selling price.

3. $180,000 of the recognised goodwill arising is to be written off due to impairment losses.

4. Encore shares were trading at $4.40 just prior to acquisition by Reprise.

Required

Prepare the consolidated statement of financial position for the Reprise group of companies as at
31 March 20X4. It is the group policy to value the non-controlling interests at full (or fair) value.

339
ACCA Financial Reporting (FR)
CHAPTER 15: CONSOLIDATED FINANCIAL STATEMENTS

Question 4

The summarised statements of financial position of the two companies at 31 March 20X5 are shown
below:

Highway Samson
$m $m
Tangible non-current assets 420 320
Development costs nil 40
Investments 400 20
820 380
Current assets 133 91
Total assets 953 471

Equity and liabilities


Ordinary shares of $1 each 270 80
Reserves:
Share premium 80 40
Revaluation surplus 45 nil
Retained earnings -1 April 20X4 160 134
- year to 31 March
20X5 190 76
745 330

Non-current liabilities
10% intercompany loan nil 60
Current liabilities 208 81
Total equity and liabilities 953 471

The following information is relevant:

(i) Highway paid $310m for 75% of ordinary shares in Samson on 1 April 20X4.

(ii) Highway has a policy of revaluing land and buildings to fair value. At the date of acquisition
Samson's land and buildings had a fair value $20 million higher than their book value and at 31
March 20X5 this had increased by a further $4 million (ignore any additional depreciation).

(iii) Included in Highway's investments is a loan of $60 million made to Samson at the date of
acquisition. Interest is payable annually in arrears. Samson paid the interest due for the year on
31 March 20X5, but Highway did not receive this until after the year end. Highway has not
accounted for the accrued interest from Samson.

(iv) Samson had established a line of products under the brand name of Titanware. Acting on behalf
of Highway, a firm of specialists, had valued the brand name at a value of $40 million with an
estimated life of 10 years as at 1 April 20X4. The brand is not included in Samson's statement of
financial position.

340
ACCA Financial Reporting (FR)
CHAPTER 15: CONSOLIDATED FINANCIAL STATEMENTS

(v) Samson sold goods to Highway during the year at a profit of $6 million. One-third of these goods
were still in the inventory of Highway at 31 March 20X5.

(vi) An impairment test at 31 March 20X5 on the consolidated goodwill concluded that it should be
written down by $22 million. No other assets were impaired.

(vii) It is the group policy to value the non-controlling interest at full fair value. At the date of
acquisition, the directors estimated the fair value of the non-controlling interest to be $74m.

Required:

(a) Prepare the consolidated statement of financial position as at 31 March 20X5. (20 marks)

(b) Explain why consolidated financial statements are useful to the users of financial statements
(as opposed to just the parent company's separate (entity) financial statements.

Answer:

The main reason for preparing consolidated accounts is that groups operate as a single economic unit,
and it is not possible to understand the affairs of the parent company without taking into account the
financial position and performance of all the companies that it controls.

The parent company's own financial statements only show the original cost of the investment and the
dividends received from the subsidiary. This hides the true value and nature of the investment in the
subsidiary, and, without consolidation, could be used to manipulate the reported results of the parent.
For example:

- The cost of the investment will include a premium for goodwill, but this is only quantified and
reported if consolidated accounts are prepared.

- A controlling interest in a subsidiary can be achieved with a 51% interest. The full value of the assets
controlled by the group is only shown through consolidation when the non-controlling interest is
taken into account.

- The parent company controls the dividend policy of the subsidiary, enabling it to smooth out profit
fluctuations with a steady dividend. Consolidation reveals the underlying profits of the group.

- Over time-the net assets of the subsidiary should increase, but the cost of the investment will stay
fixed and will soon bear no relation to the true value of the subsidiary.

341
ACCA Financial Reporting (FR)
CHAPTER 15: CONSOLIDATED FINANCIAL STATEMENTS

Question 5

The statements of financial position of the two companies at 31 March 20X7 are shown below:

Summer Spring
Assets $m $m
Property, plant and equipment (note (ii)) 640 340
Investments 405 nil
Intellectual property nil 30
1045 370
Current assets
Inventory (note (iii)) 76 22
Trade receivables (note (iii)) 84 44
Bank nil 4
Total assets 1205 440

Equity and liabilities


Equity shares of 25 cents each 375 200
Share premium 150 0
Retained earnings -1 April 20X6 210 120
-year ended 31 March 20X7 90 20
825 340
Non-current liabilities
10% loan notes 120 20
Current liabilities
Trade payables (note (iii)) 190 57
Current tax payable 45 23
Overdraft 25 nil
Total equity and liabilities 1205 440

The following information is relevant:

(i) Summer paid $405m to acquire 600m equity shares in Spring on 1 April 20X6.

(ii) At the date of acquisition the fair values of Spring's net assets were approximately equal to their
carrying amounts with the exception of its properties. These properties had a fair value of $40
million in excess of their carrying amounts which would create additional depreciation of $2
million in the post-acquisition period to 31 March 20X7. The fair values have not been reflected
in Spring's statement of financial position.

(iii) Spring sold Summer goods for $15 million in the post-acquisition period. $5 million of these goods
are included in the inventory of Summer at 31 March 20X7. The profit made by Spring on these
sales was $6 million. Spring's trade payable account (in records of Summer) of $7 million does not
agree with Summer's trade receivable account (in records of Spring) due to cash in transit of $4
million paid by Summer.

342
ACCA Financial Reporting (FR)
CHAPTER 15: CONSOLIDATED FINANCIAL STATEMENTS

(iv) Summer has concluded that the consolidated goodwill has been impaired by $28 million.

(v) It is the group policy to value the non-controlling interest at full fair value. The fair value of the
non-controlling interest at the date of acquisition was estimated at $97m.

Prepare the consolidated statement of financial position of Summer as at 31 March 20X7.

Question 6

You are presented with the following information for Brenda, a limited liability company, and its
subsidiary, Martin:

SOPL for the year ended 31 October 2009

Brenda Martin
$'000 $'000
Revenue 125,000 77,900
Cost of sales (65,000) (38,500)
Gross profit 60,000 39,400
Distribution costs (6,750) (8,050)
Administrative expenses (17,500) (9,780)
Finance costs - (20)
Income from Martin: Loan note interest 15 -
Dividends 5,200 -
Profit before tax 40,965 21,550
Income tax expense -19,250 -10,850
Profit for the year 21,715 10,700

The following information is also available:

(i) Brenda purchased 80% ordinary shares in Martin on 1 November 2008. At that date Martin’s
retained earnings were $5,338,000.

(ii) Brenda owns $150,000 of Martin’s loan notes. The annual interest of $15,000 due to Brenda has
not been paid and is included in Martin’s payables and Brenda’s receivables.

(iii) During the year ended 31 October 2009 Brenda sold goods to Martin for $15,000,000. Brenda
made a profit on these goods of $2,500,000. Martin still has all of these goods in inventory at 31
October 2009.

(iv) All Martin’s dividends of $6,500,000 were paid in the financial year ended 31 October 2009.

Required:

Prepare the consolidated SOPL for the year ended 31 October 2009.

343
ACCA Financial Reporting (FR)
CHAPTER 15: CONSOLIDATED FINANCIAL STATEMENTS

Question 7

The summarised income statements of Big Co and Small Co, for the year ended 31 October 2010, are
provided below. Big Co acquired 3,600,000 ordinary shares in Small Co for $5,250,000 on 1 November
2009 when the retained earnings of Small Co were $300,000. On the same date, Big Co also acquired
40% of Small Co’s loan notes of $400,000.

SOPL for the year ended 31 October 2010

Big Co Small Co
$'000 $'000
Revenue 9,600 3,900
Cost of sales (5,550) (2,175)
Gross profit 4,050 1,725
Distribution costs (1,050) (480)
Administrative expenses (1,650) (735)
Finance costs - (25)
Income from Small Co: Loan note interest 10 -
Dividends from Small Co 150 -
Profit before tax 1,510 485
Income tax expense (600) (120)
Profit for the year 910 365

The following information is also available:

(i) Small Co’s total share capital consists of 6,000,000 ordinary shares of $1 each.

(ii) It is group policy to value the non-controlling interest at full fair value. The fair value of the non-
controlling interest at the acquisition date was $3,200,000.

(iii) During the year ended 31 October 2010, Big Co sold goods costing $200,000 to Small Co for
$300,000. At 31 October 2010, 50% of these goods remained in Small Co’s inventory.

Required:

(a) Calculate the goodwill arising on the acquisition of Small Co.

(b) Prepare the consolidated SOPL for Big Co for the year ended 31 October 2010.

344
ACCA Financial Reporting (FR)
CHAPTER 15: CONSOLIDATED FINANCIAL STATEMENTS

Question 8

Fallowfield acquired a 60% holding in Rusholme three years ago when Rusholme's retained earnings
balance stood at $16,000. Both businesses have been very successful since the acquisition and their
respective statements of profit or loss for the year ended 30 June 20X8 are as follows:

Fallowfield Rusholme
$ $
Revenue 403,400 193,000
Cost of sales (201,400) (92,600)
Gross profit 202,000 100,400
Distribution costs (16,000) (14,600)
Administrative expenses (24,250) (17,800)
Dividends from Rusholme 15,000 -
Profit before tax 176,750 68,000
Income tax expense (61,750) (22,000)
PROFIT FOR THE YEAR 115,000 46,000

Additional information:

During the year Rusholme sold some goods to Fallowfield for $40,000, including 25% mark up. Half of
these items were still in inventories at the year-end.

Required

Produce the consolidated statement of profit or loss of Fallowfield and its subsidiary for the year
ended 30 June 20X8.

345
ACCA Financial Reporting (FR)
CHAPTER 15: CONSOLIDATED FINANCIAL STATEMENTS

Question 9

Panther operated as a single company, but in 20X4 decided to expand its operations. Panther acquired
a 60% interest in Sabre on 1 July 20X4 for $2,000,000.

The statements of profit or loss and other comprehensive income of Panther and Sabre for the year
ended 31 December 20X4 are as follows:

Panther Sabre
$'000 $'000
Revenue 22,800 4,300
Cost of sales (13,600) (2,600)
Gross profit 9,200 1,700
Distribution costs (2,900) (500)
Administrative expenses (1,800) (300)
Finance costs (200) (70)
Finance income 50 -
Profit before tax 4,350 830
Income tax expense (1,300) (220)
PROFIT FOR THE YEAR 3,050 610
Other comprehensive income for the year, net of tax 1,600 180
TOTAL COMPREHENSIVE INCOME FOR THE YEAR 4,650 790

Historically, Sabre had been a significant trading partner of Panther. During 20X4, Panther purchased
$640,000 of goods from Sabre. Of these, $60,000 remained in inventories at the year end. Sabre makes
a mark-up on cost of 20% under the transfer pricing agreement between the two companies. The fair
value of the identifiable net assets of Sabre on purchase were $200,000 greater than their book value.
The difference relates to properties with a remaining useful life of 20 years.

On 1 January 20X4 (to protect its supply lines), Panther had advanced a loan to Sabre amounting to
$800,000 at a market interest rate of 5%. The loan is due for repayment in 20X9.

Required

Prepare the consolidated statement of profit or loss and other comprehensive income for the
Panther Group for the year ended 31 December 20X4.

Assume income and expenses (other than intragroup items) accrue evenly.

346
ACCA Financial Reporting (FR)
CHAPTER 15: CONSOLIDATED FINANCIAL STATEMENTS

Question 10

On 1 April 20X8, Pedantic acquired 60% of the equity share capital of Sophistic in a share exchange of
two shares in Pedantic for three shares in Sophistic. The issue of shares has not yet been recorded by
Pedantic. At the date of acquisition shares in Pedantic had a market value of $6 each. Below are the
summarised draft financial statements of both companies:

STATEMENTS OF PROFIT OR LOSS FOR THE YEAR ENDED 30 SEPTEMBER 20X8


Pedantic Sophistic

$'000 $'000

Revenue 85,000 42,000

Cost of sales (63,000) (32,000)

Gross profit 22,000 10,000

Distribution costs (2,000) (2,000)

Administrative expenses (6,000) (3,200)

Finance costs (300) (400)

Profit before tax 13,700 4,400

Income tax expense (4,700) (1,400)

Profit for the year 9,000 3,000

STATEMENTS OF FINANCIAL POSITION AS AT 30 SEPTEMBER 20X8


Assets Pedantic Sophistic
Non-current assets $'000 $'000
Property, plant and equipment 40,600 12,600
Current assets 16,000 6,600
Total assets 56,600 19,200
Equity and liabilities
Equity shares of $1 each 10,000 4,000
Retained earnings 35,400 6,500
45,400 10,500
Non-current liabilities
10% loan notes 3,000 4,000
Current liabilities 8,200 4,700
Total equity and liabilities 56,600 19,200

347
ACCA Financial Reporting (FR)
CHAPTER 15: CONSOLIDATED FINANCIAL STATEMENTS

The following information is relevant:

i. At the date of acquisition, the fair values of Sophistic's assets were equal to their carrying amounts
with the exception of an item of plant, which had a fair value of $2 million in excess of its carrying
amount. It had a remaining life of five years at that date [straight-line depreciation is used].
Sophistic has not adjusted the carrying amount of its plant as a result of the fair value exercise.
ii. Sales from Sophistic to Pedantic in the post acquisition period were $8 million. Sophistic made a
mark up on cost of 40% on these sales. Pedantic had sold $5.2 million (at cost to Pedantic) of these
goods by 30 September 20X8.
iii. Other than where indicated, profit or loss items are deemed to accrue evenly on a time basis.
iv. Sophistic's trade receivables at 30 September 20X8 include $600,000 due from Pedantic which did
not agree with Pedantic's corresponding trade payable. This was due to cash in transit of $200,000
from Pedantic to Sophistic. Both companies have positive bank balances.
v. Pedantic has a policy of accounting for any non-controlling interest at full fair value. The fair value
of the non-controlling interest in Sophistic at the date of acquisition was estimated to be $5.9m.
Consolidated goodwill was not impaired at 30 September 20X8.

a. Prepare the consolidated statement of profit or loss for Pedantic for they year ended 30
September 20X8.
(9 marks)

b. Prepare the consolidated statement of financial position for Pedantic as at 30 September 20X8.
(16 marks)

348
ACCA Financial Reporting (FR)
CHAPTER 15: CONSOLIDATED FINANCIAL STATEMENTS

Question 11

On 1 July 20X2 Hillusion acquired 80% of the ordinary share capital of Skeptik at a cost of $10,280,000.
On the same date it also acquired 50% of Skeptik's 10% loan notes at par. The summarised draft
financial statements of both companies are:

STATEMENTS OF PROFIT OR LOSS: YEAR TO 31 MARCH 20X3


Hillusion Skeptik
$'000 $'000
Sales revenue 60,000 24,000
Cost of sales (42,000) (20,000)
Gross profit 18,000 4,000
Operating expenses (6,000) (200)
Loan interest received (paid) 75 (200)
Profit before tax 12,075 3,600
Income tax expense (3,000) (600)
Profit for the year 9,075 3,000

STATEMENTS OF FINANCIAL POSITION AS AT 31 MARCH 20X3


Hillusion Skeptik
Assets $'000 $'000
Tangible non-current Assets 19,320 8,000
Investments 11,280 Nil
30,600 8,000
Current assets 15,000 8,000
Total assets 45,600 16,000

Equity and liabilities


Equity
Ordinary shares of $1 each 10,000 2,000
Retained earnings 25,600 8,400
35,600 10,400
Non-current liabilities
10% loan notes Nil 2,000
Current liabilities 10,000 3,600
Total equity and liabilities 45,600 16,000

349
ACCA Financial Reporting (FR)
CHAPTER 15: CONSOLIDATED FINANCIAL STATEMENTS

The following information is relevant:

(i)
The fair values of Skeptik's assets were equal to their book values with the exception of its plant,
which had a fair value of $3.2 million in excess of its book value at the date of acquisition. The
remaining life of all of Skeptik's plant at the date of its acquisition was four years and this period
has not changed as a result of the acquisition. Depreciation of plant is on a straight-line basis
and charged to cost of sales. Skeptik has not adjusted the value of its plant as a result of the fair
value exercise.

(ii) In the post acquisition period Hillusion sold goods to Skeptik at a price of $12 million. These
goods had cost Hillusion $9 million. During the year Skeptik had sold $10 million (at cost to
Skeptik) of these goods for $15 million.

(iii) Revenues and profits should be deemed to accrue evenly throughout the year.

(iv) The current accounts of the two companies were reconciled at the year-end with Skeptik owing
Hillusion $750,000.

(v) The goodwill was reviewed for impairment at the end of the reporting period and had suffered
an impairment loss of $300,000, which is to be treated as an operating expense.

(vi) Hillusion's opening retained earnings were $16,525,000 and Skeptik's were $5,400,000. No
dividends were paid or declared by either entity during the year.

(vii) It is the group policy to value the non-controlling interest at acquisition at fair value. The
directors valued the non-controlling interest at $2.5m at the date of acquisition.

(a) Prepare a consolidated statement of profit or loss and statement of financial position for
Hillusion for the year to 31 March 20X3. (20 marks)

(b) Explain why it is necessary to eliminate unrealised profits when preparing group financial
statements; and how reliance on the entity financial statements of Skeptik may mislead a
potential purchaser of the company.

350
ACCA Financial Reporting (FR)
CHAPTER 15: CONSOLIDATED FINANCIAL STATEMENTS

Answer (b)

Unrealised profits arise when group companies trade with each other. In their own individual company
accounts profits and losses will be claimed on these transactions, and goods bought from a fellow
group company will be recorded at their invoiced cost by the purchaser.

However, consolidated accounts are drawn up on the principle that a group is a single economic entity.
From a group point of view, no transaction occurs when goods are traded between group companies,
and no profits or losses arise. Revenue and profits will only be claimed when the goods are sold onto
a third party from outside of the group.

In this example, Hillusion sold $12m of goods to Skeptik making a profit of $3m. By the year -end,
Skeptik still has $2m of goods bought from Hillusion. This valuation includes an element of profit
($500,000) that has not yet been realised and needs to be eliminated. This will reduce the carrying
value of the inventory to the amount originally paid for them by Hillusion. If unrealised profits were
not eliminated, then groups could boost their profits and asset values by selling goods to each other
at inflated prices.

A future purchaser of Skeptik would obviously review Skeptik's own financial statements. These show
a $3.6m profit before tax, which gives a very healthy 15% net profit on revenues. However, over 60%
of Skeptik's revenue comes from selling goods supplied by Hillusion. The gross profit earned on these
items is $5m ($15m - $10m), which is more than the $4m gross profit for the company as a whole. A
new owner might not get such favourable terms from Hillusion, leaving them with the loss making
products. Nobody would be interested in buying such a business, but this information cannot be
gleaned from the entity's own financial statements.

351
ACCA Financial Reporting (FR)
CHAPTER 15: CONSOLIDATED FINANCIAL STATEMENTS

Question 12

Kent acquired a 70% investment in Mac for $2000 2 years ago. It is group policy to measure non -
current asset at fair value at the date of acquisition. The fair value of non-controlling interest at the
DOA was $800 and the fair value of Mac's net asset was $1900.

The goodwill has not been impaired.

Kent has made a disposal of all his shares in Mac for $3000. The net asset of Mac recorded in the
consolidated financial statements at the date of disposal were $2400.

Required: Calculate the gain or loss on disposal that would be recorded in:

(i) Individual statement of profit or loss


$
Sale proceeds
Less: Cost of investment
G/ (L) on disposal

(ii) Consolidated statement of profit or loss


Sale proceeds
Less: CA of subs at DOD
NA at DOD
GW at DOD
Less: NCI at DOD
G/ (L) on disposal

352
ACCA Financial Reporting (FR)
CHAPTER 15: CONSOLIDATED FINANCIAL STATEMENTS

Question 13

Paris acquired 70% of Eiffel's 1m $1 shares on 1 Jan 20X2 for $2.5m, but decided to sell the entire
holding on 1 Jul 20X5 for $4m. Goodwill had not been impaired during the period. The non-controlling
interest was valued at fair value of $500,000 at the DOA and $620,000 at the DOD. Eiffel had retained
earnings of $600,000 on 1 Jan 20X2 and $900,000 on 1 Jan 20X5. Eiffel made a profit of $200,000
during 20X5. No fair value adjustments were required at acquisition.

Required:

(i) Calculate the gain or loss on disposal of Eiffel in the consolidated statement of profit or loss
Sale proceeds
Less: CA of subs at DOD
NA at DOD
GW at DOD
Less: NCI at DOD
G/ (L) on disposal

(ii) Calculate the gain or loss on disposal of Eiffel in the individual statement of profit or loss
Solution:

Sale proceeds
Less: Cost of investment
G/ (L) on disposal

353
ACCA Financial Reporting (FR)
CHAPTER 15: CONSOLIDATED FINANCIAL STATEMENTS

Question 14

Papa acquired 100% of Son's 1m $1 share capital on 1 Jan 20X1 and sold it on 1 Jan 20X6 for $8m. All
of the goodwill in relation to Son has been impaired in previous years. When Papa acquired Son, Son's
retained earnings were $2m and the retained earnings at disposal were $3m. The fair value of Son's
net assets was equal to their fair value with the exception of Son's property, which had a fair value of
$2m above its carrying amount. The property had a remaining life of 20 years at the DOA.

Calculate the gain or loss on disposal of Son in the consolidated statement of profit or loss

Solution:

Sale proceeds

Less: CA of subs at DOD

NA at DOD

GW at DOD
Less: NCI at DOD

G/ (L) on disposal

Question 15

Prince acquired 80% of Michael's share capital on 1 Jan 20X1 for $15m when Michael's net assets were
$10m. Prince sold all its shares on 1 Jan 20X7 for $16m when Michael's net assets at disposal were
$12m. Michael uses the proportion of net asset method for valuing NCI. There has been no impairment
of goodwill since acquisition

Calculate the gain or loss on disposal of Michael in the consolidated statement of financial position

Solution:

Sale proceeds

Less: CA of subs at DOD

NA at DOD

GW at DOD
Less: NCI at DOD

G/ (L) on disposal

354
ACCA Financial Reporting (FR)
CHAPTER 15: CONSOLIDATED FINANCIAL STATEMENTS

Question 16

Height has held 60% investment in May for several years, using the fair value method to value the
non-controlling interest. A quarter of the goodwill has been impaired. Disposal of this investment has
been made on 31 Oct 20X5. Details are:

Sales proceeds 10,000


Cost of Investment 6,000
FV of NA of May at DOA 2,000
FV of NA of May at DOD 3,000
FV of NCI at DOA 1,000

Calculate the gain or loss on disposal of Michael in the consolidated statement of profit or loss

Solution:

Sale proceeds

Less: CA of subs at DOD

NA at DOD

GW at DOD
Less: NCI at DOD

G/ (L) on disposal

355
ACCA Financial Reporting (FR)
CHAPTER 15: CONSOLIDATED FINANCIAL STATEMENTS

15.5 Chapter 15 Summary


Diagram 15.5.: Summary of Consolidated Financial Statement

356
ACCA Financial Reporting (FR)
CHAPTER 16: IAS 28 INVESTMENT IN ASSOCIATES

CHAPTER 16: IAS 28 INVESTMENT IN


ASSOCIATES
LEARNING OUTCOME
At the end of the chapter, you should be able to:

TLO A4j. Define an associate and explain the principles and reasoning for the use of equity
accounting

TLO D2a. Prepare a consolidated statement of financial position for a simple group (parent and
one subsidiary and associate) dealing with pre and post-acquisition profits, non-
controlling interests and consolidated goodwill.

357
ACCA Financial Reporting (FR)
CHAPTER 16: IAS 28 INVESTMENT IN ASSOCIATES

16.1 Overview

An associate is an entity, including an unincorporated entity such as partnership, over which the
investor has significant influence and that is neither a subsidiary nor an interest in a joint venture (JV).

Significant influence is the power to participate in the financial and operating policy decisions of the
investee, but is not control or joint control over those policies.

Therefore, a company exercises significant influence over another when it cannot dictate terms yet
the other entity has to consider what it says.

16.2 Formation of associate companies

Learning Outcome (ACCA Study Guide Area A)


A4j: Define an associate and explain the principles and reasoning for the use of equity accounting

- Direct holdings or and indirect through subsidiary’s holding – more than 20% but less than 50% of
the voting rights
- Holdings due to the existence and effect of potential voting rights

However, in some situations, investor still has significant influence even owns less than 20% of the
voting power in the investee, might demonstrate the existence of significant influence:

1. Representative on the board of directors


2. Participation in the policy making process
3. Material transactions between investee and investor
4. The interchange of management personnel
5. The provision of essential technical information

Since H cannot exercise dominant influence, for consolidation purposes, the investment in associates
to be accounted for under the equity method (not full consolidation) in its consolidated FS.

358
ACCA Financial Reporting (FR)
CHAPTER 16: IAS 28 INVESTMENT IN ASSOCIATES

16.3 Equity method

The investment in an associate is initially recognised at cost and the carrying amount is increased or
decreased to recognise the share of the profit or loss of the associate after the date of acquisition.
The share of the profit or loss of the associate is recognised in the investor’s profit or loss.

Extract of Consolidated SOFP Extract of Consolidated SOPL


NCA Expenses
Investment in Associate
Share of Associate’s Profit
(Cost of Investment + X x
(A’s post net profit x H%)
(A’s post reserves x H%))
Goodwill x Profit before tax x

OCI
Gain on revaluation X
Share of associate’s OCI
X
(A’s post OCI x H%)
OCI for the year net of tax X

Note:
The revenue and expenses of associate are not included on a line by line basis in the (CSOPL). The
investor’s share of profit or loss of the associate is consolidated in SOPL ie. One-line consolidation.

The assets and liabilities of associate are not included on a line by line basis in the (CSOFP). It’s a one-
line consolidation of investor’s equity interest in associate.

Example 16.2:

A Ltd purchased 30% of the OSC of B Ltd for $80,000 on 1/1/04. At this date the RE of B Ltd stood at
$50,000. At 31/12/04 the SOFP of B Ltd showed the following.

$’000
Equity share capital 100
Retained earnings 240
340

At what amount should A Ltd’s interest in B Ltd be stated in its CSOFP as at 31/12/04?

359
ACCA Financial Reporting (FR)
CHAPTER 16: IAS 28 INVESTMENT IN ASSOCIATES

16.4 Adjustment

Generally, the associate is considered to be outside the group. NO elimination of balances between
the Parent and Associate.

16.4.1 URP on sale of inventory

H: Parent A: Associate

URP = closing inventory x profit % x H%

Must check who sells parent or associate:

A to H
H to A
H A A H

A made the profit


H made the Closing inventory still with A (do Closing inventory

profit not consolidate A's inventory in still with H
CSOFP) Must reduce from A's
 profit 

Must reduce Must reduce

from H's profit Must reduce from calculation of from closing
Investment in Associate CSOPL inventory
Minus from share
CSOFP of A’s profit In CSOFP
CSOPL
Dr Consolidated RE 
Decrease the
profit of H Cr Invst in Assoc CSOFP
Dr CRE/RE of A
  Cr Inventory
Add URP to Computation of Invst in Assoc
COS
Cost of investment x

+Post reserves of A x H% x CSOPL

- URP on closing inv (H to A) (x) Computation of Share


of A's profit CSOFP
A's post PAT x H%
 x  Dr Consol RE
Example Example / RE of A
- URP on closing inv (A
During the year, H sold goods to A at a SP of $4m. During the year, A sold goods to H atCra Cl
to H (x) SPinv
of
These goods had cost H $2.4m. A had $2.5m of $4m. These goods had cost A $2.4m. H had
these goods still in inventory at the year end. H $2.5m of these goods still in inventory at the
own 30% of A. year end. H own 30% of A.

360
ACCA Financial Reporting (FR)
CHAPTER 16: IAS 28 INVESTMENT IN ASSOCIATES

16.4.2 Impairment of investment in associates

CSOFP Dr Consolidated RE/ RE of A


Cr Investment in Associate

Computation of investment in Assoc $


Cost of investment x
+ Post reserves of A x H% x
- URP on closing inventory (H to A) (x)
- Impairment losses (x)

CSOPL Computation of Share of A's profit $


A's post PAT x H% x
- URP on closing inv (A to H) (x)
- Impairment losses (x)

16.4.3 Dividends received/receivables from A

Consolidated Statement of Financial Position


- NO elimination of inter-company dividend between Holdings and Associate

CSOPL - A's dividend paid/declared will become H's dividend received/receivable (H% only)
 
do not appear in A's SOPL Appear as dividend income in H's SOPL

Appear in A's SOCIE Eliminate in preparing CSOPL as it is


subsumed within Share of A’s Net Profit

361
ACCA Financial Reporting (FR)
CHAPTER 16: IAS 28 INVESTMENT IN ASSOCIATES

16.5 Steps in preparing Consolidated Statement of Financial Position

Learning Outcome (ACCA Study Guide Area D)


D2a: Prepare a consolidated statement of financial position for a simple group (parent and one
subsidiary and associate) dealing with pre and post-acquisition profits, non-controlling
interests and consolidated goodwill.

S1. Determine the Group Structure


H

S A

S2. Determine Pre and Post acquisition reserve

Net Assets of Subs Date of Reporting DOR) Date of Acq DOA) Post Reserve
OSC
RE

S3. Elimination of Cost of Investment in Subs in order to calculate GW


FULL GW/ FV Method $’000
FV of Consideration Paid
+ FV of NCI
- FV of NA acquired
GW at acquisition date

S4. Cal. Consol RE


Consolidated RE $'000
H's RE x
+ S's adjusted post RE x H% x
+ A's post RE x H% x
- URP on closing inventory (H to S only) (x)
- URP net of additional depreciation on NCA (H to S only) (x)
- GW impairment (partial GW only) (x)
- URP on closing inventory (btw H & A) x H% (x)
- Impairment of Associate (x)

S5. Cal. NCI


FV of NCI x
+ S's adjusted post reserves x NCI% x

S6.
Investment in associates $'000
Cost of investment x
+ Post RE of A x H% x
- URP on closing inventory (H to A only) x H% (x)
- Impairment of associate (x)
xx

362
ACCA Financial Reporting (FR)
CHAPTER 16: IAS 28 INVESTMENT IN ASSOCIATES

S7. Adding ALL A & L on line by line (full consolidation basis)


 To include a line for GW, NCI and Investment in Associate

363
ACCA Financial Reporting (FR)
CHAPTER 16: IAS 28 INVESTMENT IN ASSOCIATES

16.6 Check Understanding

Question 1

P owns 80% of S and 40% of A. Statements of financial position of the three companies at 31 December
20X8 are as follows.

P S A
Investment: shares in S 800
Investment: shares in A 600
Sundry assets 6,600 5,800 5,400

Equity & liabilities


Share capital of $1 each 1,000 400 800
Retained earnings 4,000 3,400 3,600
5,000 3,800 4,400
Liabilities 3,000 2,000 1,000
8,000 5,800 5,400

P acquired its shares in S when S’s retained earnings were $520, and P acquired its shares in A when
A’s retained earnings were $400. The fair value of shares held by the non-controlling interest of S is
$2.40 each.

Prepare the consolidated SOFP at 31 December 20X8

Solution:
S3: Calculation of goodwill
Investment in S shares
FV of NCI
Less: FV of sub’s NA at DOA
Share capital
RE

S4: Consolidated RE
Holding’s RE
Share of subsidiary’s post RE (H%)
Share of associate’s post RE

S5: NCI
FV at [Link]
Share of sub’s post RE (NCI%)

S6: Investment in associates


Cost of investment
Share of associate’s post RE

364
ACCA Financial Reporting (FR)
CHAPTER 16: IAS 28 INVESTMENT IN ASSOCIATES

P Consolidated SOFP as at 31 December 20X8


$
Intangible asset – Goodwill
Investment in associates

Sundry assets (6,600 + 5,800)


Total Assets

Equity & liabilities


Equity attributable to owners of parent:
Share capital
Retained earnings
Non-controlling interests

Liabilities (3,000 + 2,000)


Total equity and liabilities

365
ACCA Financial Reporting (FR)
CHAPTER 16: IAS 28 INVESTMENT IN ASSOCIATES

Question 2
P owns 80% of S and 40% of A for several years. Statements of profit or loss for the year ended 31
December 20X8 are as follows.
P S A
$ $ $
Revenue 14,000 12,000 10,000
Cost of sales (9,000) (4,000) (3,000)
Gross profit 5,000 8,000 7,000
Administrative expenses (2,000) (6,000) (3,000)
Profit from operations 3,000 2,000 4,000
Investment income (not intra-group) 1,000 - 400
Profit before tax 4,000 2,000 4,400
Income tax expenses (1,000) (1,200) (2,000)
Profit for the year 3,000 800 2,400

Prepare the consolidated statement of profit or loss for the year ended 31 December 20X8.
Solution:

$
Revenue
Cost of sales
Gross profit
Administrative expenses
Investment income
Share of profit of associates
Profit before tax
Income tax expense
Profit for the year

Profit attributable to:


Equity of parent
NCI

366
ACCA Financial Reporting (FR)
CHAPTER 16: IAS 28 INVESTMENT IN ASSOCIATES

Question 3
The summarised statements of financial position of the three companies at 31 Dec 20X9 are shown
below:

Papa Mama Aunty


ASSETS $'000 $'000 $'000
Non-current assets
Tangible assets 1,120 980 840
Investments:
672,000 shares in Mama 644 - -
168,000 shares in Aunty 224 - -
1,988 980 840
Current assets
Inventory, at cost 380 640 190
Receivables 190 310 100
Cash and cash equivalents 35 58 46
Total assets 2,593 1,988 1,176

EQUITY AND LIABILITES


Capital and Reserves
$1 Ordinary shares 1,120 840 560
Retained earnings 1,232 602 448
Total equity 2,352 1,442 1,008

Current liabilities
Payables 150 480 136
Tax 91 66 32
Total equity and liabilities 2,593 1,988 1,176

The following information is also available:


(i) Papa acquired its shares in Mama on 1 Jan 20X9 when Mama had retained losses of $56,000.
(ii) Papa acquired its shares in Aunty on 1 Jan 20X9 when Aunty had retained earnings of $140,000.

(iii) An impairment test at the year-end shows that goodwill for Mama remains unimpaired and the
investment in Aunty impaired by $2,800.
(iv) The Papa group values the NCI at full value. The fair value on 1 Jan 20X9 was $157,000.

Required:

Prepare the consolidated statement of financial position as at 31 Dec 20X9 (incorporating its
associated company in accordance with IAS 28).

367
ACCA Financial Reporting (FR)
CHAPTER 16: IAS 28 INVESTMENT IN ASSOCIATES

Question 4
Laurel acquired 80% of the ordinary share capital of Hardy for $160m and 40% of the ordinary share
capital of Comic for $70m on 1 January 20X7 when the retained earnings balances were $64m in Hardy
and $24m in Comic. Laurel, Comic and Hardy are public limited companies.
The statements of financial position of the three companies at 31 December 20X9 are set out below:

Laurel Hardy Comic


$'m $'m $'m
Non-current assets
Property, plant and equipment 220 160 78
Investments 230 - -
450 160 78
Current assets
Inventories 384 234 122
Trade receivables 275 166 67
Cash at bank 42 10 34
701 410 223
1,151 570 301
Equity
Share capital - $1 ordinary shares 400 96 80
Share premium 16 3 -
Retained earnings 278 128 97
694 227 177
Current liabilities
Trade payables 457 343 124
1,151 570 301
You are also given the following information
(i) On 30 November 20X9 Laurel sold some goods to Hardy for cash for $32m These goods had
originally cost $22m and none had been sold by the year-end. On the same date Laurel also sold
goods to Comic for cash for $22m. These goods originally cost $1 Om and Comic had sold half by
the year end.

(ii) On 1 January 20X7 Hardy owned some items of equipment with a book value of $45m that had a
fair value of $57m. These assets were originally purchased by Hardy on 1 January 20X5 and are
being depreciated over 6 years.

(iii) Group policy is to measure non-controlling interests at acquisition at fair value. The fair value of
the non-controlling interests in Hardy on 1 January 20X7 was calculated as $39m.

(iv) Cumulative impairment losses on recognised goodwill amounted to $15m at 31 December 20X9.
No impairment losses have been necessary to date relating to the investment in the associate.

Required:
Prepare a consolidated statement of financial position for Laurel and its subsidiary as at 31
December 20X9, incorporating its associate in accordance with IAS 28.

368
ACCA Financial Reporting (FR)
CHAPTER 16: IAS 28 INVESTMENT IN ASSOCIATES

Question 5
Below are the statements of profit or loss and other comprehensive income of Tyson, its subsidiary
Douglas and associate Frank at 31 December 20X8. Tyson, Douglas and Frank are public limited
companies.

Tyson Douglas Frank


$'m $'m $'m
Revenue 500 150 70
Cost of sales (270) (80) (30)
Gross profit 230 70 40
Other expenses (150) (20) (15)
Finance income 15 10 -
Finance costs (20) - (10)
Profit before tax 75 60 15
Income tax expense (25) (15) (5)
PROFIT FOR THE YEAR 50 45 10
Other comprehensive income:
Gains on property revaluation, net of tax 20 10 5
TOTAL COMPREHENSIVE INCOME FOR THE YEAR 70 55 15

The following information is relevant:

(i) Tyson acquired 80m shares in Douglas for $188m 3 years ago when Douglas had a credit balance
on its reserves of $40m. Douglas has 100m $1 ordinary shares.
(ii) Tyson acquired 40m shares in Frank for $60m 2 years ago when that company had a credit
balance on its reserves of $20m. Frank has 100m $1 ordinary shares.

(iii) During the year Douglas sold some goods to Tyson for $66m (cost $48m). None of the goods had
been sold by the year end.
(iv) Group policy is to measure non-controlling interests at acquisition at fair value. The fair value of
the non-controlling interests in Douglas at acquisition was $40m. An impairment test carried out
at the year-end resulted in $15m of the recognised goodwill relating to Douglas being written off
and recognition of impairment losses of $2.4m relating to the investment in Frank.

Required:
Prepare the consolidated statement of profit or loss and other comprehensive income for the year
ended 31 December 20X8 for Tyson, incorporating its associate.

369
ACCA Financial Reporting (FR)
CHAPTER 16: IAS 28 INVESTMENT IN ASSOCIATES

Question 6
On 1 April 20X9 Polly purchased 80% of the equity shares in Sally. On the same date Polly acquired
40% of the equity shares in Ally paying $2 per share.
The summarised statements of profit or loss for the three companies for the year ended 30 September
20X9 are:

Polly Sally Ally


$'000 $'000 $'000
Revenue 210,000 150,000 50,000
Cost of sales (126,000) (100,000) (40,000)
Gross profit 84,000 50,000 10,000
Distribution costs (11,200) (7,000) (5,000)
Administrative expenses (18,300) (9,000) (11,000)
Investment income (interest and dividends) 9,500 - -
Finance costs (1,800) (3,000) -
Profit (loss) before tax 62,200 31,000 (6,000)
Income tax (expense) relief (15,000) (10,000) 1,000
Profit (loss) for the year 47,200 21,000 (5,000)

The following information is relevant:


(i) The fair values of the net assets of Sally at the date of acquisition were equal to their carrying
amounts with the exception of an item of plant which had a carrying amount of $12 million and a
fair value of $17 million. This plant had a remaining life of five years (straight-line depreciation) at
the date of acquisition of Sally. All depreciation is charged to cost of sales.

(ii) Polly has credited the whole of the dividend it received from Sally of $6.4m to investment income.

(iii) After the acquisition, Polly sold goods to Sally for $15 million on which Polly made a gross profit
of 20%. Sally had one third of these goods still in its inventory at 30 September 20X9.

(iv) The goodwill of Sally has not suffered any impairment; however, due to its losses, the value of
Polly's investment in Ally has been impaired by $3 million at 30 September 20X9.

(v) All items in the above statements of profit or loss are deemed to accrue evenly over the year unless
otherwise indicated.

Required:
Prepare the consolidated statement of profit or loss for the Polly Group for the year ended 30
September 20X9.

370
ACCA Financial Reporting (FR)
CHAPTER 16: IAS 28 INVESTMENT IN ASSOCIATES

16.7 Chapter 16 Summary

Diagram 16.7.: Summary of IAS 28 Investment in Associates

371
ACCA Financial Reporting (FR)
CHAPTER 16: IAS 28 INVESTMENT IN ASSOCIATES

372

You might also like